Re: [obm-l] Divisibilidade, pedido de esclarecimento

2024-03-04 Por tôpico Pedro José
Bom dia!
Mas provar que ocorrendo as duas está certo, não é o que foi pedido.
Pode ser que ocorrendo as duas esteja OK e também que haja pelo menos um
caso, que dá certo para a primeira assertiva e não ocorre para a segunda ou
pode ter pelo menos um caso que ocorra para a segunda e não ocorra para
primeira, já é suficiente para furar.
O certo é:
supor (i) e mostrar que ocorre(ii) e depois provar a volta, supor (ii) e
mostrar que ocorre (i).
(i) 9r + 5s | 17. 17s + 17r | 17 (iii) logo 4*(i)-2(iii) ==>  2r - 14s | 17
(iv).
Como 17s! 17 (v); (1v)+ (v) ==> 2r+3s | 17. Provada a ida. 17
2r +3s |17 (ii) . Mas 17r + 17 s | 17 (iii). (iii)- 4*(i) ==> 9r +5s | 17
Provada a volta.
logo 9r + 5s | 17 <=> 2r+ 3s | 17 C.Q.D.


Cordialmente,
PJMS

Em sáb., 2 de mar. de 2024 às 12:37, Marcone Borges <
marconeborge...@hotmail.com> escreveu:

> Sendo r e s inteiros, mostre que 9r +5s divide 17 se, e somente se, 2r +
> 3s divide 17.
> De 9r + 5s ==0(mod 17), assim como de 2r + 3s ==0(mod17), segue que
> r==7s (mod17). Daí sai a resposta.
> Ou podemos mostrar o que foi pedido usando 9r + 5r +4(2r +3s) = 17(r + s)
> Mas, do ponto de vista de quem elaborou a questão, por que vincular essas
> expressões ao fato de que quando uma for um múltiplo de 17 a outra também
> será?
>
> --
> Esta mensagem foi verificada pelo sistema de antivírus e
> acredita-se estar livre de perigo.
>

-- 
Esta mensagem foi verificada pelo sistema de antiv�rus e
 acredita-se estar livre de perigo.



Re: [obm-l] OBM 88 Problema 6.

2023-12-30 Por tôpico Pedro José
Boa noite!
Cláudio, minha preocupação é com a solução em si da equação.
O problema original pede que demonstre que k é um quadrado perfeito. Todas
soluções que vi são baseadas nas relações de Girad ou Vieta's fórmula como
chamam lá fora.
Eu parti do conhecimento de que k tem de ser quadrado perfeito.
Consegui provar que tirando as soluções triviais a=0 ou b=0 ou a=b=1
b>=raiz(k)
Aí achei a primeira solução para a equação, sem perda de generalidade,
considerei a>b, a=b só ocorre para a=b=1 ou a=b=0. Lá fora acho que nem
consideram 0 natural. Seguem a risca como foi o postulado de Peano.
Então para cada k=w^2 com w>1
Tem um conjunto com uma sequência infinita de soluções.
Sk={si=(ai,bi,k): i natural e i>=1| s1=(w^3,w,w^2) e si+1=(ai*w^2-bi, ai,
w^2).
Consigo provar que todos termos da sequência são soluções.
Não consigo provar que se há uma solução (a*,b*, k*) então (a*,b*,k*) ou
(b*,a*, k*) pertence a sequência Sk para k=w^2.
Eu não acho a solução da equação, só do problema como foi pedido, mostrar
que k é um QP, sem no entanto achar todas as soluções

Cordialmente,
PJMS

Em sex., 29 de dez. de 2023 09:18, Claudio Buffara <
claudio.buff...@gmail.com> escreveu:

> Dá um Google em "IMO 88".
> Vai ter até vídeo com a solução deste problema.
>
> On Thu, Dec 28, 2023 at 4:35 PM Pedro José  wrote:
>
>> Boa tarde!
>> Com referência a esse problema criei uma conjectura, não consegui provar
>> com a pretensão de abranger todas as soluções da equação:
>>
>> (a^2+b^2)/(ab+1)= k, com a,b,k Naturais e a>1, b>1 e k>1 Fiz essa
>> restrição para retirar as soluções triviais.
>> E SPG considerei a>b, já que a=b só ocorre para a=b=1, que está fora pela
>> restrição acima e por ser uma equação simétrica em relação à a e b.
>> O problema era provar que k era um quadrado perfeito.
>> Gostaria de saber se alguém teria conhecimento da resolução em si do
>> problema, i.e., quais ternos (a*,b*,k*) são solução da equação.
>> Caso ninguém tenha resolvido a equação, ainda, gostaria como faço para
>> dar divulgação da minha conjectura, onde tenho a pretenção de ter
>> encontrado todas as soluções possíveis para a equação em epígrafe, no
>> Universo dos Naturais, com a restrição a>1, b>1 e K>1.
>>
>> Agradeço quem puder me orientar.
>>
>> Cordialmente,
>> PJMS
>>
>> --
>> Esta mensagem foi verificada pelo sistema de antivírus e
>> acredita-se estar livre de perigo.
>
>
> --
> Esta mensagem foi verificada pelo sistema de antivírus e
> acredita-se estar livre de perigo.

-- 
Esta mensagem foi verificada pelo sistema de antiv�rus e
 acredita-se estar livre de perigo.



Re: [obm-l] OBM 88 Problema 6.

2023-12-28 Por tôpico Pedro José
Peço máxima vênia. Nem.reparata que fizera uma referência errada. OBM ao
invés de IMO. Interpretei erroneamente como uma censura. Só depois é que
reparei que falhará na referência.
Minhas escusas.

Cordialmente, PJMS.

Em qui., 28 de dez. de 2023 19:47, Anderson Torres <
torres.anderson...@gmail.com> escreveu:

>
>
> Em qui, 28 de dez de 2023 19:01, Pedro José 
> escreveu:
>
>> E daí?
>>
>
> E daí e daí?
>
>
>> Em qui., 28 de dez. de 2023 18:42, Anderson Torres <
>> torres.anderson...@gmail.com> escreveu:
>>
>>> Isso não é da OBM mas da IMO
>>>
>>> Em qui, 28 de dez de 2023 16:35, Pedro José 
>>> escreveu:
>>>
>>>> Boa tarde!
>>>> Com referência a esse problema criei uma conjectura, não consegui
>>>> provar com a pretensão de abranger todas as soluções da equação:
>>>>
>>>> (a^2+b^2)/(ab+1)= k, com a,b,k Naturais e a>1, b>1 e k>1 Fiz essa
>>>> restrição para retirar as soluções triviais.
>>>> E SPG considerei a>b, já que a=b só ocorre para a=b=1, que está fora
>>>> pela restrição acima e por ser uma equação simétrica em relação à a e b.
>>>> O problema era provar que k era um quadrado perfeito.
>>>> Gostaria de saber se alguém teria conhecimento da resolução em si do
>>>> problema, i.e., quais ternos (a*,b*,k*) são solução da equação.
>>>>
>>>
>>> Sim, o próprio método de resolução por descenso provê um método de
>>> listagem das soluções.
>>>
>>> Caso ninguém tenha resolvido a equação, ainda, gostaria como faço para
>>>> dar divulgação da minha conjectura, onde tenho a pretenção de ter
>>>> encontrado todas as soluções possíveis para a equação em epígrafe, no
>>>> Universo dos Naturais, com a restrição a>1, b>1 e K>1.
>>>>
>>>> Agradeço quem puder me orientar.
>>>>
>>>> Cordialmente,
>>>> PJMS
>>>>
>>>> --
>>>> Esta mensagem foi verificada pelo sistema de antivírus e
>>>> acredita-se estar livre de perigo.
>>>
>>>
>>> --
>>> Esta mensagem foi verificada pelo sistema de antivírus e
>>> acredita-se estar livre de perigo.
>>
>>
>> --
>> Esta mensagem foi verificada pelo sistema de antivírus e
>> acredita-se estar livre de perigo.
>
>
> --
> Esta mensagem foi verificada pelo sistema de antivírus e
> acredita-se estar livre de perigo.

-- 
Esta mensagem foi verificada pelo sistema de antiv�rus e
 acredita-se estar livre de perigo.



Re: [obm-l] OBM 88 Problema 6.

2023-12-28 Por tôpico Pedro José
E daí?

Em qui., 28 de dez. de 2023 18:42, Anderson Torres <
torres.anderson...@gmail.com> escreveu:

> Isso não é da OBM mas da IMO
>
> Em qui, 28 de dez de 2023 16:35, Pedro José 
> escreveu:
>
>> Boa tarde!
>> Com referência a esse problema criei uma conjectura, não consegui provar
>> com a pretensão de abranger todas as soluções da equação:
>>
>> (a^2+b^2)/(ab+1)= k, com a,b,k Naturais e a>1, b>1 e k>1 Fiz essa
>> restrição para retirar as soluções triviais.
>> E SPG considerei a>b, já que a=b só ocorre para a=b=1, que está fora pela
>> restrição acima e por ser uma equação simétrica em relação à a e b.
>> O problema era provar que k era um quadrado perfeito.
>> Gostaria de saber se alguém teria conhecimento da resolução em si do
>> problema, i.e., quais ternos (a*,b*,k*) são solução da equação.
>>
>
> Sim, o próprio método de resolução por descenso provê um método de
> listagem das soluções.
>
> Caso ninguém tenha resolvido a equação, ainda, gostaria como faço para dar
>> divulgação da minha conjectura, onde tenho a pretenção de ter encontrado
>> todas as soluções possíveis para a equação em epígrafe, no Universo dos
>> Naturais, com a restrição a>1, b>1 e K>1.
>>
>> Agradeço quem puder me orientar.
>>
>> Cordialmente,
>> PJMS
>>
>> --
>> Esta mensagem foi verificada pelo sistema de antivírus e
>> acredita-se estar livre de perigo.
>
>
> --
> Esta mensagem foi verificada pelo sistema de antivírus e
> acredita-se estar livre de perigo.

-- 
Esta mensagem foi verificada pelo sistema de antiv�rus e
 acredita-se estar livre de perigo.



[obm-l] OBM 88 Problema 6.

2023-12-28 Por tôpico Pedro José
Boa tarde!
Com referência a esse problema criei uma conjectura, não consegui provar
com a pretensão de abranger todas as soluções da equação:

(a^2+b^2)/(ab+1)= k, com a,b,k Naturais e a>1, b>1 e k>1 Fiz essa restrição
para retirar as soluções triviais.
E SPG considerei a>b, já que a=b só ocorre para a=b=1, que está fora pela
restrição acima e por ser uma equação simétrica em relação à a e b.
O problema era provar que k era um quadrado perfeito.
Gostaria de saber se alguém teria conhecimento da resolução em si do
problema, i.e., quais ternos (a*,b*,k*) são solução da equação.
Caso ninguém tenha resolvido a equação, ainda, gostaria como faço para dar
divulgação da minha conjectura, onde tenho a pretenção de ter encontrado
todas as soluções possíveis para a equação em epígrafe, no Universo dos
Naturais, com a restrição a>1, b>1 e K>1.

Agradeço quem puder me orientar.

Cordialmente,
PJMS

-- 
Esta mensagem foi verificada pelo sistema de antiv�rus e
 acredita-se estar livre de perigo.



Re: [obm-l] Conjuntos

2023-09-21 Por tôpico Pedro José
Boa tarde!
Vou considerar 3 números mesmo.
3, 3, 3 é um número só repetido três  vezes.
Os três números obrigatoriamente estarão em P.A. Então usando a menor razão
r <>0;
temos r=1
{1,2,3} {2,3,4}...{2020, 2021, 2022}
{2021, 2022, 2023} temos 2021 conjuntos para r=1.
É fácil observar que para r=2 o último
conjunto será  {2019, 2021, 2023} assim sendo teremos 2019 conjuntos.
E a cada unidade que aumentamos em r diminuímos em 2 o número de conjuntos
Até que chegaremos a um conjunto apenas. {1, 1012, 2023}
Logo o número de conjuntos N será a soma de:
N= 1 + 3 +5+..2019+2021, que é uma PA de razão 2.
seja n o número de termos da PA
n=(2021-1)/2+1=1011
N=(1+2021)*1011/2=1.022.121

Cordialmente,
PJMS


Em ter., 8 de ago. de 2023 19:53, Anderson Torres <
torres.anderson...@gmail.com> escreveu:

> mande uma vez somente.
>
> Em ter, 8 de ago de 2023 12:33, Jamil Silva 
> escreveu:
>
>> Quantos conjuntos de três números inteiros positivos menores ou iguais a
>> 2023 contêm a média aritmética de seus elementos ?
>>
>> --
>> Esta mensagem foi verificada pelo sistema de antivírus e
>> acredita-se estar livre de perigo.
>>
>
> --
> Esta mensagem foi verificada pelo sistema de antivírus e
> acredita-se estar livre de perigo.

-- 
Esta mensagem foi verificada pelo sistema de antiv�rus e
 acredita-se estar livre de perigo.



[obm-l] Re: [obm-l] Re: [obm-l] Caracterização de Inteiros

2022-11-15 Por tôpico Pedro José
Obrigado a você e ao Cláudio. Mas não sou criativo para inventar. Mas já vi
que terei que fazer uma homotetia, para as classes de equivalência para
representar só como um número e não como um par, creio eu.

Cordialmente,
PJMS

Em ter., 15 de nov. de 2022 às 16:00, Anderson Torres <
torres.anderson...@gmail.com> escreveu:

>
>
> Em ter, 15 de nov de 2022 14:33, Pedro José 
> escreveu:
>
>> Boa tarde!
>> Para os |Naturais, temos os postulados de Peano.
>>
>> Para os Inteiros há alguma formalização?
>>
>
> invente uma!
>
> Pode ser por exemplo o conjunto de pares (p,q) tais que p-q é constante.
>
> ou melhor (p1,q1)=(p2,q2) se e só se p1+q2=p2+q1.
>
>
>> Acho pobre dizer que é necessário ter outros números devido ao problema
>> de fechamento nos naturais para a subtração que é fato e daí introduzir os
>> simétricos que são inteiros e ainda não foram caracterizados.
>>
>> No meu antigo ginásio aprendi que os Reais era a união dos conjuntos
>> disjuntos irracionais e racionais. Os racionais haviam sido bem definidos.
>> Aí questionei e o que são irracionais? resposta: são os Reais que não são
>> racionais, os que não podem ser escritos na forma p/q p e q inteiros e
>> q<>0. Mas me deram um tombo. Definiram os |Reais com base nos irracionais e
>> os irracionais com base nos |Reais. 3 +2i também não pode ser inscrito na
>> forma p/q. Só mais tarde no científico, é que meu professor definiu
>> irracional como um número que não podia ser escrito na forma p/q e cuja
>> representação decimal tinha uma infinidade de algarismos, sem haver uma
>> periodicidade.
>> Na época foi o maior nó que tive com a matemática. O mestre demonstrou
>> que os racionais eram densos, mas entre eles ainda cabiam os irracionais.
>> Não satisfeito mostrou que os racionais eram enumeráveis e por absurdo
>> mostrou que os |Reais não. Não satisfeito mostrou que a cardinalidade do
>> intervalo [0,1] era maior que a dos |Naturais. Não conseguia conceber que
>> havia um infinito maior que outro. Outra coisa que demorei a aceitar,mesmo
>> vendo a bijeção, era que os inteiros e naturais tinham a mesma
>> cardinalidade. Na minha cabeça, os inteiros têm todos os naturais ainda
>> sobram os negativos, como é igual?
>> Hoje, depois de velho, arrumei uma enteada, que muito me pergunta e estou
>> enrolado. Para dar um ar de superioridade, questionei se conhecia os
>> inteiros de Gaus, que 5 não era primo nos inteiros de Gaus. Estrepei-me, a
>> danada foi pesquisar e me questiona sobre o que não tenho um domínio pleno.
>> Em suma, como apresentei a ela os postulados de Peano para a
>> caracterização dos Naturais, ela me cobra por algo semelhante para os
>> Inteiros, e não sei responder.
>> HELP! SOCORRO! AU SECOURS! AYUDA! AIUTO! HILFE!
>> Cordialmente,
>> PJMS
>>
>> --
>> Esta mensagem foi verificada pelo sistema de antivírus e
>> acredita-se estar livre de perigo.
>
>
> --
> Esta mensagem foi verificada pelo sistema de antivírus e
> acredita-se estar livre de perigo.

-- 
Esta mensagem foi verificada pelo sistema de antiv�rus e
 acredita-se estar livre de perigo.



[obm-l] Caracterização de Inteiros

2022-11-15 Por tôpico Pedro José
Boa tarde!
Para os |Naturais, temos os postulados de Peano.

Para os Inteiros há alguma formalização?

Acho pobre dizer que é necessário ter outros números devido ao problema de
fechamento nos naturais para a subtração que é fato e daí introduzir os
simétricos que são inteiros e ainda não foram caracterizados.

No meu antigo ginásio aprendi que os Reais era a união dos conjuntos
disjuntos irracionais e racionais. Os racionais haviam sido bem definidos.
Aí questionei e o que são irracionais? resposta: são os Reais que não são
racionais, os que não podem ser escritos na forma p/q p e q inteiros e
q<>0. Mas me deram um tombo. Definiram os |Reais com base nos irracionais e
os irracionais com base nos |Reais. 3 +2i também não pode ser inscrito na
forma p/q. Só mais tarde no científico, é que meu professor definiu
irracional como um número que não podia ser escrito na forma p/q e cuja
representação decimal tinha uma infinidade de algarismos, sem haver uma
periodicidade.
Na época foi o maior nó que tive com a matemática. O mestre demonstrou que
os racionais eram densos, mas entre eles ainda cabiam os irracionais. Não
satisfeito mostrou que os racionais eram enumeráveis e por absurdo mostrou
que os |Reais não. Não satisfeito mostrou que a cardinalidade do intervalo
[0,1] era maior que a dos |Naturais. Não conseguia conceber que havia um
infinito maior que outro. Outra coisa que demorei a aceitar,mesmo vendo a
bijeção, era que os inteiros e naturais tinham a mesma cardinalidade. Na
minha cabeça, os inteiros têm todos os naturais ainda sobram os negativos,
como é igual?
Hoje, depois de velho, arrumei uma enteada, que muito me pergunta e estou
enrolado. Para dar um ar de superioridade, questionei se conhecia os
inteiros de Gaus, que 5 não era primo nos inteiros de Gaus. Estrepei-me, a
danada foi pesquisar e me questiona sobre o que não tenho um domínio pleno.
Em suma, como apresentei a ela os postulados de Peano para a caracterização
dos Naturais, ela me cobra por algo semelhante para os Inteiros, e não sei
responder.
HELP! SOCORRO! AU SECOURS! AYUDA! AIUTO! HILFE!
Cordialmente,
PJMS

-- 
Esta mensagem foi verificada pelo sistema de antiv�rus e
 acredita-se estar livre de perigo.



[obm-l] Re: [obm-l] Re: [obm-l] Questão de probabilidade

2022-06-20 Por tôpico Pedro José
Eu na minha humilde opinião creio que a probabilidade exista quando pode
ser uma coisa ou outra. No caso já é definido o que os animais são. Então
já está tudo errado. A questão seria viável se dessem esses limitantes para
uma criança que pintaria os desenhos dos animais. Aí sim há probabilidade.

Em sáb., 18 de jun. de 2022 03:33, Rogerio Ponce da Silva <
abrlw...@gmail.com> escreveu:

> Ola' Vanderlei e pessoal da lista!
>
> Sem perda de generalidade, podemos imaginar que vamos fazer o seguinte:
>
> - uma pintura preta em um dos caes, escolhido aleatoriamente
>
> - uma pintura "malhada" em um dos animais, escolhido aleatoriamente entre
> os 7 animais nao pintados
>
> - duas pintura pretas, em dois animais, escolhidos aleatoriamente entre os
> 6 animais restantes,
>
> - quatro pinturas brancas nos 4 animais restantes
>
>
> Analisando a afirmacao 04, por exemplo, verificamos que, no segundo passo
> (pintura malhada) existem 4 opcoes de cachorro e 3 opcoes de gato.
>
> Assim, a probabilidade de haver um cachorro malhado (4/7) e' maior que a
> probabilidade de haver um gato malhado (3/7).
> Portanto, a afirmacao 04 esta' correta.
> (e o gabarito esta' errado).
>
> []'s
> Rogerio Ponce
>
>
>
> On Wed, Mar 16, 2022 at 8:08 AM Professor Vanderlei Nemitz <
> vanderma...@gmail.com> wrote:
>
>> Bom dia!
>> Na questão a seguir, do vestibular da UEM, penso que o espaço amostral
>> tem 105 elementos, pois um cachorro é preto (desconsideramos esse). Porém,
>> com esse pensamento, não consigo obter o gabarito, que diz que 02 e 16 são
>> corretas.
>> Alguém poderia ajudar?
>> Muito obrigado!
>>
>> *Em um pet shop há 3 gatos e 5 cães. Sabemos que 3 desses animais são
>> pretos, 4 são brancos e 1 é malhado. Além disso, pelo menos 1 cachorro é
>> preto. Assinale o que for correto. *
>> *01) A probabilidade de haver exatamente 1 cachorro preto é de 1/6. *
>> *02) A probabilidade de haver pelo menos 1 gato branco e pelo menos 2
>> cachorros brancos é de 2/3.*
>> *04) A probabilidade de haver um cachorro malhado é maior do que a
>> probabilidade de haver um gato malhado. *
>> *08) Se um animal for escolhido ao acaso, a probabilidade de ele ser um
>> cachorro preto é de 1/8. *
>> *16) Se um animal for escolhido ao acaso, a probabilidade de ele ser um
>> gato malhado é de 1/16.   *
>>
>> --
>> Esta mensagem foi verificada pelo sistema de antivírus e
>> acredita-se estar livre de perigo.
>
>
> --
> Esta mensagem foi verificada pelo sistema de antivírus e
> acredita-se estar livre de perigo.

-- 
Esta mensagem foi verificada pelo sistema de antiv�rus e
 acredita-se estar livre de perigo.



[obm-l] Re: [obm-l] Re: [obm-l] Dúvida e ajuda.

2022-04-08 Por tôpico Pedro José
Grato a todos!
Já, já tenho de voltar ao trabalho.
Depois dou uma olhada.
Mas achei a demonstração usando casa de pombos, simples e prática.
Já que tem de haver um p/q com pp temos w=x+p/q,
onde x é a parte inteira de w/q, então pq e os restos só podem q-1, uma hora tem de
repetir e aí volta a sequência.
Mas saindo do trabalho dou uma olhada.
Mais uma vez, minha gratidão.

Cordialmente,
PJMS



Em sex., 8 de abr. de 2022 às 13:02, Claudio Buffara <
claudio.buff...@gmail.com> escreveu:

> A volta é fácil também: ao calcular a representação decimal de a/b (a e b
> naturais), nas divisões sucessivas por b só existem b-1 restos possíveis
> (resto = 0 em alguma etapa implica numa decimal finita) e, portanto, após
> não mais do que b-1 divisões, um resto vai se repetir, marcando o início de
> um novo período na representação decimal.
>
> Agora, suponha que  X =
> 0,123456789112233445566778899111222333444555666777888999... seja racional.
> Então existirão n e p naturais tais que, a partir da n-ésima casa decimal
> (1/10^n), os algarismos de X vão se repetir numa sequência com período p.
>
> Mas, pela lei de formação de X, vai existir uma sequência de n+p+1
> algarismos iguais a 1, e esta sequência vai começar após a n-ésima casa
> decimal.
> Ou seja, a sequência vai estar incluída na parte periódica da
> representação decimal de X.
> Mas como o período é p, isso implica que a parte periódica teria que
> ser 111..11 (p algarismos 1) ==> contradição à lei de formação de X.
>
> []s,
> Claudio.
>
>
> On Fri, Apr 8, 2022 at 11:17 AM Pedro José  wrote:
>
>> Bom dia!
>> Posso concluir que um número representado por uma infinidade de
>> algarismos decimais é racional se e somente se tem um período de repetições
>> desses algarismos?
>> A ida é fácil se tiver o período é racional.
>> Já a volta não sei se é verdade e se for há como provar?
>>
>> Meu objetivo primário é saber se:
>> 0,123456789112233445566778899111222333444555666777888999... é racional.
>> As reticências se referem ao aumento de mais um algarismo repetido a cada
>> sequência, ou seja a primeira aparição de 1 será 1, a 2a 11 a 3a 111 e
>> assim sucessivamente, o mesmo vale para os demais algarismos.
>>
>> Alguém poderia me ajudar?
>> Grato,
>> PJMS
>>
>> --
>> Esta mensagem foi verificada pelo sistema de antivírus e
>> acredita-se estar livre de perigo.
>
>
> --
> Esta mensagem foi verificada pelo sistema de antivírus e
> acredita-se estar livre de perigo.

-- 
Esta mensagem foi verificada pelo sistema de antiv�rus e
 acredita-se estar livre de perigo.



[obm-l] Re: Dúvida e ajuda.

2022-04-08 Por tôpico Pedro José
Bom dia!
Última forma!
Achei uma demonstração simples e bela, usando casa dos pombos. Uma hora
haverá de ter repetição, portanto, tem que ter um grupamento de dígitos que
se repita caso seja uma série infinita de algarismos decimais.
Portanto o número é irracional.
Grato!
PJMS

Em sex., 8 de abr. de 2022 às 11:06, Pedro José 
escreveu:

> Bom dia!
> Posso concluir que um número representado por uma infinidade de algarismos
> decimais é racional se e somente se tem um período de repetições desses
> algarismos?
> A ida é fácil se tiver o período é racional.
> Já a volta não sei se é verdade e se for há como provar?
>
> Meu objetivo primário é saber se:
> 0,123456789112233445566778899111222333444555666777888999... é racional. As
> reticências se referem ao aumento de mais um algarismo repetido a cada
> sequência, ou seja a primeira aparição de 1 será 1, a 2a 11 a 3a 111 e
> assim sucessivamente, o mesmo vale para os demais algarismos.
>
> Alguém poderia me ajudar?
> Grato,
> PJMS
>

-- 
Esta mensagem foi verificada pelo sistema de antiv�rus e
 acredita-se estar livre de perigo.



[obm-l] Dúvida e ajuda.

2022-04-08 Por tôpico Pedro José
Bom dia!
Posso concluir que um número representado por uma infinidade de algarismos
decimais é racional se e somente se tem um período de repetições desses
algarismos?
A ida é fácil se tiver o período é racional.
Já a volta não sei se é verdade e se for há como provar?

Meu objetivo primário é saber se:
0,123456789112233445566778899111222333444555666777888999... é racional. As
reticências se referem ao aumento de mais um algarismo repetido a cada
sequência, ou seja a primeira aparição de 1 será 1, a 2a 11 a 3a 111 e
assim sucessivamente, o mesmo vale para os demais algarismos.

Alguém poderia me ajudar?
Grato,
PJMS

-- 
Esta mensagem foi verificada pelo sistema de antiv�rus e
 acredita-se estar livre de perigo.



[obm-l] Re: [obm-l] Re: [obm-l] Dúvida

2021-11-22 Por tôpico Pedro José
Boa tarde!

Grato, pela ajuda!
Não conheço.
Vou abrir um leque de estudo para tentar entender!
Valeu a curiosidade, com o que cheguei consegui matar o problema.
Genericamente, consegui que a solução levaria a uma expressão que era um
quadrado perfeito,esse era o objetivo. Só que me deu curiosidade, quanto a
resolução. Vou me enveredar no tema.

Cordialmente,
PJMS.

Em ter., 16 de nov. de 2021 às 17:29, Prof. Douglas Oliveira <
profdouglaso.del...@gmail.com> escreveu:

> Equação de Pell
>
> Em seg., 15 de nov. de 2021 13:36, Pedro José 
> escreveu:
>
>> Boa tarde!
>>
>> Alguém saberia como resolver a seguinte equação:
>>
>> x^2-7y^2=1, x,y em Z?
>>
>> Fiz a-7b=1 e achei a= 8 +7k e b=1 +K
>> Logo fica fácil que para k=-1 funciona x^2=1 e y^2=0.
>> Também funciona para k=8 x^2=64 e y^2=9.
>> Mas não sei nem como achar mais soluções nem como provar que só são essas.
>> Alguém poderia me dar uma orientação?
>>
>> Cordialmente,
>> PJMS
>>
>> --
>> Esta mensagem foi verificada pelo sistema de antivírus e
>> acredita-se estar livre de perigo.
>
>
> --
> Esta mensagem foi verificada pelo sistema de antivírus e
> acredita-se estar livre de perigo.

-- 
Esta mensagem foi verificada pelo sistema de antiv�rus e
 acredita-se estar livre de perigo.



[obm-l] Dúvida

2021-11-15 Por tôpico Pedro José
Boa tarde!

Alguém saberia como resolver a seguinte equação:

x^2-7y^2=1, x,y em Z?

Fiz a-7b=1 e achei a= 8 +7k e b=1 +K
Logo fica fácil que para k=-1 funciona x^2=1 e y^2=0.
Também funciona para k=8 x^2=64 e y^2=9.
Mas não sei nem como achar mais soluções nem como provar que só são essas.
Alguém poderia me dar uma orientação?

Cordialmente,
PJMS

-- 
Esta mensagem foi verificada pelo sistema de antiv�rus e
 acredita-se estar livre de perigo.



[obm-l] Re: [obm-l] Re: [obm-l] Re: [obm-l] Re: [obm-l] Ajuda em teoria dos números

2020-10-22 Por tôpico Pedro José
Boa tarde!
Na verdade: 2^a=64; a= 6 e y=12.

Em qui., 22 de out. de 2020 às 11:17, Pedro José 
escreveu:

> Bom dia!
> Recebi esse problema hoje: 615 + x^2 = 2^y., para x,y inteiros Não saberia
> fazer, como não soube resolver esse, acima. Mas devido a solução do colega
> Esdras, pensei:"já vi algo parecido".
> Basta restringir y aos pares.
> Se y é ímpar x^2=2 mod3, absurdo então y é par. Logo y=2a, com a inteiro.
> (2^a + x) (2^a-x)= 615= 1*615=3*205=5*123=15*41 e como a soma dos fatores
> necessita ser uma potência de 2, só serve para 123 e 5.
> Logo 2^y=64 e y=6 e x= 59 ou x=-59.
> Uma resolução levou a outra, não pelo talento nato, mas por aprendizado, o
> que é válido.
> Teve uma feita que estava tentando provar que o triângulo órtico, era o
> triângulo de menor perímetro que poderia ser inscrito em um triângulo
> acutângulo. Tentei por geometria analítica e só levando tinta. Tinha
> desistido. Quando me deparei com um problema que não consegui resolver, mas
> que tinha um caminho para a solução. Quando vi o rebatimento feito, pensei
> está resolvido. O curioso, é que, quando desisti, pesquisei na internet e
> não achei nada. Depois que consegui resolver, achei duas soluções, e
> infelizmente e como esperado, a minha não era novidade, era clássica.
> Obrigado, Esdras! Sem a sua solução, certamente, não teria resolvido essa
> última questão.
>
> Cordialmente,
> PJMS
>
> Em sex., 24 de jul. de 2020 às 12:19, Prof. Douglas Oliveira <
> profdouglaso.del...@gmail.com> escreveu:
>
>> Obrigado Claudio e Esdras, fatoração show
>>
>>
>> Em sex., 24 de jul. de 2020 às 11:12, Esdras Muniz <
>> esdrasmunizm...@gmail.com> escreveu:
>>
>>> Se for solução inteira positiva, acho que só tem 3 e 4. Vc supõe spdg x
>>> maior ou igual a y, vê que y=1 não tem solução e x=y tb não. Daí, x>y>1.
>>> Fatorando a expressão, fica: (xy-8-(x-y))(xy-8+(x-y))=15. Como
>>> (xy-8-(x-y))>(xy-8+(x-y))>-2. Temos que ou (xy-8-(x-y))=1 e (xy-8+(x-y))=15,
>>> o que não tem soluções inteiras positivas, ou (xy-8-(x-y))=3 e 
>>> (xy-8+(x-y))=5,
>>> cujas únicas soluções inteiras são x=4 e y=3.
>>>
>>> Em sex, 24 de jul de 2020 10:36, Claudio Buffara <
>>> claudio.buff...@gmail.com> escreveu:
>>>
>>>> Pelo que entendi, a solução é a porção dessa curva algébrica situada no
>>>> 1o quadrante.
>>>> Dá pra fazer isso no Wolfram Alpha, com o comando plot (x*y-7)^2 - x^2
>>>> - y^2 = 0.
>>>>
>>>> []s,
>>>> Claudio.
>>>>
>>>> On Fri, Jul 24, 2020 at 9:58 AM Prof. Douglas Oliveira <
>>>> profdouglaso.del...@gmail.com> wrote:
>>>>
>>>>> Preciso de ajuda para encontrar todas as soluções não negativas da
>>>>> equação
>>>>> (xy-7)^2=x^2+y^2.
>>>>>
>>>>> Desde já agradeço a ajuda
>>>>> Douglas Oliveira
>>>>>
>>>>> --
>>>>> Esta mensagem foi verificada pelo sistema de antivírus e
>>>>> acredita-se estar livre de perigo.
>>>>
>>>>
>>>> --
>>>> Esta mensagem foi verificada pelo sistema de antivírus e
>>>> acredita-se estar livre de perigo.
>>>
>>>
>>> --
>>> Esta mensagem foi verificada pelo sistema de antivírus e
>>> acredita-se estar livre de perigo.
>>
>>
>> --
>> Esta mensagem foi verificada pelo sistema de antivírus e
>> acredita-se estar livre de perigo.
>
>


[obm-l] Re: [obm-l] Re: [obm-l] Re: [obm-l] Re: [obm-l] Ajuda em teoria dos números

2020-10-22 Por tôpico Pedro José
Bom dia!
Recebi esse problema hoje: 615 + x^2 = 2^y., para x,y inteiros Não saberia
fazer, como não soube resolver esse, acima. Mas devido a solução do colega
Esdras, pensei:"já vi algo parecido".
Basta restringir y aos pares.
Se y é ímpar x^2=2 mod3, absurdo então y é par. Logo y=2a, com a inteiro.
(2^a + x) (2^a-x)= 615= 1*615=3*205=5*123=15*41 e como a soma dos fatores
necessita ser uma potência de 2, só serve para 123 e 5.
Logo 2^y=64 e y=6 e x= 59 ou x=-59.
Uma resolução levou a outra, não pelo talento nato, mas por aprendizado, o
que é válido.
Teve uma feita que estava tentando provar que o triângulo órtico, era o
triângulo de menor perímetro que poderia ser inscrito em um triângulo
acutângulo. Tentei por geometria analítica e só levando tinta. Tinha
desistido. Quando me deparei com um problema que não consegui resolver, mas
que tinha um caminho para a solução. Quando vi o rebatimento feito, pensei
está resolvido. O curioso, é que, quando desisti, pesquisei na internet e
não achei nada. Depois que consegui resolver, achei duas soluções, e
infelizmente e como esperado, a minha não era novidade, era clássica.
Obrigado, Esdras! Sem a sua solução, certamente, não teria resolvido essa
última questão.

Cordialmente,
PJMS

Em sex., 24 de jul. de 2020 às 12:19, Prof. Douglas Oliveira <
profdouglaso.del...@gmail.com> escreveu:

> Obrigado Claudio e Esdras, fatoração show
>
>
> Em sex., 24 de jul. de 2020 às 11:12, Esdras Muniz <
> esdrasmunizm...@gmail.com> escreveu:
>
>> Se for solução inteira positiva, acho que só tem 3 e 4. Vc supõe spdg x
>> maior ou igual a y, vê que y=1 não tem solução e x=y tb não. Daí, x>y>1.
>> Fatorando a expressão, fica: (xy-8-(x-y))(xy-8+(x-y))=15. Como
>> (xy-8-(x-y))>(xy-8+(x-y))>-2. Temos que ou (xy-8-(x-y))=1 e (xy-8+(x-y))=15,
>> o que não tem soluções inteiras positivas, ou (xy-8-(x-y))=3 e 
>> (xy-8+(x-y))=5,
>> cujas únicas soluções inteiras são x=4 e y=3.
>>
>> Em sex, 24 de jul de 2020 10:36, Claudio Buffara <
>> claudio.buff...@gmail.com> escreveu:
>>
>>> Pelo que entendi, a solução é a porção dessa curva algébrica situada no
>>> 1o quadrante.
>>> Dá pra fazer isso no Wolfram Alpha, com o comando plot (x*y-7)^2 - x^2 -
>>> y^2 = 0.
>>>
>>> []s,
>>> Claudio.
>>>
>>> On Fri, Jul 24, 2020 at 9:58 AM Prof. Douglas Oliveira <
>>> profdouglaso.del...@gmail.com> wrote:
>>>
 Preciso de ajuda para encontrar todas as soluções não negativas da
 equação
 (xy-7)^2=x^2+y^2.

 Desde já agradeço a ajuda
 Douglas Oliveira

 --
 Esta mensagem foi verificada pelo sistema de antivírus e
 acredita-se estar livre de perigo.
>>>
>>>
>>> --
>>> Esta mensagem foi verificada pelo sistema de antivírus e
>>> acredita-se estar livre de perigo.
>>
>>
>> --
>> Esta mensagem foi verificada pelo sistema de antivírus e
>> acredita-se estar livre de perigo.
>
>
> --
> Esta mensagem foi verificada pelo sistema de antivírus e
> acredita-se estar livre de perigo.


Re: [obm-l] Problema da IMO

2020-09-11 Por tôpico Pedro José
Boa noite!
Atrapalhou meu vinho e o filme que estava assistindo mas consegui. Não
gostei tanto, agora que consegui, é muito trabalhoso.

2= [3(y+1)(z+1)-1]/2yz
yz= 3(yz+2) (i)
z(y-3)= 3y +2 (ii)
y(z-3)=3z+2 (iii)
(i)*(ii) yz(z-3)(y-3)= 9yz+6(y+z)+4 e Voilá: (z-3)(y-3)=11.

Saudações,
PJMS





Em sáb., 12 de set. de 2020 às 00:35, Pedro José 
escreveu:

> Boa noite!
> Fui em uma linha parecida com a primeira solução, embora não visse
> necessidade de mudança de variáveis.
> Mas o b achei sempre por restrição.
> Esse "it implies" e aparece um número fatorado, não consegui captar,
> embora tenha gostado do recurso, já que é bem restritivo.
>
> Sudações,
> PJMS
>
>
> Em sáb., 12 de set. de 2020 às 00:08, Pedro José 
> escreveu:
>
>> Boa noite!
>> Grato, Ralph!
>>
>> Estou estudando a solução. Pelo menos, não me decepcionei. A resposta
>> estava correta,
>>
>> Saudações.
>> PJMS
>>
>> Em sex., 11 de set. de 2020 às 22:33, Ralph Costa Teixeira <
>> ralp...@gmail.com> escreveu:
>>
>>> Essa eh da IMO 1992. Tem uma solucao aqui:
>>> http://sms.math.nus.edu.sg/Simo/IMO_Problems/92.pdf
>>>
>>> On Fri, Sep 11, 2020 at 10:06 PM Pedro José  wrote:
>>>
>>>> Bom dia!
>>>>
>>>> Recebi de um filho de um amigo, um problema que já o fizera.
>>>> (a-1)(b-1)(c-1) | abc-1;  1>>>
>>>> Confesso que desta feita gastei mais tempo que da  primeira vez.
>>>> Curioso, da primeira ,eu pensei, dessa vez, eu tentei lembrar como eu
>>>> resolvera, aí nem lembrava, nem pensava. Apelei para a internet, mas não
>>>> encontrei nada. Mas no fim, recordei o que havia feito.
>>>> (1+1/(a-1))(1+1/(b-1))(1+1/(c-1)) = k, onde k é inteiro.
>>>> vê-se que k>1, e para um dado a k é máximo para b e c mínimos logo
>>>> b=a+1 e c=a+2
>>>> [a(a+1)(a+2)]/[(a-1)(a)(a+1)] > [a(a+1)(a+2)-1]/[(a-1)(a)(a+1)]>=2,
>>>> então (a+2)/(a-1)>2 ==> a <4, a=2 ou a=3.
>>>> O k é máximo para a=2, b=3 e c=4 ==> k <4, logo k=2 ou k=3.
>>>> S.p.g, se a é ímpar (a-1)(b-1)(c-1) é par; então b,c ímpares e k é
>>>> livre.
>>>> S.p.g se a é par abc-1 é ímpar; então b,c são pares e k ímpar.
>>>> a=2, temos  2b(b+1)/[(b-1)b] >3, não usei a restrição de paridade para
>>>> c para facilitar a simplificação. b<5 Logo a=2>>> k é ímpar k=3. Logo c= 8. (2,4,8) é uma solução.
>>>>
>>>> a=3 temos 3b(b+1)/[2(b-1)b] > 2; b<7 e 3>>> para a=3 e b=5. kmax <= (15*7-1)/(2*4*6) <=2;pois k é inteiro.
>>>> 1 k=2 e c= 15. (3,5,15) é a outra solução.
>>>>
>>>> Só agora me apercebi de que c=ab nas duas soluções. Então tentei uma
>>>> nova solução.
>>>> (a-1)(b-1)(c-1) | abc-1 e (a-1)(b-1)(c-1) | abc + c(1 - (a+b)) -ab+
>>>> (a+b) - 1 logo divide a diferença:
>>>> (a-1)(b-1)(c-1) | (a+b) (c-1) + ab -1 - (c-1) logo c-1 | ab-1, então
>>>> ab-1= w(c-1), para algum w inteiro e ab=w(c-1) +1 (i)
>>>> Como a=2 ou a=3
>>>> Se a=2. e w>=2
>>>> Temos por (i) 2b>= 2 (c-1) +1 c-1>=b, logo absurdo.
>>>> Se a=3
>>>> Temos por (i) 3b>= w(c-1)+1; w=3 ==>3b< 3 (c-1) +1 pois c>b
>>>> w=2 ==> 3b =2(c-1) +1 ==> c=(3b+1)/2
>>>> 2(b-1)(3b-1)/2 | 3b(3b+1)/2 -1 ==>  2(b-1)(3b-1)/ | 3b(3b+1) -2 ==>
>>>> 6b^2-8b-2 | 9b^2+3b-1 ==>  6b^2-8b-2 | 3b^2 +11b+ 4
>>>> ==> b <=5. Como b>a=3 ==> b=5 e c= 8, ferindo a paridade.
>>>> Logo ab-1=c-1 ==> ab=c ==> (a-1)(b-1)(c-1) | c^2-1 ==>  (a-1)(b-1) |
>>>> c+1 (a-1)(b-1) |ab+1==> (a-1)(b-1)!a+b
>>>> a=2 ==> 2b-2= 2+b. b=4 e c=ab=8 (2,4,8)
>>>> a=3 ==> 2(b-1) | 3+b ==> 2(b-1) = 3+. b=5 e c=ab=15. (3,5,15),
>>>> Forcei um pouco a barra para mostrar que c=ab.
>>>> Alguém teria uma outra solução, ou um endereço onde se tem as questões
>>>> da IMO e suas resoluções?
>>>>
>>>> Grato!
>>>> Saudações,
>>>> PJMS
>>>>
>>>>
>>>>
>>>> --
>>>> Esta mensagem foi verificada pelo sistema de antivírus e
>>>> acredita-se estar livre de perigo.
>>>
>>>
>>> --
>>> Esta mensagem foi verificada pelo sistema de antivírus e
>>> acredita-se estar livre de perigo.
>>
>>

-- 
Esta mensagem foi verificada pelo sistema de antiv�rus e
 acredita-se estar livre de perigo.



Re: [obm-l] Problema da IMO

2020-09-11 Por tôpico Pedro José
Boa noite!
Fui em uma linha parecida com a primeira solução, embora não visse
necessidade de mudança de variáveis.
Mas o b achei sempre por restrição.
Esse "it implies" e aparece um número fatorado, não consegui captar, embora
tenha gostado do recurso, já que é bem restritivo.

Sudações,
PJMS


Em sáb., 12 de set. de 2020 às 00:08, Pedro José 
escreveu:

> Boa noite!
> Grato, Ralph!
>
> Estou estudando a solução. Pelo menos, não me decepcionei. A resposta
> estava correta,
>
> Saudações.
> PJMS
>
> Em sex., 11 de set. de 2020 às 22:33, Ralph Costa Teixeira <
> ralp...@gmail.com> escreveu:
>
>> Essa eh da IMO 1992. Tem uma solucao aqui:
>> http://sms.math.nus.edu.sg/Simo/IMO_Problems/92.pdf
>>
>> On Fri, Sep 11, 2020 at 10:06 PM Pedro José  wrote:
>>
>>> Bom dia!
>>>
>>> Recebi de um filho de um amigo, um problema que já o fizera.
>>> (a-1)(b-1)(c-1) | abc-1;  1>>
>>> Confesso que desta feita gastei mais tempo que da  primeira vez.
>>> Curioso, da primeira ,eu pensei, dessa vez, eu tentei lembrar como eu
>>> resolvera, aí nem lembrava, nem pensava. Apelei para a internet, mas não
>>> encontrei nada. Mas no fim, recordei o que havia feito.
>>> (1+1/(a-1))(1+1/(b-1))(1+1/(c-1)) = k, onde k é inteiro.
>>> vê-se que k>1, e para um dado a k é máximo para b e c mínimos logo b=a+1
>>> e c=a+2
>>> [a(a+1)(a+2)]/[(a-1)(a)(a+1)] > [a(a+1)(a+2)-1]/[(a-1)(a)(a+1)]>=2,
>>> então (a+2)/(a-1)>2 ==> a <4, a=2 ou a=3.
>>> O k é máximo para a=2, b=3 e c=4 ==> k <4, logo k=2 ou k=3.
>>> S.p.g, se a é ímpar (a-1)(b-1)(c-1) é par; então b,c ímpares e k é livre.
>>> S.p.g se a é par abc-1 é ímpar; então b,c são pares e k ímpar.
>>> a=2, temos  2b(b+1)/[(b-1)b] >3, não usei a restrição de paridade para c
>>> para facilitar a simplificação. b<5 Logo a=2>> é ímpar k=3. Logo c= 8. (2,4,8) é uma solução.
>>>
>>> a=3 temos 3b(b+1)/[2(b-1)b] > 2; b<7 e 3>> para a=3 e b=5. kmax <= (15*7-1)/(2*4*6) <=2;pois k é inteiro.
>>> 1 k=2 e c= 15. (3,5,15) é a outra solução.
>>>
>>> Só agora me apercebi de que c=ab nas duas soluções. Então tentei uma
>>> nova solução.
>>> (a-1)(b-1)(c-1) | abc-1 e (a-1)(b-1)(c-1) | abc + c(1 - (a+b)) -ab+
>>> (a+b) - 1 logo divide a diferença:
>>> (a-1)(b-1)(c-1) | (a+b) (c-1) + ab -1 - (c-1) logo c-1 | ab-1, então
>>> ab-1= w(c-1), para algum w inteiro e ab=w(c-1) +1 (i)
>>> Como a=2 ou a=3
>>> Se a=2. e w>=2
>>> Temos por (i) 2b>= 2 (c-1) +1 c-1>=b, logo absurdo.
>>> Se a=3
>>> Temos por (i) 3b>= w(c-1)+1; w=3 ==>3b< 3 (c-1) +1 pois c>b
>>> w=2 ==> 3b =2(c-1) +1 ==> c=(3b+1)/2
>>> 2(b-1)(3b-1)/2 | 3b(3b+1)/2 -1 ==>  2(b-1)(3b-1)/ | 3b(3b+1) -2 ==>
>>> 6b^2-8b-2 | 9b^2+3b-1 ==>  6b^2-8b-2 | 3b^2 +11b+ 4
>>> ==> b <=5. Como b>a=3 ==> b=5 e c= 8, ferindo a paridade.
>>> Logo ab-1=c-1 ==> ab=c ==> (a-1)(b-1)(c-1) | c^2-1 ==>  (a-1)(b-1) |
>>> c+1 (a-1)(b-1) |ab+1==> (a-1)(b-1)!a+b
>>> a=2 ==> 2b-2= 2+b. b=4 e c=ab=8 (2,4,8)
>>> a=3 ==> 2(b-1) | 3+b ==> 2(b-1) = 3+. b=5 e c=ab=15. (3,5,15),
>>> Forcei um pouco a barra para mostrar que c=ab.
>>> Alguém teria uma outra solução, ou um endereço onde se tem as questões
>>> da IMO e suas resoluções?
>>>
>>> Grato!
>>> Saudações,
>>> PJMS
>>>
>>>
>>>
>>> --
>>> Esta mensagem foi verificada pelo sistema de antivírus e
>>> acredita-se estar livre de perigo.
>>
>>
>> --
>> Esta mensagem foi verificada pelo sistema de antivírus e
>> acredita-se estar livre de perigo.
>
>

-- 
Esta mensagem foi verificada pelo sistema de antiv�rus e
 acredita-se estar livre de perigo.



Re: [obm-l] Problema da IMO

2020-09-11 Por tôpico Pedro José
Boa noite!
Grato, Ralph!

Estou estudando a solução. Pelo menos, não me decepcionei. A resposta
estava correta,

Saudações.
PJMS

Em sex., 11 de set. de 2020 às 22:33, Ralph Costa Teixeira <
ralp...@gmail.com> escreveu:

> Essa eh da IMO 1992. Tem uma solucao aqui:
> http://sms.math.nus.edu.sg/Simo/IMO_Problems/92.pdf
>
> On Fri, Sep 11, 2020 at 10:06 PM Pedro José  wrote:
>
>> Bom dia!
>>
>> Recebi de um filho de um amigo, um problema que já o fizera.
>> (a-1)(b-1)(c-1) | abc-1;  1>
>> Confesso que desta feita gastei mais tempo que da  primeira vez. Curioso,
>> da primeira ,eu pensei, dessa vez, eu tentei lembrar como eu resolvera, aí
>> nem lembrava, nem pensava. Apelei para a internet, mas não encontrei nada.
>> Mas no fim, recordei o que havia feito.
>> (1+1/(a-1))(1+1/(b-1))(1+1/(c-1)) = k, onde k é inteiro.
>> vê-se que k>1, e para um dado a k é máximo para b e c mínimos logo b=a+1
>> e c=a+2
>> [a(a+1)(a+2)]/[(a-1)(a)(a+1)] > [a(a+1)(a+2)-1]/[(a-1)(a)(a+1)]>=2, então
>> (a+2)/(a-1)>2 ==> a <4, a=2 ou a=3.
>> O k é máximo para a=2, b=3 e c=4 ==> k <4, logo k=2 ou k=3.
>> S.p.g, se a é ímpar (a-1)(b-1)(c-1) é par; então b,c ímpares e k é livre.
>> S.p.g se a é par abc-1 é ímpar; então b,c são pares e k ímpar.
>> a=2, temos  2b(b+1)/[(b-1)b] >3, não usei a restrição de paridade para c
>> para facilitar a simplificação. b<5 Logo a=2> é ímpar k=3. Logo c= 8. (2,4,8) é uma solução.
>>
>> a=3 temos 3b(b+1)/[2(b-1)b] > 2; b<7 e 3> para a=3 e b=5. kmax <= (15*7-1)/(2*4*6) <=2;pois k é inteiro.
>> 1 k=2 e c= 15. (3,5,15) é a outra solução.
>>
>> Só agora me apercebi de que c=ab nas duas soluções. Então tentei uma nova
>> solução.
>> (a-1)(b-1)(c-1) | abc-1 e (a-1)(b-1)(c-1) | abc + c(1 - (a+b)) -ab+ (a+b)
>> - 1 logo divide a diferença:
>> (a-1)(b-1)(c-1) | (a+b) (c-1) + ab -1 - (c-1) logo c-1 | ab-1, então
>> ab-1= w(c-1), para algum w inteiro e ab=w(c-1) +1 (i)
>> Como a=2 ou a=3
>> Se a=2. e w>=2
>> Temos por (i) 2b>= 2 (c-1) +1 c-1>=b, logo absurdo.
>> Se a=3
>> Temos por (i) 3b>= w(c-1)+1; w=3 ==>3b< 3 (c-1) +1 pois c>b
>> w=2 ==> 3b =2(c-1) +1 ==> c=(3b+1)/2
>> 2(b-1)(3b-1)/2 | 3b(3b+1)/2 -1 ==>  2(b-1)(3b-1)/ | 3b(3b+1) -2 ==>
>> 6b^2-8b-2 | 9b^2+3b-1 ==>  6b^2-8b-2 | 3b^2 +11b+ 4
>> ==> b <=5. Como b>a=3 ==> b=5 e c= 8, ferindo a paridade.
>> Logo ab-1=c-1 ==> ab=c ==> (a-1)(b-1)(c-1) | c^2-1 ==>  (a-1)(b-1) |  c+1
>> (a-1)(b-1) |ab+1==> (a-1)(b-1)!a+b
>> a=2 ==> 2b-2= 2+b. b=4 e c=ab=8 (2,4,8)
>> a=3 ==> 2(b-1) | 3+b ==> 2(b-1) = 3+. b=5 e c=ab=15. (3,5,15),
>> Forcei um pouco a barra para mostrar que c=ab.
>> Alguém teria uma outra solução, ou um endereço onde se tem as questões da
>> IMO e suas resoluções?
>>
>> Grato!
>> Saudações,
>> PJMS
>>
>>
>>
>> --
>> Esta mensagem foi verificada pelo sistema de antivírus e
>> acredita-se estar livre de perigo.
>
>
> --
> Esta mensagem foi verificada pelo sistema de antivírus e
> acredita-se estar livre de perigo.

-- 
Esta mensagem foi verificada pelo sistema de antiv�rus e
 acredita-se estar livre de perigo.



[obm-l] Problema da IMO

2020-09-11 Por tôpico Pedro José
Bom dia!

Recebi de um filho de um amigo, um problema que já o fizera.
(a-1)(b-1)(c-1) | abc-1;  11, e para um dado a k é máximo para b e c mínimos logo b=a+1 e
c=a+2
[a(a+1)(a+2)]/[(a-1)(a)(a+1)] > [a(a+1)(a+2)-1]/[(a-1)(a)(a+1)]>=2, então
(a+2)/(a-1)>2 ==> a <4, a=2 ou a=3.
O k é máximo para a=2, b=3 e c=4 ==> k <4, logo k=2 ou k=3.
S.p.g, se a é ímpar (a-1)(b-1)(c-1) é par; então b,c ímpares e k é livre.
S.p.g se a é par abc-1 é ímpar; então b,c são pares e k ímpar.
a=2, temos  2b(b+1)/[(b-1)b] >3, não usei a restrição de paridade para c
para facilitar a simplificação. b<5 Logo a=2 2; b<7 e 3 k=2 e c= 15. (3,5,15) é a outra solução.

Só agora me apercebi de que c=ab nas duas soluções. Então tentei uma nova
solução.
(a-1)(b-1)(c-1) | abc-1 e (a-1)(b-1)(c-1) | abc + c(1 - (a+b)) -ab+ (a+b) -
1 logo divide a diferença:
(a-1)(b-1)(c-1) | (a+b) (c-1) + ab -1 - (c-1) logo c-1 | ab-1, então ab-1=
w(c-1), para algum w inteiro e ab=w(c-1) +1 (i)
Como a=2 ou a=3
Se a=2. e w>=2
Temos por (i) 2b>= 2 (c-1) +1 c-1>=b, logo absurdo.
Se a=3
Temos por (i) 3b>= w(c-1)+1; w=3 ==>3b< 3 (c-1) +1 pois c>b
w=2 ==> 3b =2(c-1) +1 ==> c=(3b+1)/2
2(b-1)(3b-1)/2 | 3b(3b+1)/2 -1 ==>  2(b-1)(3b-1)/ | 3b(3b+1) -2 ==>
6b^2-8b-2 | 9b^2+3b-1 ==>  6b^2-8b-2 | 3b^2 +11b+ 4
==> b <=5. Como b>a=3 ==> b=5 e c= 8, ferindo a paridade.
Logo ab-1=c-1 ==> ab=c ==> (a-1)(b-1)(c-1) | c^2-1 ==>  (a-1)(b-1) |  c+1
(a-1)(b-1) |ab+1==> (a-1)(b-1)!a+b
a=2 ==> 2b-2= 2+b. b=4 e c=ab=8 (2,4,8)
a=3 ==> 2(b-1) | 3+b ==> 2(b-1) = 3+. b=5 e c=ab=15. (3,5,15),
Forcei um pouco a barra para mostrar que c=ab.
Alguém teria uma outra solução, ou um endereço onde se tem as questões da
IMO e suas resoluções?

Grato!
Saudações,
PJMS

-- 
Esta mensagem foi verificada pelo sistema de antiv�rus e
 acredita-se estar livre de perigo.



[obm-l] Re: [obm-l] Re: [obm-l] Re: [obm-l] Re: [obm-l] Re: [obm-l] Geometria plana com desigualdade de médias?

2020-08-26 Por tôpico Pedro José
Boa noite!
Anderson,
achei legal a sua visão. Mas não consegui evoluir com nada.
Todavia, fiquei com uma dúvida. Como x+y é um dos ângulos do triângulo
temos a restrição 0 escreveu:

> Em qui., 20 de ago. de 2020 às 22:03, Anderson Torres
>  escreveu:
> >
> > Em ter., 18 de ago. de 2020 às 19:51, Pedro José 
> escreveu:
> > >
> > > Boa noite!
> > > Cláudio,
> > > não consegui nada geométrico.
> > > O máximo que atingi foi:
> > > a/ha + b/hb + c/hc= [cotg(A1) +cotg (A2)]  + [cotg(B1) +cotg (B2)] +
> co[tg(C1) +cotg (C2)] com A1 + A2 = A; B1 + B2 + B e C1 + C2 = C.
> > > Para ser mínimo cada termo entre colchetes deve ser mínimo, o que
> ocorre quando A1 = A2; B1 = B2 e C1 = C2. Logo P seria o encontro das
> bissetrizes e logo I.
> > > Onde: A1= PAB e A2=PAC; B1=PBA e B2=PBC; C1=PCA e C2=PCB.
> >
> > Acho que daqui poderia sair uma interpretação mais escamoteada.
> > Afinal, trigonometria é uma espécie de "ponto de contato" entre a
> > geometria analítica e a sintética, entre a nuvem de desenhos e a de
> > números.
> >
> > Acredito que a solução aqui seria arranjar uma interpretação
> > geométrica desses colchetes de co-tangentes. Acredito que possamos
> > apelar para Ptolomeu em algum momento ou para um macete de
> > semelhanças, pois as projeções de um ponto sobre duas retas criam um
> > quadrilátero cíclico.
>
> Acrescentando mais coisas: se queremos minimizar cot(x) +cot(y) com
> x+y fixo, isto é equivalente a minimizar tan(90-x)+tan(90-y) com
> 90-x+90-y fixo. Ou como maximizar tan(x) + tan(y) com x+y fixo.
>
> Geometricamente, tangente é cateto oposto dividido por cateto
> adjacente. Logo uma soma de tangentes com catetos adjacentes iguais
> equivale a uma soma de catetos opostos! Assim sendo, nosso problema
> pode ser pensado da seguinte forma:
>
> Dados um ponto A e uma reta d fixos, temos que construir duas retas x
> e y, com ângulo 'alfa' entre elas, ambas passando por A e tais que a
> distância entre os pontos X e Y, que elas geram ao intersectar d, seja
> mínima.
>
> Daí fica fácil argumentar que a altura por A também tem que ser a
> bissetriz por A.
>
> No fundo do fundo é uma forma de geometrizar a solução trigonométrica.
> A trigonometria se torna apenas um atalho.
>
> Vou formalizar isso mais tarde, com desenhos e tudo.
>
>
>
> >
> > Isso até me lembra o famoso artigo do Shine sobre geometria cearense
> > VS geometria paulista:
> > https://cyshine.webs.com/geometria-2005.pdf
> >
> >
> > >
> > > Saudações,
> > > PJMS
> > >
> > > Em ter., 18 de ago. de 2020 às 11:34, Claudio Buffara <
> claudio.buff...@gmail.com> escreveu:
> > >>
> > >> Será que tem uma demonstração mais geométrica e menos algébrica
> disso? E que torne o resultado mais intuitivo?
> > >> É razoável que o ponto P não esteja muito próximo de qualquer dos
> lados, pois neste caso, se P se aproximasse do lado a, por exemplo, a/h_a
> cresceria e a expressão se afastaria do valor mínimo.
> > >> Mas, com lados não necessariamente congruentes, não é óbvio, a
> priori, que P deva ser equidistante dos três.
> > >> De fato, seria razoável esperar que P estivesse mais próximo do maior
> lado e conjecturar, por exemplo, que o P que minimiza a expressão é tal que
> a/h_a = b/h_b = c/h_c.
> > >> O fato de P ser o incentro não me parece a conjectura mais evidente
> neste caso.
> > >>
> > >>
> > >> On Sun, Aug 16, 2020 at 10:11 AM Matheus Secco <
> matheusse...@gmail.com> wrote:
> > >>>
> > >>> Olá, Vanderlei.
> > >>> Por Cauchy-Schwarz, temos
> > >>>
> > >>> (a/ha + b/hb + c/hc) * (a*ha + b*hb + c*hc) >= (a+b+c)^2.  (#)
> > >>>
> > >>> Como (a*ha + b*hb + c*hc) = 2S, onde S é a área de ABC, segue que a
> expressão a/ha + b/hb + c/hc é pelo menos 2p^2/S, onde p é o semi-perimetro.
> > >>>
> > >>> Por outro lado, a igualdade em (#) ocorre se, e somente se, ha = hb
> = hc, ou seja, quando P é o incentro do triângulo
> > >>>
> > >>> Abraços,
> > >>> Matheus
> > >>>
> > >>> Em dom, 16 de ago de 2020 08:59, Professor Vanderlei Nemitz <
> vanderma...@gmail.com> escreveu:
> > >>>>
> > >>>> Bom dia!
> > >>>>
> > >>>> Tentei utilizar alguma desigualdade de médias aqui, mas não tive
> êxito. Alguém ajuda?
> > >>>> Muito agradecido!
> > >>>>
>

[obm-l] Re: [obm-l] Re: [obm-l] Re: [obm-l] Geometria plana com desigualdade de médias?

2020-08-18 Por tôpico Pedro José
Boa noite!
Cláudio,
não consegui nada geométrico.
O máximo que atingi foi:
a/ha + b/hb + c/hc= [cotg(A1) +cotg (A2)]  + [cotg(B1) +cotg (B2)] +
co[tg(C1) +cotg (C2)] com A1 + A2 = A; B1 + B2 + B e C1 + C2 = C.
Para ser mínimo cada termo entre colchetes deve ser mínimo, o que ocorre
quando A1 = A2; B1 = B2 e C1 = C2. Logo P seria o encontro das bissetrizes
e logo I.
Onde: A1= PAB e A2=PAC; B1=PBA e B2=PBC; C1=PCA e C2=PCB.

Saudações,
PJMS

Em ter., 18 de ago. de 2020 às 11:34, Claudio Buffara <
claudio.buff...@gmail.com> escreveu:

> Será que tem uma demonstração mais geométrica e menos algébrica disso? E
> que torne o resultado mais intuitivo?
> É razoável que o ponto P não esteja muito próximo de qualquer dos lados,
> pois neste caso, se P se aproximasse do lado a, por exemplo,
> a/h_a cresceria e a expressão se afastaria do valor mínimo.
> Mas, com lados não necessariamente congruentes, não é óbvio, a priori, que
> P deva ser equidistante dos três.
> De fato, seria razoável esperar que P estivesse mais próximo do maior lado
> e conjecturar, por exemplo, que o P que minimiza a expressão é tal que
> a/h_a = b/h_b = c/h_c.
> O fato de P ser o incentro não me parece a conjectura mais evidente neste
> caso.
>
>
> On Sun, Aug 16, 2020 at 10:11 AM Matheus Secco 
> wrote:
>
>> Olá, Vanderlei.
>> Por Cauchy-Schwarz, temos
>>
>> (a/ha + b/hb + c/hc) * (a*ha + b*hb + c*hc) >= (a+b+c)^2.  (#)
>>
>> Como (a*ha + b*hb + c*hc) = 2S, onde S é a área de ABC, segue que a
>> expressão a/ha + b/hb + c/hc é pelo menos 2p^2/S, onde p é o
>> semi-perimetro.
>>
>> Por outro lado, a igualdade em (#) ocorre se, e somente se, ha = hb = hc,
>> ou seja, quando P é o incentro do triângulo
>>
>> Abraços,
>> Matheus
>>
>> Em dom, 16 de ago de 2020 08:59, Professor Vanderlei Nemitz <
>> vanderma...@gmail.com> escreveu:
>>
>>> Bom dia!
>>>
>>> Tentei utilizar alguma desigualdade de médias aqui, mas não tive êxito.
>>> Alguém ajuda?
>>> Muito agradecido!
>>>
>>> Seja P um ponto no interior de um triângulo e sejam ha, hb e hc as
>>> distâncias de P aos lados a, b e c, respectivamente. Mostre que o valor
>>> mínimo de (a/ha) + (b/hb) + (c/hc) ocorre quando P é o incentivo do
>>> triângulo ABC.
>>>
>>> --
>>> Esta mensagem foi verificada pelo sistema de antivírus e
>>> acredita-se estar livre de perigo.
>>
>>
>> --
>> Esta mensagem foi verificada pelo sistema de antivírus e
>> acredita-se estar livre de perigo.
>
>
> --
> Esta mensagem foi verificada pelo sistema de antivírus e
> acredita-se estar livre de perigo.

-- 
Esta mensagem foi verificada pelo sistema de antiv�rus e
 acredita-se estar livre de perigo.



[obm-l] Problema da IMO

2020-07-23 Por tôpico Pedro José
 Encontre todos os (k,n), k,n pertencentes à Z+, tal que k!=
(2^n-1)*(2^n-2)*(2^n-4)*...(2^n-2^(n-2))*(2^n-2(n-1))

Gostaria de saber se está correto?

Como os dois termos iniciais são consecutivos, é intuitivo que haja
baixíssima probabilidade de termos respostas que não sejam as triviais, com
um ou dois fatores.
(1,1) ==> 1! = 2-1 correto! e (3,2) --> 3!= 3*2, correto!
Para n= 3 temos k!= 7*6*4 com um fator 7 e não temos um 5, não atende.
Para n=4 temos k!=15*14*12*8=20160, mas 7! <2 0.060 < 8!;não atende.

G(n) = (2^n-1)*(2^n-2)*(2^n-4)*...(2^n-2^(n-2))*(2^n-2(n-1)) ==> G(n+1)=
(2^(n+1) -1)**G(n)2^n

Vamos mostrar por absurdo que k! <>
(2^n-1)*(2^n-2)*(2^n-4)*...(2^n-2^(n-2))*(2^n-2(n-1))para qualquer n>=5.
Seja , por hipótese, k! =
(2^n-1)*(2^n-2)*(2^n-4)*...(2^n-2^(n-2))*(2^n-2(n-1))e n>=5.
Fica claro que 2^i || (2^n-2î) com n>i e n, i inteiros.
Então obtém-se uma P.A. para o expoente da fatoração de cada termo do
produto, nos levando a: k!=2^((n-1)*n/2)*m, com m ímpar.Ou seja, a
fatoração de k! tem como o expoente de 2, a= (n-1)*n/2
Todavia, por contagem é muito simples chegar-se a: a=
[k/2]+[k/4]+[k/8]+[k/16]+...; Note que embora haja uma infinidade de
parcela, haverá um j, tal que 2^j>k e a partir desse termo todas as
parcelas irão zerar. Onde [x] representa, parte inteira de x.
a= [k/2]+[k/4]+[k/8]+[k/16]+...= [k/2]+[k/4]+[k/8]+[k/16]+ ...[k/2^(j-1] <
[k/2]+[k/4] + ...+[k/2^(j-1)] + k/2^j + k/2^(j+1)+ k/2^(j+2+< k/2 + k/4
+ k/8 + k/16+...= k ==>
==> k>a ==> k>= (a+1), já que k e a são inteiros. então k! >=(a+1)!
Vamos por indução.
Para n=5 a=10 e k!>=11! > 31*30*28*24*16=k!, por hipótese; k! >k!, absurdo.

Se k!>=(a+1)! > (2^n-1)*(2^n-2)*(2^n-4)*...(2^n-2^(n-2))*(2^n-2(n-1)) para
n>=5
temos F=((n-1)*n/2+1)*(n-1)*n/2*((n-1)*n/2-1)*...2*1>
(2^n-1)*(2^n-2)*(2^n-4)*...(2^n-2^(n-2))*(2^n-2(n-1)=P

Para n+1
k! >= (a+1)! >
(n*(n+1)/2+1)*n*(n+1)/2*(n*(n+1)/2-1)**((n-1)*n/2+2)*F>(2^(n+1)-1)*2*(2^n-1)*2(2^n-2)*...*2*(2^n-2(n-1)=
(2^(n+1)-1)*2^n*P
Mas como F> P se mostramos que
(n*(n+1)/2+1)*n*(n+1)/2*(n*(n+1)/2-1)**((n-1)*n/2+2)>2^n*(2^(n+1)*2^n,
está provado.
O lado esquerdo é composto de n fatores todos maiores que o último que é
maior ou igual 12, para n=5.
O lado esquerdo é menor que 2^(2n+1)
Como 12^n= 3^n*2^2^n > 2* 2^2n > 2^n*(2^(n+1)*2^n,==>
(n*(n+1)/2+1)*n*(n+1)/2*(n*(n+1)/2-1)**((n-1)*n/2+2)>2^n*(2^(n+1)*2^.
então k!>k!, absurdo. Não existem soluções para k>=5. E como só existiam
duas para k<5, temos S= {(1,1);(3,2)}

Saudações,
PJMS

-- 
Esta mensagem foi verificada pelo sistema de antiv�rus e
 acredita-se estar livre de perigo.



Re: [obm-l] RE: Dois problemas

2020-04-27 Por tôpico Pedro José
Mandei espuriamente. Corrigi o que faltava.

Seja b=7
n é formado por 17C7 e R(n) por 7C71, logo 4*c+3 >=30 e 4c+3=7 mod 10,
portanto não há c que atenda.
Seja b=9
n é formado por 19C7 e R(n) por 7C91, logo 4c+3=9 mod 10 e
4*9+int((4c+3)/10)=c mod10
Só atende a primeira 4 ou 9, desses só atende a segunda 9.

Logo n= 1997 e 4n +3= 7991, resposta única. Havia mandado adiantado.


Em seg, 27 de abr de 2020 21:19, Pedro José  escreveu:

> Boa noite!
> 1) a_n = [n(n+1)]/2 mod 10
> Fácilmente se vê que 20 é múltiplo do período minímo pois:
> a_19= 0 e a_20=0 logo a_21= a_20+21mod10 =0+1=1=a1.
> a_22=a_1+22 mod10 = 3=a_3 É assim sucessivamente.
> Então o período é um divisor de 20
> p<>1, pois, a_1<>a_2
> p<>2, pois, a_1<>a_3
> p<>4, pois a_1<>a_5
> p<>5, pois a_1<> a_6
> p<>10 pois a_1<>a_11
> logo p = 20.
> Não vem outra forma se calcular a_1 até a_20
>
> a_1=1 e S_1=1
> a_2=3  e S_2=4
> a_3=6 e S_3=10
> a_4=0 e S_4=10
> a_5=5 e S_5=15
> a_6=1 e S_6= 16
> a_7=8 e S_7=24
> a_8=6 e S_8=30
> a_9=5 e S_9=35
> a_10=5 e S_10=40
> a_11=6 e S_11=46
> a_12=8 e S_12=54
> a_13=1 e S_13=55
> a_14=5 e S_14=60
> a_15=0 e S_15=60
> a_16=6 e S_16= 66
> a_17=3 e S_17=69
> a_18=1 e S_18=70
> a_19=0 e S_19=70
> a_20=0 e S_20=70
> É fácil ver que a cada 20 números a mais a soma dará 70 de novo.,  1992 =
> 20 x 99 +12; logo S_1992 =99 S_×0+S_12=6984.
>
> 2) seja n formado por ABCD, vamos representar a ideia número pela letra
> minúscula a, b, c, d e U={a, b, c, d} max(U)=9 e min(U)=0.
> a<=2 e a ímpar pois R(n)=DCBA é ímpar.
> Então a=1
>  4n+3= 4x(a*10^3+b*10^2+c*10+d)+3=
> =4d+3 mod10 4d+3=a mod10 ==> d=2 ou d=7, mas d>=4 pois 4n+3= d*10^3 +
> c*10^2 + b*10 + a > 4 *10^3. Logo d=7
> Logo b=7 ou b=9
> pois 4*b +3>30 É b é ímpar pois
> DCBA é côngruo 3 mod4
> Seja b=7
> n é formado por 17C7 e R(n) por 7C71, logo 4*c+3 >=30 e 4c+3=7 mod 10,
> portanto não há c que atenda.
> Seja b=9
> n é formado por 19C7 e R(n) por 7C91, logo 4c+3=9 mod 10 e
> 4*9+int((4c+3)/10)=c mod10
> Só atende a primeira 4 ou 9, desses só atende a segunda 9.
>
Logo n= 1997 e 4n +3= 7991, resposta única. Havia mandado adiantado.


>
>
>
>
> Em dom, 26 de abr de 2020 22:56, Julio Mohnsam <
> prof.juliomat...@hotmail.com> escreveu:
>
>> se n=2019
>>
>> --
>> *De:* owner-ob...@mat.puc-rio.br  em nome de
>> Rogério Possi Júnior 
>> *Enviado:* domingo, 26 de abril de 2020 18:21
>> *Para:* Lista de Olímpiada OBM 
>> *Assunto:* [obm-l] Dois problemas
>>
>> Boa noite.
>>
>> Quem pode ajudar com esses dois problemas:
>>
>> 1) (Ibero-1992) Para cada inteiro positivo n, seja a_n o último dígito de
>> 1+2+3+...+n. Calcule a_1+a_2+...+a_n.
>>
>> 2) (UK-1997) N é um número inteiro de 4 dígitos não terminado em zero, e
>> R(N) é o número inteiro de 4 dígitos obtido pela reversão dos dígitos de N;
>> por exemplo R(3275)=5723. Determine todos os inteiros N ára os quais
>> R(N)=4N+3.
>>
>> Sds,
>>
>> Rogério
>>
>>
>> --
>> Esta mensagem foi verificada pelo sistema de antivírus e
>> acredita-se estar livre de perigo.
>>
>> --
>> Esta mensagem foi verificada pelo sistema de antivírus e
>> acredita-se estar livre de perigo.
>>
>

-- 
Esta mensagem foi verificada pelo sistema de antiv�rus e
 acredita-se estar livre de perigo.



Re: [obm-l] RE: Dois problemas

2020-04-27 Por tôpico Pedro José
Boa noite!
1) a_n = [n(n+1)]/2 mod 10
Fácilmente se vê que 20 é múltiplo do período minímo pois:
a_19= 0 e a_20=0 logo a_21= a_20+21mod10 =0+1=1=a1.
a_22=a_1+22 mod10 = 3=a_3 É assim sucessivamente.
Então o período é um divisor de 20
p<>1, pois, a_1<>a_2
p<>2, pois, a_1<>a_3
p<>4, pois a_1<>a_5
p<>5, pois a_1<> a_6
p<>10 pois a_1<>a_11
logo p = 20.
Não vem outra forma se calcular a_1 até a_20

a_1=1 e S_1=1
a_2=3  e S_2=4
a_3=6 e S_3=10
a_4=0 e S_4=10
a_5=5 e S_5=15
a_6=1 e S_6= 16
a_7=8 e S_7=24
a_8=6 e S_8=30
a_9=5 e S_9=35
a_10=5 e S_10=40
a_11=6 e S_11=46
a_12=8 e S_12=54
a_13=1 e S_13=55
a_14=5 e S_14=60
a_15=0 e S_15=60
a_16=6 e S_16= 66
a_17=3 e S_17=69
a_18=1 e S_18=70
a_19=0 e S_19=70
a_20=0 e S_20=70
É fácil ver que a cada 20 números a mais a soma dará 70 de novo.,  1992 =
20 x 99 +12; logo S_1992 =99 S_×0+S_12=6984.

2) seja n formado por ABCD, vamos representar a ideia número pela letra
minúscula a, b, c, d e U={a, b, c, d} max(U)=9 e min(U)=0.
a<=2 e a ímpar pois R(n)=DCBA é ímpar.
Então a=1
 4n+3= 4x(a*10^3+b*10^2+c*10+d)+3=
=4d+3 mod10 4d+3=a mod10 ==> d=2 ou d=7, mas d>=4 pois 4n+3= d*10^3 +
c*10^2 + b*10 + a > 4 *10^3. Logo d=7
Logo b=7 ou b=9
pois 4*b +3>30 É b é ímpar pois
DCBA é côngruo 3 mod4
Seja b=7
n é formado por 17C7 e R(n) por 7C71, logo 4*c+3 >=30 e 4c+3=7 mod 10,
portanto não há c que atenda.
Seja b=9
n é formado por 19C7 e R(n) por 7C91, logo 4c+3=9 mod 10 e






Em dom, 26 de abr de 2020 22:56, Julio Mohnsam 
escreveu:

> se n=2019
>
> --
> *De:* owner-ob...@mat.puc-rio.br  em nome de
> Rogério Possi Júnior 
> *Enviado:* domingo, 26 de abril de 2020 18:21
> *Para:* Lista de Olímpiada OBM 
> *Assunto:* [obm-l] Dois problemas
>
> Boa noite.
>
> Quem pode ajudar com esses dois problemas:
>
> 1) (Ibero-1992) Para cada inteiro positivo n, seja a_n o último dígito de
> 1+2+3+...+n. Calcule a_1+a_2+...+a_n.
>
> 2) (UK-1997) N é um número inteiro de 4 dígitos não terminado em zero, e
> R(N) é o número inteiro de 4 dígitos obtido pela reversão dos dígitos de N;
> por exemplo R(3275)=5723. Determine todos os inteiros N ára os quais
> R(N)=4N+3.
>
> Sds,
>
> Rogério
>
>
> --
> Esta mensagem foi verificada pelo sistema de antivírus e
> acredita-se estar livre de perigo.
>
> --
> Esta mensagem foi verificada pelo sistema de antivírus e
> acredita-se estar livre de perigo.
>

-- 
Esta mensagem foi verificada pelo sistema de antiv�rus e
 acredita-se estar livre de perigo.



Re: [obm-l] teoria dos numeros

2020-03-30 Por tôpico Pedro José
Boa noite!

errata:
Ao invés de: 128=2^7 então 2^7| 49^{n} + 81^{n} −2<==> x= 2^7| 49^{n} +
81^{n}=2 mod2^7
128=2^7 então 2^7| 49^{n} + 81^{n} −2<==> x= 49^{n} + 81^{n}=2 mod2^7

Saudações,
PJMS

Em dom., 29 de mar. de 2020 às 14:04, Pedro José 
escreveu:

> Bom dia!
> Prove que 128 divide 49^{n} + 81^{n} −2, para todo n ≥ 1.
> 128=2^7 então 2^7| 49^{n} + 81^{n} −2<==> x= 2^7| 49^{n} + 81^{n}=2 mod2^7
> x= a + b , a= 49^n e b=81^n
> a= (64-15)^n = n(-1)^n*n*64*(15)^(n-1) + (-1)^n*15^n mod2^7; pois, os
> demais termos do binômio de Newton terão o fator (2^6)^m com m>1 que é
> côngruo 0 mod2^7.
> b= (64+17)^n = n*64*17^(n-1) + 17^n mod2^7 pelo mesmo motivo anterior.
> a+b = n*64(17^n-1 +(-1)^(n-1)*15^(n-1)) + 17^n + (-1)^n*15^n =
> (-1)^(n-1)*15^(n-1)) + 17^n + (-1)^n*15^n mod2^; pois a primeira parcela é
> côngrua a 0 mod2^7; já que o termo entre parêntesis é par.
> (16+1)^n= n*16+1 mod2^7 ,pois, (2^4)^m =0 mod2^7 para m>1
> (-1)^n*(16-1)= (-1)^n*[(-1)^(n-1)*n*16+(-1)^n]=-16n +1
> então x = a+b= 2 mod2^7 ==> 2^7 | a+b-2
>
> Saudações,
> PJMS
>
>
>
>
> Em sáb., 28 de mar. de 2020 às 14:05, Israel Meireles Chrisostomo <
> israelmchrisost...@gmail.com> escreveu:
>
>> Eu sei resolver o problema abaixo,porém não sei se é a forma mais simples
>> de se fazer.Vcs poderiam por favor colocar suas soluções nos comentários
>> dessa publicação? O problema é o seguinte:
>> Prove que 128 divide 49^{n} + 81^{n} −2, para todo n ≥ 1.Se possível não
>> use indução, pois eu já estou usando indução.
>>
>> --
>> Israel Meireles Chrisostomo
>>
>> --
>> Esta mensagem foi verificada pelo sistema de antivírus e
>> acredita-se estar livre de perigo.
>
>

-- 
Esta mensagem foi verificada pelo sistema de antiv�rus e
 acredita-se estar livre de perigo.



Re: [obm-l] teoria dos numeros

2020-03-29 Por tôpico Pedro José
Bom dia!
Prove que 128 divide 49^{n} + 81^{n} −2, para todo n ≥ 1.
128=2^7 então 2^7| 49^{n} + 81^{n} −2<==> x= 2^7| 49^{n} + 81^{n}=2 mod2^7
x= a + b , a= 49^n e b=81^n
a= (64-15)^n = n(-1)^n*n*64*(15)^(n-1) + (-1)^n*15^n mod2^7; pois, os
demais termos do binômio de Newton terão o fator (2^6)^m com m>1 que é
côngruo 0 mod2^7.
b= (64+17)^n = n*64*17^(n-1) + 17^n mod2^7 pelo mesmo motivo anterior.
a+b = n*64(17^n-1 +(-1)^(n-1)*15^(n-1)) + 17^n + (-1)^n*15^n =
(-1)^(n-1)*15^(n-1)) + 17^n + (-1)^n*15^n mod2^; pois a primeira parcela é
côngrua a 0 mod2^7; já que o termo entre parêntesis é par.
(16+1)^n= n*16+1 mod2^7 ,pois, (2^4)^m =0 mod2^7 para m>1
(-1)^n*(16-1)= (-1)^n*[(-1)^(n-1)*n*16+(-1)^n]=-16n +1
então x = a+b= 2 mod2^7 ==> 2^7 | a+b-2

Saudações,
PJMS




Em sáb., 28 de mar. de 2020 às 14:05, Israel Meireles Chrisostomo <
israelmchrisost...@gmail.com> escreveu:

> Eu sei resolver o problema abaixo,porém não sei se é a forma mais simples
> de se fazer.Vcs poderiam por favor colocar suas soluções nos comentários
> dessa publicação? O problema é o seguinte:
> Prove que 128 divide 49^{n} + 81^{n} −2, para todo n ≥ 1.Se possível não
> use indução, pois eu já estou usando indução.
>
> --
> Israel Meireles Chrisostomo
>
> --
> Esta mensagem foi verificada pelo sistema de antivírus e
> acredita-se estar livre de perigo.

-- 
Esta mensagem foi verificada pelo sistema de antiv�rus e
 acredita-se estar livre de perigo.



[obm-l] Re: período de dízima

2020-03-24 Por tôpico Pedro José
Bom dia!
Consegui demonstrar que é verdadeira.
Só faltou 2^a||t e 2^b||t ou seja (10,n)=1.

Saudações,
PJMS

Em ter., 10 de mar. de 2020 às 18:39, Pedro José 
escreveu:

> Boa noite!
>
> Alguém poderia provar ou derrubar a conjectura a seguir?
>
> Seja s/t uma fração em que t não divide s e (s,t)=1; seja t=2^a.2^b.n
> O número de algarismos da parte não periódica é o max(a,b) e o número de
> algarismos da parte periódica é a ord 10 mod n.
> Representação decimal.
>
> Saudações,
> PJMS
>

-- 
Esta mensagem foi verificada pelo sistema de antiv�rus e
 acredita-se estar livre de perigo.



[obm-l] Re: [obm-l] Re: [obm-l] Re: [obm-l] Teoria dos números

2020-03-22 Por tôpico Pedro José
Boa tarde!
Perfeita a sua correção.
Quanto ao questionamento, nem tenho formação em matemática, meu sonho é
cursar no IMPA ao me aposentar. Sou pitaqueiro. Ouço um assunto que não
conheço, tento aprendê-lo. Na verdade, gosto de matemática. Talvez seja ela
o "Mundo das ideias", o mundo ideal, a qual Platão se referiu.
Saudações,
PJMS

Em dom, 22 de mar de 2020 12:25, Israel Meireles Chrisostomo <
israelmchrisost...@gmail.com> escreveu:

> Acho q tem uma ´pequena correção no seguinte passo "4k+1. Pegando os
> fatores (4n-1)^2 e (4n+1)^2, teremos que 2^6 |p(n) para qualquer n=4k+1."O
> correto seria "Para n=4k+1.Pegando os fatores (n-1)^2 e (n+1)^2"
>
> Em dom., 22 de mar. de 2020 às 10:14, Israel Meireles Chrisostomo <
> israelmchrisost...@gmail.com> escreveu:
>
>> Primeiramente obrigado pela solução.Mas Pedro, tenho uma pergunta :   o
>> sr. é professor de Matemática?
>>
>> Em dom., 22 de mar. de 2020 às 01:34, Pedro José 
>> escreveu:
>>
>>> Bom dia!
>>> Dei uma mancada.
>>> O expoente de 3 é 3 e não 2.
>>> Retornando às classes mod 3.
>>> Ao último fator é côngruo à (n-1)*n
>>> Para n=3k aparece outro fator e 3^3|p(n), n=3k.
>>> n=3k+1, tenho (n-1)^2 e (n-1), 3^3|p(n), n=3k+1
>>> n=3k+2, tenho(n-2)^2 é (n+1)^2, 3^3|p(n), n=3k+2,
>>> Logo 3^3|p(n) para todo n inteiro.
>>> D>=2^6*3^3*5. Mas D<=2^6*3^3*5, então D=8640
>>> Desculpem-me pelo erro.
>>> Saudações,
>>> PJMS.
>>>
>>>
>>>
>>> Em sáb, 21 de mar de 2020 13:20, Pedro José 
>>> escreveu:
>>>
>>>> Boa tarde!
>>>> Nem carece método numérico.
>>>> Para n=1 ou n=0 ou n=2 temos que qualquer inteiro divide o polinômio
>>>> p(n)=(n-2)^2*(n-1)^2*n^2*(n+1)^2*(4n^2-4n-9)
>>>>
>>>> p(3)=8640
>>>> p(4)=561600 então (p(3),p(4))=8640=2^6*3^3*5.
>>>> Seja D o maior inteiro que divide p(n) para todo n inteiro, D<=8640
>>>> Vamos pegar as classes de equivalência mod 4. Seja k um inteiro.
>>>> Para 4k temos que n^2= 16k^2 e (n-2) é par logo (n-2)^2= 4s^2 com s
>>>> inteiro. Logo 2^6 divide p(n) para qualquer n =4k.
>>>> 4k+1. Pegando os fatores (4n-1)^2 e (4n+1)^2, teremos que 2^6 |p(n)
>>>> para qualquer n=4k+1.
>>>> 4k+2. Pegando (n-2)^2 e n^2, teremos que 2^6|p(n) para qualquer n=4k+2
>>>> 4k+3, pegando os mesmos fatores de 4k+1, 2^6|p(n) para n=4k+3.
>>>> Portanto 2^6|p(n) para qualquer inteiro
>>>> Agora classes de equivalência mod 3
>>>> 3k, pegando n^2, 3^2|p(n) para n=3k
>>>> 3k+1, pegando (n-1)^2; 3^2| p(n), n=3k+1
>>>> 3k+2, pegando (n-2)^2, 3^2| p(n), n=3k+2
>>>> Daí 3^2|p(n) para qualquer n inteiro.
>>>> Classes de equivalência mod 5.
>>>> 5k, n^2 , 5 |p(n), n =5
>>>> 5k +1, (n-1)^2, 5|p(n), n=5k+1
>>>> 5k+2, (n-2)^2, 5|p(n), n=5k+2
>>>> 5k+3, (4n^2-4n-9)=(100k^2-100k+15)
>>>> 5|p(n), n=5k+3
>>>> 5k+4, (n+1)^2, 5|p(n) , n=5k+4.
>>>> Então 5|p(n) para todo inteiro
>>>> D>=2^6*3^2×*5
>>>> Mas D<=2^6*3^2*5, logo D=8640
>>>>
>>>> Saudações,
>>>> PJMS
>>>>
>>>> Em sáb, 21 de mar de 2020 04:39, Pedro José 
>>>> escreveu:
>>>>
>>>>> Bom dia!
>>>>> Falta de novo, em seu questionamento, informar que n é inteiro ou
>>>>> natural e colocar a condição para qualquer valor de n. Chamando o 
>>>>> polinômio
>>>>> de p(n)
>>>>> Para n=0, 1 ou 2, qualquer inteiro divide.
>>>>> Faria mdc(p(3),p(4))= A1
>>>>> Se der "pequeno", com poucos fatores primos e expoentes pequenos. Paro
>>>>> em A1, se não.
>>>>> (p(5),A1)=A2 uso o mesmo critério de parar
>>>>> (p(6),A2)=A3 até parar em:
>>>>> Ai=(p(i+3),A(i-1)).
>>>>> Aí faço o polinômio módfi^xi, onde fi é um fator primo de Aí e xi seu
>>>>> expoente. verifico se para cada resíduon= 1, 2...fi^n-1 se P(n)=0 mod 
>>>>> fi^si
>>>>> Se falhar diminuto xi em 1 e repito o teste para todos resíduos de
>>>>> fi^(xi-1)-1 até um dado xki em que todos os p(resíduos) foram equivalente 
>>>>> a
>>>>> zero módulo fi^xki ou quando fizer para o expoente 1  e não zerar para
>>>>> todos resíduos de fi, quando o fator será descartado.
>>>>> Depois repito para cada fator primo f e seu respectivo expoe

[obm-l] Re: [obm-l] Teoria dos números

2020-03-21 Por tôpico Pedro José
Bom dia!
Dei uma mancada.
O expoente de 3 é 3 e não 2.
Retornando às classes mod 3.
Ao último fator é côngruo à (n-1)*n
Para n=3k aparece outro fator e 3^3|p(n), n=3k.
n=3k+1, tenho (n-1)^2 e (n-1), 3^3|p(n), n=3k+1
n=3k+2, tenho(n-2)^2 é (n+1)^2, 3^3|p(n), n=3k+2,
Logo 3^3|p(n) para todo n inteiro.
D>=2^6*3^3*5. Mas D<=2^6*3^3*5, então D=8640
Desculpem-me pelo erro.
Saudações,
PJMS.



Em sáb, 21 de mar de 2020 13:20, Pedro José  escreveu:

> Boa tarde!
> Nem carece método numérico.
> Para n=1 ou n=0 ou n=2 temos que qualquer inteiro divide o polinômio
> p(n)=(n-2)^2*(n-1)^2*n^2*(n+1)^2*(4n^2-4n-9)
>
> p(3)=8640
> p(4)=561600 então (p(3),p(4))=8640=2^6*3^3*5.
> Seja D o maior inteiro que divide p(n) para todo n inteiro, D<=8640
> Vamos pegar as classes de equivalência mod 4. Seja k um inteiro.
> Para 4k temos que n^2= 16k^2 e (n-2) é par logo (n-2)^2= 4s^2 com s
> inteiro. Logo 2^6 divide p(n) para qualquer n =4k.
> 4k+1. Pegando os fatores (4n-1)^2 e (4n+1)^2, teremos que 2^6 |p(n) para
> qualquer n=4k+1.
> 4k+2. Pegando (n-2)^2 e n^2, teremos que 2^6|p(n) para qualquer n=4k+2
> 4k+3, pegando os mesmos fatores de 4k+1, 2^6|p(n) para n=4k+3.
> Portanto 2^6|p(n) para qualquer inteiro
> Agora classes de equivalência mod 3
> 3k, pegando n^2, 3^2|p(n) para n=3k
> 3k+1, pegando (n-1)^2; 3^2| p(n), n=3k+1
> 3k+2, pegando (n-2)^2, 3^2| p(n), n=3k+2
> Daí 3^2|p(n) para qualquer n inteiro.
> Classes de equivalência mod 5.
> 5k, n^2 , 5 |p(n), n =5
> 5k +1, (n-1)^2, 5|p(n), n=5k+1
> 5k+2, (n-2)^2, 5|p(n), n=5k+2
> 5k+3, (4n^2-4n-9)=(100k^2-100k+15)
> 5|p(n), n=5k+3
> 5k+4, (n+1)^2, 5|p(n) , n=5k+4.
> Então 5|p(n) para todo inteiro
> D>=2^6*3^2×*5
> Mas D<=2^6*3^2*5, logo D=8640
>
> Saudações,
> PJMS
>
> Em sáb, 21 de mar de 2020 04:39, Pedro José 
> escreveu:
>
>> Bom dia!
>> Falta de novo, em seu questionamento, informar que n é inteiro ou natural
>> e colocar a condição para qualquer valor de n. Chamando o polinômio de p(n)
>> Para n=0, 1 ou 2, qualquer inteiro divide.
>> Faria mdc(p(3),p(4))= A1
>> Se der "pequeno", com poucos fatores primos e expoentes pequenos. Paro em
>> A1, se não.
>> (p(5),A1)=A2 uso o mesmo critério de parar
>> (p(6),A2)=A3 até parar em:
>> Ai=(p(i+3),A(i-1)).
>> Aí faço o polinômio módfi^xi, onde fi é um fator primo de Aí e xi seu
>> expoente. verifico se para cada resíduon= 1, 2...fi^n-1 se P(n)=0 mod fi^si
>> Se falhar diminuto xi em 1 e repito o teste para todos resíduos de
>> fi^(xi-1)-1 até um dado xki em que todos os p(resíduos) foram equivalente a
>> zero módulo fi^xki ou quando fizer para o expoente 1  e não zerar para
>> todos resíduos de fi, quando o fator será descartado.
>> Depois repito para cada fator primo f e seu respectivo expoente.
>> Ao final D = Produtório de cada fator fi elevado ao expoente xki que
>> zerou p(n) mod fi^xki para todos os resíduos, descartando os fí em que xji
>> chegou a 1 e não atendeu ou considerando nesse caso xki=0.
>>
>> Mas resolveria por método numérico.
>> Depois poste sua solução.
>>
>> Saudações,
>> PJMS.
>>
>>
>>
>>
>> Em sex, 20 de mar de 2020 12:42, Israel Meireles Chrisostomo <
>> israelmchrisost...@gmail.com> escreveu:
>>
>>> Qual o maior inteiro que divide (n - 2)^2 (n - 1)^2 n^2 (n + 1)^2 (4 n^2
>>> - 4 n - 9))?
>>> Eu sei resolver esse problema com meu algoritmo, porém gostaria de saber
>>> como os colegas o resolveriam.
>>> --
>>> Israel Meireles Chrisostomo
>>>
>>> --
>>> Esta mensagem foi verificada pelo sistema de antivírus e
>>> acredita-se estar livre de perigo.
>>
>>

-- 
Esta mensagem foi verificada pelo sistema de antiv�rus e
 acredita-se estar livre de perigo.



[obm-l] Re: [obm-l] Teoria dos números

2020-03-21 Por tôpico Pedro José
Boa tarde!
Nem carece método numérico.
Para n=1 ou n=0 ou n=2 temos que qualquer inteiro divide o polinômio
p(n)=(n-2)^2*(n-1)^2*n^2*(n+1)^2*(4n^2-4n-9)

p(3)=8640
p(4)=561600 então (p(3),p(4))=8640=2^6*3^3*5.
Seja D o maior inteiro que divide p(n) para todo n inteiro, D<=8640
Vamos pegar as classes de equivalência mod 4. Seja k um inteiro.
Para 4k temos que n^2= 16k^2 e (n-2) é par logo (n-2)^2= 4s^2 com s
inteiro. Logo 2^6 divide p(n) para qualquer n =4k.
4k+1. Pegando os fatores (4n-1)^2 e (4n+1)^2, teremos que 2^6 |p(n) para
qualquer n=4k+1.
4k+2. Pegando (n-2)^2 e n^2, teremos que 2^6|p(n) para qualquer n=4k+2
4k+3, pegando os mesmos fatores de 4k+1, 2^6|p(n) para n=4k+3.
Portanto 2^6|p(n) para qualquer inteiro
Agora classes de equivalência mod 3
3k, pegando n^2, 3^2|p(n) para n=3k
3k+1, pegando (n-1)^2; 3^2| p(n), n=3k+1
3k+2, pegando (n-2)^2, 3^2| p(n), n=3k+2
Daí 3^2|p(n) para qualquer n inteiro.
Classes de equivalência mod 5.
5k, n^2 , 5 |p(n), n =5
5k +1, (n-1)^2, 5|p(n), n=5k+1
5k+2, (n-2)^2, 5|p(n), n=5k+2
5k+3, (4n^2-4n-9)=(100k^2-100k+15)
5|p(n), n=5k+3
5k+4, (n+1)^2, 5|p(n) , n=5k+4.
Então 5|p(n) para todo inteiro
D>=2^6*3^2×*5
Mas D<=2^6*3^2*5, logo D=8640

Saudações,
PJMS

Em sáb, 21 de mar de 2020 04:39, Pedro José  escreveu:

> Bom dia!
> Falta de novo, em seu questionamento, informar que n é inteiro ou natural
> e colocar a condição para qualquer valor de n. Chamando o polinômio de p(n)
> Para n=0, 1 ou 2, qualquer inteiro divide.
> Faria mdc(p(3),p(4))= A1
> Se der "pequeno", com poucos fatores primos e expoentes pequenos. Paro em
> A1, se não.
> (p(5),A1)=A2 uso o mesmo critério de parar
> (p(6),A2)=A3 até parar em:
> Ai=(p(i+3),A(i-1)).
> Aí faço o polinômio módfi^xi, onde fi é um fator primo de Aí e xi seu
> expoente. verifico se para cada resíduon= 1, 2...fi^n-1 se P(n)=0 mod fi^si
> Se falhar diminuto xi em 1 e repito o teste para todos resíduos de
> fi^(xi-1)-1 até um dado xki em que todos os p(resíduos) foram equivalente a
> zero módulo fi^xki ou quando fizer para o expoente 1  e não zerar para
> todos resíduos de fi, quando o fator será descartado.
> Depois repito para cada fator primo f e seu respectivo expoente.
> Ao final D = Produtório de cada fator fi elevado ao expoente xki que zerou
> p(n) mod fi^xki para todos os resíduos, descartando os fí em que xji chegou
> a 1 e não atendeu ou considerando nesse caso xki=0.
>
> Mas resolveria por método numérico.
> Depois poste sua solução.
>
> Saudações,
> PJMS.
>
>
>
>
> Em sex, 20 de mar de 2020 12:42, Israel Meireles Chrisostomo <
> israelmchrisost...@gmail.com> escreveu:
>
>> Qual o maior inteiro que divide (n - 2)^2 (n - 1)^2 n^2 (n + 1)^2 (4 n^2
>> - 4 n - 9))?
>> Eu sei resolver esse problema com meu algoritmo, porém gostaria de saber
>> como os colegas o resolveriam.
>> --
>> Israel Meireles Chrisostomo
>>
>> --
>> Esta mensagem foi verificada pelo sistema de antivírus e
>> acredita-se estar livre de perigo.
>
>

-- 
Esta mensagem foi verificada pelo sistema de antiv�rus e
 acredita-se estar livre de perigo.



[obm-l] Re: [obm-l] Teoria dos números

2020-03-21 Por tôpico Pedro José
Bom dia!
Falta de novo, em seu questionamento, informar que n é inteiro ou natural e
colocar a condição para qualquer valor de n. Chamando o polinômio de p(n)
Para n=0, 1 ou 2, qualquer inteiro divide.
Faria mdc(p(3),p(4))= A1
Se der "pequeno", com poucos fatores primos e expoentes pequenos. Paro em
A1, se não.
(p(5),A1)=A2 uso o mesmo critério de parar
(p(6),A2)=A3 até parar em:
Ai=(p(i+3),A(i-1)).
Aí faço o polinômio módfi^xi, onde fi é um fator primo de Aí e xi seu
expoente. verifico se para cada resíduon= 1, 2...fi^n-1 se P(n)=0 mod fi^si
Se falhar diminuto xi em 1 e repito o teste para todos resíduos de
fi^(xi-1)-1 até um dado xki em que todos os p(resíduos) foram equivalente a
zero módulo fi^xki ou quando fizer para o expoente 1  e não zerar para
todos resíduos de fi, quando o fator será descartado.
Depois repito para cada fator primo f e seu respectivo expoente.
Ao final D = Produtório de cada fator fi elevado ao expoente xki que zerou
p(n) mod fi^xki para todos os resíduos, descartando os fí em que xji chegou
a 1 e não atendeu ou considerando nesse caso xki=0.

Mas resolveria por método numérico.
Depois poste sua solução.

Saudações,
PJMS.




Em sex, 20 de mar de 2020 12:42, Israel Meireles Chrisostomo <
israelmchrisost...@gmail.com> escreveu:

> Qual o maior inteiro que divide (n - 2)^2 (n - 1)^2 n^2 (n + 1)^2 (4 n^2 -
> 4 n - 9))?
> Eu sei resolver esse problema com meu algoritmo, porém gostaria de saber
> como os colegas o resolveriam.
> --
> Israel Meireles Chrisostomo
>
> --
> Esta mensagem foi verificada pelo sistema de antivírus e
> acredita-se estar livre de perigo.

-- 
Esta mensagem foi verificada pelo sistema de antiv�rus e
 acredita-se estar livre de perigo.



Re: [obm-l] Inteiros (divisibilidade)

2020-03-18 Por tôpico Pedro José
Bom dia!
Caso contrário fica simples.
b=-1 ==> a= -1 (-1,-1)
b=0 ou b=-2  ==> qualquer a
a=-1 ==> b qualquer
Para outros casos: a+1 é múltiplo de b+1
Generalizando:  |a+1|= |k(b+1)| com k inteiro

Em qua., 18 de mar. de 2020 às 09:04, Pedro José 
escreveu:

> Bom dia!
> Não há outra restrição?
> É igual perguntar quais os pares de inteiros (x,y) tais que x|y, com x=b+1
> e y=a+1.
>
> Saudações,
> PJMS
>
> Em qua., 18 de mar. de 2020 às 08:51, marcone augusto araújo borges <
> marconeborge...@hotmail.com> escreveu:
>
>> Determine todos os pares de inteiros a e b tais que a divide b+1 e b
>> divide a+1
>> Desde já agradeço
>> --
>> Esta mensagem foi verificada pelo sistema de antivírus e
>> acredita-se estar livre de perigo.
>>
>

-- 
Esta mensagem foi verificada pelo sistema de antiv�rus e
 acredita-se estar livre de perigo.



Re: [obm-l] Inteiros (divisibilidade)

2020-03-18 Por tôpico Pedro José
Bom dia!
Não há outra restrição?
É igual perguntar quais os pares de inteiros (x,y) tais que x|y, com x=b+1
e y=a+1.

Saudações,
PJMS

Em qua., 18 de mar. de 2020 às 08:51, marcone augusto araújo borges <
marconeborge...@hotmail.com> escreveu:

> Determine todos os pares de inteiros a e b tais que a divide b+1 e b
> divide a+1
> Desde já agradeço
> --
> Esta mensagem foi verificada pelo sistema de antivírus e
> acredita-se estar livre de perigo.
>

-- 
Esta mensagem foi verificada pelo sistema de antiv�rus e
 acredita-se estar livre de perigo.



Re: [obm-l] Problema

2020-03-17 Por tôpico Pedro José
Boa noite!
Você já formulou esse problema em set/2019 e Daniel Jelin apresentou uma
bela solução.
Saudações,
PJMS

Em ter, 17 de mar de 2020 19:26,  escreveu:

> Problema
> Um mágico e seu assistente realizam um truque da maneira seguinte. Existem
> 12 caixas vazias e fechadas, colocadas em fila. O mágico sai da sala e uma
> pessoa do público escolhe duas caixas e esconde em cada uma delas uma
> moeda, deixando a fila de caixas da mesma forma como era, mas o assistente
> sabe quais são as duas caixas que têm as moedas. O mágico retorna para a
> sala e o assistente escolhe uma caixa que ele sabe que está vazia. Das
> restantes, o mágico então escolhe quatro caixas que são abertas
> simultaneamente. O objetivo do mágico é que, entre essas quatro caixas,
> duas contenham as moedas.
> Desenvolva um método que permita que o mágico e seu assistente realizem a
> mágica com sucesso
>
>
> --
> Esta mensagem foi verificada pelo sistema de antivírus e
> acredita-se estar livre de perigo.Booa
>

-- 
Esta mensagem foi verificada pelo sistema de antiv�rus e
 acredita-se estar livre de perigo.



[obm-l] Re: [obm-l] Re: [obm-l] Re: [obm-l] Re: [obm-l] Re: [obm-l] Re: [obm-l] Teoria dos números

2020-03-17 Por tôpico Pedro José
Boa noite!
Aí, como dizia minha falecida vó, são outros quinhentos.
Como nas propostas anteriores n era natural. Vamos seguir nessa linha, se
não for reformule o problema.
Seja f(n)=   n (427 - 90n - 70n^2 + 45n^3 + 18n^4)
f(0)=0 qualquer natural divide, portanto, é indiferente.
f(1)= 330
f(2)= 1230
É fácil verificar que mdc(330,1230)=30 então D<=30, onde D é o máximo
inteiro que divide f(n) para todo n natural.
f(n) = 7n +5n^4 + 8 n^5 mod 10.
f(0)=0 mod10
f(1)= 20 = 0 mod10
f(2)= 350= 0 mod10
f(3)= 2370 = 0 mod10
f(4)= 9500 = 0 mod10
f(5)= 28160 = 0 mod10
f(6)=68730 = 0 mod10
f(7)=146510 = 0 mod10
f(8)=282680 = 0 mod10
f(9)=505260 = 0 mod10
logo 10 | f(n) para qualquer n natural.

f(n) = n -n^3 mod 3
f(0) = 0 mod 3
f(1) = 0 mod 3
f(2)= -6 = 0 mod 3
logo 3| f(n) para todo n natural
então D = 30.

Saudações,
PJMS



Em ter., 17 de mar. de 2020 às 11:57, Israel Meireles Chrisostomo <
israelmchrisost...@gmail.com> escreveu:

> Sim é isso q eu quis dizer
>
> Em ter, 17 de mar de 2020 11:12, Carlos Gustavo Tamm de Araujo Moreira <
> g...@impa.br> escreveu:
>
>> Acho que a pergunta deve ser qual é o maior inteiro positivo que divide
>> essa expressão para todo valor de n ao mesmo tempo.
>>
>> On Tue, Mar 17, 2020 at 6:58 AM Pedro José  wrote:
>>
>>> Bom dia!
>>> Se você considerar a expressão n(427-90n-70n^2+45n^3+18n^4)
>>> D=|n(427-90n-70n^2+45n^3+18n^4)|
>>> Por exemplo, n=1
>>> D=330.
>>> Agora se liberar n para variar D tende a oo.
>>>
>>> Se n for raiz da expressão, também tende a oi, pois qualquer inteiro
>>> divide 0.
>>>
>>>
>>> Em seg, 16 de mar de 2020 22:16, Israel Meireles Chrisostomo <
>>> israelmchrisost...@gmail.com> escreveu:
>>>
>>>> não entendi
>>>>
>>>> Em seg., 16 de mar. de 2020 às 22:01, Pedro José 
>>>> escreveu:
>>>>
>>>>> Para um dado n é o módulo do valor da expressão.
>>>>>
>>>>> Em seg, 16 de mar de 2020 21:49, Pedro José 
>>>>> escreveu:
>>>>>
>>>>>> Boa noite!
>>>>>> O módulo dessa expressão tende a oo. Não existe máximo.
>>>>>> Saudações,
>>>>>> PJMS
>>>>>>
>>>>>> Em seg, 16 de mar de 2020 20:36, Israel Meireles Chrisostomo <
>>>>>> israelmchrisost...@gmail.com> escreveu:
>>>>>>
>>>>>>> Qual é o maior inteiro que divide  n (427 - 90n - 70n^2 + 45n^3 +
>>>>>>> 18n^4)?
>>>>>>>
>>>>>>> --
>>>>>>> Israel Meireles Chrisostomo
>>>>>>>
>>>>>>> --
>>>>>>> Esta mensagem foi verificada pelo sistema de antivírus e
>>>>>>> acredita-se estar livre de perigo.
>>>>>>
>>>>>>
>>>>> --
>>>>> Esta mensagem foi verificada pelo sistema de antivírus e
>>>>> acredita-se estar livre de perigo.
>>>>
>>>>
>>>>
>>>> --
>>>> Israel Meireles Chrisostomo
>>>>
>>>> --
>>>> Esta mensagem foi verificada pelo sistema de antivírus e
>>>> acredita-se estar livre de perigo.
>>>
>>>
>>> --
>>> Esta mensagem foi verificada pelo sistema de antivírus e
>>> acredita-se estar livre de perigo.
>>
>>
>> --
>> Esta mensagem foi verificada pelo sistema de antivírus e
>> acredita-se estar livre de perigo.
>
>
> --
> Esta mensagem foi verificada pelo sistema de antivírus e
> acredita-se estar livre de perigo.

-- 
Esta mensagem foi verificada pelo sistema de antiv�rus e
 acredita-se estar livre de perigo.



[obm-l] Re: [obm-l] Re: [obm-l] Re: [obm-l] Teoria dos números

2020-03-17 Por tôpico Pedro José
Bom dia!
Se você considerar a expressão n(427-90n-70n^2+45n^3+18n^4)
D=|n(427-90n-70n^2+45n^3+18n^4)|
Por exemplo, n=1
D=330.
Agora se liberar n para variar D tende a oo.

Se n for raiz da expressão, também tende a oi, pois qualquer inteiro divide
0.


Em seg, 16 de mar de 2020 22:16, Israel Meireles Chrisostomo <
israelmchrisost...@gmail.com> escreveu:

> não entendi
>
> Em seg., 16 de mar. de 2020 às 22:01, Pedro José 
> escreveu:
>
>> Para um dado n é o módulo do valor da expressão.
>>
>> Em seg, 16 de mar de 2020 21:49, Pedro José 
>> escreveu:
>>
>>> Boa noite!
>>> O módulo dessa expressão tende a oo. Não existe máximo.
>>> Saudações,
>>> PJMS
>>>
>>> Em seg, 16 de mar de 2020 20:36, Israel Meireles Chrisostomo <
>>> israelmchrisost...@gmail.com> escreveu:
>>>
>>>> Qual é o maior inteiro que divide  n (427 - 90n - 70n^2 + 45n^3 +
>>>> 18n^4)?
>>>>
>>>> --
>>>> Israel Meireles Chrisostomo
>>>>
>>>> --
>>>> Esta mensagem foi verificada pelo sistema de antivírus e
>>>> acredita-se estar livre de perigo.
>>>
>>>
>> --
>> Esta mensagem foi verificada pelo sistema de antivírus e
>> acredita-se estar livre de perigo.
>
>
>
> --
> Israel Meireles Chrisostomo
>
> --
> Esta mensagem foi verificada pelo sistema de antivírus e
> acredita-se estar livre de perigo.

-- 
Esta mensagem foi verificada pelo sistema de antiv�rus e
 acredita-se estar livre de perigo.



[obm-l] Re: [obm-l] Teoria dos números

2020-03-16 Por tôpico Pedro José
Boa noite!
O módulo dessa expressão tende a oo. Não existe máximo.
Saudações,
PJMS

Em seg, 16 de mar de 2020 20:36, Israel Meireles Chrisostomo <
israelmchrisost...@gmail.com> escreveu:

> Qual é o maior inteiro que divide  n (427 - 90n - 70n^2 + 45n^3 + 18n^4)?
>
> --
> Israel Meireles Chrisostomo
>
> --
> Esta mensagem foi verificada pelo sistema de antivírus e
> acredita-se estar livre de perigo.

-- 
Esta mensagem foi verificada pelo sistema de antiv�rus e
 acredita-se estar livre de perigo.



[obm-l] Re: [obm-l] Teoria dos números

2020-03-16 Por tôpico Pedro José
Para um dado n é o módulo do valor da expressão.

Em seg, 16 de mar de 2020 21:49, Pedro José  escreveu:

> Boa noite!
> O módulo dessa expressão tende a oo. Não existe máximo.
> Saudações,
> PJMS
>
> Em seg, 16 de mar de 2020 20:36, Israel Meireles Chrisostomo <
> israelmchrisost...@gmail.com> escreveu:
>
>> Qual é o maior inteiro que divide  n (427 - 90n - 70n^2 + 45n^3 + 18n^4)?
>>
>> --
>> Israel Meireles Chrisostomo
>>
>> --
>> Esta mensagem foi verificada pelo sistema de antivírus e
>> acredita-se estar livre de perigo.
>
>

-- 
Esta mensagem foi verificada pelo sistema de antiv�rus e
 acredita-se estar livre de perigo.



[obm-l] Re: [obm-l] Teoria dos números

2020-03-16 Por tôpico Pedro José
Boa tarde!

Difícil generalizar. Mas consegui dois valores que não zeram a expressão
(soluções triviais), a duras penas, n=32 e n=43.
Vou continuar pensando no assunto.

Saudações,
PJMS


Em dom., 15 de mar. de 2020 às 18:48, Pedro José 
escreveu:

> Boa tarde!
> Faltou um contraexemplo.
> n=5
> 3^2*4^2*5^2*6^2*71 não é múltiplo de 11 nem de 37.
>
> Saudações,
> PJMS
>
> Em sáb, 14 de mar de 2020 19:47, Pedro José 
> escreveu:
>
>> Boa noite!
>> Creio que a pergunta correta seria, para que valores de n natural...
>> 8140=2^2*5*11*37. Então a solução só se dará para um subconjunto dos
>> naturais diferente de|N.
>>
>> Saudações,
>> PJMS
>>
>>
>> Em sex, 13 de mar de 2020 20:05, Israel Meireles Chrisostomo <
>> israelmchrisost...@gmail.com> escreveu:
>>
>>>   Dado n natural verifique se a expressão
>>>  (n − 2)² (n − 1)²n² (n + 1)² (4n²− 4n − 9)/8140 é um número inteiro
>>>
>>> --
>>> Israel Meireles Chrisostomo
>>>
>>> --
>>> Esta mensagem foi verificada pelo sistema de antivírus e
>>> acredita-se estar livre de perigo.
>>
>>

-- 
Esta mensagem foi verificada pelo sistema de antiv�rus e
 acredita-se estar livre de perigo.



[obm-l] Re: [obm-l] Teoria dos números

2020-03-15 Por tôpico Pedro José
Boa tarde!
Faltou um contraexemplo.
n=5
3^2*4^2*5^2*6^2*71 não é múltiplo de 11 nem de 37.

Saudações,
PJMS

Em sáb, 14 de mar de 2020 19:47, Pedro José  escreveu:

> Boa noite!
> Creio que a pergunta correta seria, para que valores de n natural...
> 8140=2^2*5*11*37. Então a solução só se dará para um subconjunto dos
> naturais diferente de|N.
>
> Saudações,
> PJMS
>
>
> Em sex, 13 de mar de 2020 20:05, Israel Meireles Chrisostomo <
> israelmchrisost...@gmail.com> escreveu:
>
>>   Dado n natural verifique se a expressão
>>  (n − 2)² (n − 1)²n² (n + 1)² (4n²− 4n − 9)/8140 é um número inteiro
>>
>> --
>> Israel Meireles Chrisostomo
>>
>> --
>> Esta mensagem foi verificada pelo sistema de antivírus e
>> acredita-se estar livre de perigo.
>
>

-- 
Esta mensagem foi verificada pelo sistema de antiv�rus e
 acredita-se estar livre de perigo.



[obm-l] Re: [obm-l] Teoria dos números

2020-03-14 Por tôpico Pedro José
Boa noite!
Creio que a pergunta correta seria, para que valores de n natural...
8140=2^2*5*11*37. Então a solução só se dará para um subconjunto dos
naturais diferente de|N.

Saudações,
PJMS


Em sex, 13 de mar de 2020 20:05, Israel Meireles Chrisostomo <
israelmchrisost...@gmail.com> escreveu:

>   Dado n natural verifique se a expressão
>  (n − 2)² (n − 1)²n² (n + 1)² (4n²− 4n − 9)/8140 é um número inteiro
>
> --
> Israel Meireles Chrisostomo
>
> --
> Esta mensagem foi verificada pelo sistema de antivírus e
> acredita-se estar livre de perigo.

-- 
Esta mensagem foi verificada pelo sistema de antiv�rus e
 acredita-se estar livre de perigo.



[obm-l] período de dízima

2020-03-10 Por tôpico Pedro José
Boa noite!

Alguém poderia provar ou derrubar a conjectura a seguir?

Seja s/t uma fração em que t não divide s e (s,t)=1; seja t=2^a.2^b.n
O número de algarismos da parte não periódica é o max(a,b) e o número de
algarismos da parte periódica é a ord 10 mod n.
Representação decimal.

Saudações,
PJMS

-- 
Esta mensagem foi verificada pelo sistema de antiv�rus e
 acredita-se estar livre de perigo.



[obm-l] Re: [obm-l] Re: [obm-l] Re: [obm-l] Ajuda com dízima

2020-03-09 Por tôpico Pedro José
Boa noite!
Errata da nota anterior independente de m e não de m, supondo (m,n)=1 e m/n
não inteiro.
Outro ponto não há necessidade a verificação de se o proposto vale para
quando n for múltiplo de 2 ou de 10, pois a ordem m mod n só existe se
(10,n)=1. Foi bobagem só ter aventado a possibilidade.
não coloquei como cheguei a conclusão de que era ordem 10 mod n, pois achei
bem intuitivo. Mas na hora que fui mostrar, achei complicado o que julgara
fácil. Mas quanto a isso estou seguro.
Para (n,m)=1 e (n,10)=1 e n/m não inteiro.
Se m>n pode-se representar por uma fração q j/n com q, j e n inteiros e
(j,n)=1 pois m=qn+j e se d<>1 divide n e j então d|m pois m é uma Z
combinação linear de j e n. Absurdo pois(m,n)=1 por hipótese.
Então sem perda de generalidade podemos só trabalhar para o caso m=2, está correta.

Saudações,
PJMS


Em dom, 8 de mar de 2020 16:09, Pedro José  escreveu:

> Boa tarde!
> Douglas,
> Não creio, no meu entendimento 3^2003 é o número de algarismos da dízima
> pois, é a ordem 10 módulo 3^2005.
> 1/3^2005 tem uma montoeira de algarismos zeros no início do período o que
> não acontece em 3^2005.
> O número de algarismos do período de uma dízima m/n, pelo menos quando n
> não  é múltiplo dos primos 2 e 5 é ord10mod n e independe de n. Nao
> verifiquei se vale sem a restriçao.
> Por exemplo o período de 1/7 é 142857 e ord 10 mod 7 = 6.
> Se aquele fosse o período da dízima bastaria fazer n =[log10 (3^2003)+1]
> onde colchetes representam parte inteira..
> Minha dúvida está na prova por absurdo, que ord 10 mod 3^n= 3^(n-2).
>
> Saudações,
> PJMS
>
>
>
> Em dom, 8 de mar de 2020 11:31, Prof. Douglas Oliveira <
> profdouglaso.del...@gmail.com> escreveu:
>
>> 3^2003 é o período certo??, o número de dígitos disso que seria a
>> pergunta.
>> 
>>
>> Douglas oliveira
>>
>> Em dom, 8 de mar de 2020 11:13, Prof. Douglas Oliveira <
>> profdouglaso.del...@gmail.com> escreveu:
>>
>>> Olá Pedro, primeiramente muito obrigado pela sua solução, eu dei uma
>>> olhada rápida e acredito estar correta. Estarei olhando com mais calma,
>>> assim que tiver um tempinho.
>>>
>>> Douglas Oliveira.
>>>
>>> Em dom, 8 de mar de 2020 11:05, Pedro José 
>>> escreveu:
>>>
>>>> Bom dia!
>>>> Não compreendi o porquê dessa questão ter sido vilipendiada. Não sou
>>>> matemático, sou pitaqueiro, ouço falar em inteiros de Gauss vou atrás, de
>>>> espaço fibrado idem, equações de Pell idem..., o que não consigo aprender
>>>> fica para o futuro. Quando me aposentar  cursar uma faculdade de
>>>> matemática. Portanto, nem tudo que resolvo me dá segurança. Reforço, alguém
>>>> poderia me informar se está correto?
>>>> Saudações,
>>>> PJMS.
>>>>
>>>> Em ter, 3 de mar de 2020 12:03, Pedro José 
>>>> escreveu:
>>>>
>>>>> Boa tarde!
>>>>> Não me senti muito seguro na resposta. Está correto?
>>>>>
>>>>> Saudações,
>>>>> PJMS
>>>>>
>>>>> Em seg., 2 de mar. de 2020 às 23:27, Pedro José 
>>>>> escreveu:
>>>>>
>>>>>> Boa noite!
>>>>>> Creio ter conseguido.
>>>>>> Seja k o número de algarismos do período de 1/3^2005. Como (3,10)=1
>>>>>> então k é a ordem 10 mod 3^2005.
>>>>>> 3^(n-2)|| 3^(n-2); (|| significa divide exatamente) e 3^2||10-1 então
>>>>>> pelo lema de Hensel 3^n||10^(3^(n-2))-1 para n>=2.(i)
>>>>>> Então 10^(3^(n-2))= 1 mod 3^n logo ord 10 mod 3^n | 3^(n-2) Se
>>>>>> x<>3^(n-2) absurdo; pois, teria que ser 3^k com k>>>>> e por (i) 3^(k+2)||10^(3^k)-1 e k+2>>>>> ord 10 mod 3^2005 =3^2003
>>>>>> 3^2003 algarismos
>>>>>> Saudações,
>>>>>> PJMS
>>>>>>
>>>>>> Em sáb, 29 de fev de 2020 16:13, Pedro José 
>>>>>> escreveu:
>>>>>>
>>>>>>> Boa tarde!
>>>>>>> 3^2005 e não 10^2005.
>>>>>>>
>>>>>>> Em sex, 28 de fev de 2020 16:06, Pedro José 
>>>>>>> escreveu:
>>>>>>>
>>>>>>>> Boa tarde!
>>>>>>>> Questão complicada.
>>>>>>>> Como (3^2005; 10) =1, o número de dígitos x deve ser a ordem de 10
>>>>>>>> mod 10^2005. Portanto x | 2*3^2004.
>>>>>>>> Se 10 fosse uma raiz primitiva de 3^2005 aí da

[obm-l] Re: [obm-l] Re: [obm-l] Re: [obm-l] Ajuda com dízima

2020-03-08 Por tôpico Pedro José
Boa tarde!
Douglas,
Não creio, no meu entendimento 3^2003 é o número de algarismos da dízima
pois, é a ordem 10 módulo 3^2005.
1/3^2005 tem uma montoeira de algarismos zeros no início do período o que
não acontece em 3^2005.
O número de algarismos do período de uma dízima m/n, pelo menos quando n
não  é múltiplo dos primos 2 e 5 é ord10mod n e independe de n. Nao
verifiquei se vale sem a restriçao.
Por exemplo o período de 1/7 é 142857 e ord 10 mod 7 = 6.
Se aquele fosse o período da dízima bastaria fazer n =[log10 (3^2003)+1]
onde colchetes representam parte inteira..
Minha dúvida está na prova por absurdo, que ord 10 mod 3^n= 3^(n-2).

Saudações,
PJMS



Em dom, 8 de mar de 2020 11:31, Prof. Douglas Oliveira <
profdouglaso.del...@gmail.com> escreveu:

> 3^2003 é o período certo??, o número de dígitos disso que seria a pergunta.
> 
>
> Douglas oliveira
>
> Em dom, 8 de mar de 2020 11:13, Prof. Douglas Oliveira <
> profdouglaso.del...@gmail.com> escreveu:
>
>> Olá Pedro, primeiramente muito obrigado pela sua solução, eu dei uma
>> olhada rápida e acredito estar correta. Estarei olhando com mais calma,
>> assim que tiver um tempinho.
>>
>> Douglas Oliveira.
>>
>> Em dom, 8 de mar de 2020 11:05, Pedro José 
>> escreveu:
>>
>>> Bom dia!
>>> Não compreendi o porquê dessa questão ter sido vilipendiada. Não sou
>>> matemático, sou pitaqueiro, ouço falar em inteiros de Gauss vou atrás, de
>>> espaço fibrado idem, equações de Pell idem..., o que não consigo aprender
>>> fica para o futuro. Quando me aposentar  cursar uma faculdade de
>>> matemática. Portanto, nem tudo que resolvo me dá segurança. Reforço, alguém
>>> poderia me informar se está correto?
>>> Saudações,
>>> PJMS.
>>>
>>> Em ter, 3 de mar de 2020 12:03, Pedro José 
>>> escreveu:
>>>
>>>> Boa tarde!
>>>> Não me senti muito seguro na resposta. Está correto?
>>>>
>>>> Saudações,
>>>> PJMS
>>>>
>>>> Em seg., 2 de mar. de 2020 às 23:27, Pedro José 
>>>> escreveu:
>>>>
>>>>> Boa noite!
>>>>> Creio ter conseguido.
>>>>> Seja k o número de algarismos do período de 1/3^2005. Como (3,10)=1
>>>>> então k é a ordem 10 mod 3^2005.
>>>>> 3^(n-2)|| 3^(n-2); (|| significa divide exatamente) e 3^2||10-1 então
>>>>> pelo lema de Hensel 3^n||10^(3^(n-2))-1 para n>=2.(i)
>>>>> Então 10^(3^(n-2))= 1 mod 3^n logo ord 10 mod 3^n | 3^(n-2) Se
>>>>> x<>3^(n-2) absurdo; pois, teria que ser 3^k com k>>>> e por (i) 3^(k+2)||10^(3^k)-1 e k+2>>>> ord 10 mod 3^2005 =3^2003
>>>>> 3^2003 algarismos
>>>>> Saudações,
>>>>> PJMS
>>>>>
>>>>> Em sáb, 29 de fev de 2020 16:13, Pedro José 
>>>>> escreveu:
>>>>>
>>>>>> Boa tarde!
>>>>>> 3^2005 e não 10^2005.
>>>>>>
>>>>>> Em sex, 28 de fev de 2020 16:06, Pedro José 
>>>>>> escreveu:
>>>>>>
>>>>>>> Boa tarde!
>>>>>>> Questão complicada.
>>>>>>> Como (3^2005; 10) =1, o número de dígitos x deve ser a ordem de 10
>>>>>>> mod 10^2005. Portanto x | 2*3^2004.
>>>>>>> Se 10 fosse uma raiz primitiva de 3^2005 aí daria x=2.3^2004. Mas
>>>>>>> parece que não...
>>>>>>> Achar essa ordem é muito difícil, pelo menos para mim.
>>>>>>> O que achei empiricamente foi a conjectura: ord 10 mod 3^n = 3^(n-2)
>>>>>>> para n>=2.
>>>>>>> Será que sai por indução, aí seriam 3^2003 algarismos. Caso a
>>>>>>> conjectura esteja correta.
>>>>>>>
>>>>>>> Saudações,
>>>>>>> PJMS
>>>>>>>
>>>>>>> Em qui., 20 de fev. de 2020 às 18:12, Prof. Douglas Oliveira <
>>>>>>> profdouglaso.del...@gmail.com> escreveu:
>>>>>>>
>>>>>>>> Qual o número de dígitos do período de 1/(3^2005) ?
>>>>>>>>
>>>>>>>>
>>>>>>>> Saudações
>>>>>>>> Douglas Oliveira
>>>>>>>>
>>>>>>>> --
>>>>>>>> Esta mensagem foi verificada pelo sistema de antivírus e
>>>>>>>> acredita-se estar livre de perigo.
>>>>>>>
>>>>>>>
>>> --
>>> Esta mensagem foi verificada pelo sistema de antivírus e
>>> acredita-se estar livre de perigo.
>>
>>
> --
> Esta mensagem foi verificada pelo sistema de antivírus e
> acredita-se estar livre de perigo.

-- 
Esta mensagem foi verificada pelo sistema de antiv�rus e
 acredita-se estar livre de perigo.



[obm-l] Re: [obm-l] Ajuda com dízima

2020-03-08 Por tôpico Pedro José
Bom dia!
Não compreendi o porquê dessa questão ter sido vilipendiada. Não sou
matemático, sou pitaqueiro, ouço falar em inteiros de Gauss vou atrás, de
espaço fibrado idem, equações de Pell idem..., o que não consigo aprender
fica para o futuro. Quando me aposentar  cursar uma faculdade de
matemática. Portanto, nem tudo que resolvo me dá segurança. Reforço, alguém
poderia me informar se está correto?
Saudações,
PJMS.

Em ter, 3 de mar de 2020 12:03, Pedro José  escreveu:

> Boa tarde!
> Não me senti muito seguro na resposta. Está correto?
>
> Saudações,
> PJMS
>
> Em seg., 2 de mar. de 2020 às 23:27, Pedro José 
> escreveu:
>
>> Boa noite!
>> Creio ter conseguido.
>> Seja k o número de algarismos do período de 1/3^2005. Como (3,10)=1 então
>> k é a ordem 10 mod 3^2005.
>> 3^(n-2)|| 3^(n-2); (|| significa divide exatamente) e 3^2||10-1 então
>> pelo lema de Hensel 3^n||10^(3^(n-2))-1 para n>=2.(i)
>> Então 10^(3^(n-2))= 1 mod 3^n logo ord 10 mod 3^n | 3^(n-2) Se x<>3^(n-2)
>> absurdo; pois, teria que ser 3^k com k> e por (i) 3^(k+2)||10^(3^k)-1 e k+2> ord 10 mod 3^2005 =3^2003
>> 3^2003 algarismos
>> Saudações,
>> PJMS
>>
>> Em sáb, 29 de fev de 2020 16:13, Pedro José 
>> escreveu:
>>
>>> Boa tarde!
>>> 3^2005 e não 10^2005.
>>>
>>> Em sex, 28 de fev de 2020 16:06, Pedro José 
>>> escreveu:
>>>
>>>> Boa tarde!
>>>> Questão complicada.
>>>> Como (3^2005; 10) =1, o número de dígitos x deve ser a ordem de 10 mod
>>>> 10^2005. Portanto x | 2*3^2004.
>>>> Se 10 fosse uma raiz primitiva de 3^2005 aí daria x=2.3^2004. Mas
>>>> parece que não...
>>>> Achar essa ordem é muito difícil, pelo menos para mim.
>>>> O que achei empiricamente foi a conjectura: ord 10 mod 3^n = 3^(n-2)
>>>> para n>=2.
>>>> Será que sai por indução, aí seriam 3^2003 algarismos. Caso a
>>>> conjectura esteja correta.
>>>>
>>>> Saudações,
>>>> PJMS
>>>>
>>>> Em qui., 20 de fev. de 2020 às 18:12, Prof. Douglas Oliveira <
>>>> profdouglaso.del...@gmail.com> escreveu:
>>>>
>>>>> Qual o número de dígitos do período de 1/(3^2005) ?
>>>>>
>>>>>
>>>>> Saudações
>>>>> Douglas Oliveira
>>>>>
>>>>> --
>>>>> Esta mensagem foi verificada pelo sistema de antivírus e
>>>>> acredita-se estar livre de perigo.
>>>>
>>>>

-- 
Esta mensagem foi verificada pelo sistema de antiv�rus e
 acredita-se estar livre de perigo.



[obm-l] Re: [obm-l] Ajuda com dízima

2020-03-03 Por tôpico Pedro José
Boa tarde!
Não me senti muito seguro na resposta. Está correto?

Saudações,
PJMS

Em seg., 2 de mar. de 2020 às 23:27, Pedro José 
escreveu:

> Boa noite!
> Creio ter conseguido.
> Seja k o número de algarismos do período de 1/3^2005. Como (3,10)=1 então
> k é a ordem 10 mod 3^2005.
> 3^(n-2)|| 3^(n-2); (|| significa divide exatamente) e 3^2||10-1 então pelo
> lema de Hensel 3^n||10^(3^(n-2))-1 para n>=2.(i)
> Então 10^(3^(n-2))= 1 mod 3^n logo ord 10 mod 3^n | 3^(n-2) Se x<>3^(n-2)
> absurdo; pois, teria que ser 3^k com k e por (i) 3^(k+2)||10^(3^k)-1 e k+2 ord 10 mod 3^2005 =3^2003
> 3^2003 algarismos
> Saudações,
> PJMS
>
> Em sáb, 29 de fev de 2020 16:13, Pedro José 
> escreveu:
>
>> Boa tarde!
>> 3^2005 e não 10^2005.
>>
>> Em sex, 28 de fev de 2020 16:06, Pedro José 
>> escreveu:
>>
>>> Boa tarde!
>>> Questão complicada.
>>> Como (3^2005; 10) =1, o número de dígitos x deve ser a ordem de 10 mod
>>> 10^2005. Portanto x | 2*3^2004.
>>> Se 10 fosse uma raiz primitiva de 3^2005 aí daria x=2.3^2004. Mas parece
>>> que não...
>>> Achar essa ordem é muito difícil, pelo menos para mim.
>>> O que achei empiricamente foi a conjectura: ord 10 mod 3^n = 3^(n-2)
>>> para n>=2.
>>> Será que sai por indução, aí seriam 3^2003 algarismos. Caso a conjectura
>>> esteja correta.
>>>
>>> Saudações,
>>> PJMS
>>>
>>> Em qui., 20 de fev. de 2020 às 18:12, Prof. Douglas Oliveira <
>>> profdouglaso.del...@gmail.com> escreveu:
>>>
>>>> Qual o número de dígitos do período de 1/(3^2005) ?
>>>>
>>>>
>>>> Saudações
>>>> Douglas Oliveira
>>>>
>>>> --
>>>> Esta mensagem foi verificada pelo sistema de antivírus e
>>>> acredita-se estar livre de perigo.
>>>
>>>

-- 
Esta mensagem foi verificada pelo sistema de antiv�rus e
 acredita-se estar livre de perigo.



[obm-l] Re: [obm-l] Ajuda com dízima

2020-03-02 Por tôpico Pedro José
Boa noite!
Creio ter conseguido.
Seja k o número de algarismos do período de 1/3^2005. Como (3,10)=1 então k
é a ordem 10 mod 3^2005.
3^(n-2)|| 3^(n-2); (|| significa divide exatamente) e 3^2||10-1 então pelo
lema de Hensel 3^n||10^(3^(n-2))-1 para n>=2.(i)
Então 10^(3^(n-2))= 1 mod 3^n logo ord 10 mod 3^n | 3^(n-2) Se x<>3^(n-2)
absurdo; pois, teria que ser 3^k com k escreveu:

> Boa tarde!
> 3^2005 e não 10^2005.
>
> Em sex, 28 de fev de 2020 16:06, Pedro José 
> escreveu:
>
>> Boa tarde!
>> Questão complicada.
>> Como (3^2005; 10) =1, o número de dígitos x deve ser a ordem de 10 mod
>> 10^2005. Portanto x | 2*3^2004.
>> Se 10 fosse uma raiz primitiva de 3^2005 aí daria x=2.3^2004. Mas parece
>> que não...
>> Achar essa ordem é muito difícil, pelo menos para mim.
>> O que achei empiricamente foi a conjectura: ord 10 mod 3^n = 3^(n-2) para
>> n>=2.
>> Será que sai por indução, aí seriam 3^2003 algarismos. Caso a conjectura
>> esteja correta.
>>
>> Saudações,
>> PJMS
>>
>> Em qui., 20 de fev. de 2020 às 18:12, Prof. Douglas Oliveira <
>> profdouglaso.del...@gmail.com> escreveu:
>>
>>> Qual o número de dígitos do período de 1/(3^2005) ?
>>>
>>>
>>> Saudações
>>> Douglas Oliveira
>>>
>>> --
>>> Esta mensagem foi verificada pelo sistema de antivírus e
>>> acredita-se estar livre de perigo.
>>
>>

-- 
Esta mensagem foi verificada pelo sistema de antiv�rus e
 acredita-se estar livre de perigo.



[obm-l] Re: [obm-l] Ajuda com dízima

2020-02-29 Por tôpico Pedro José
Boa tarde!
3^2005 e não 10^2005.

Em sex, 28 de fev de 2020 16:06, Pedro José  escreveu:

> Boa tarde!
> Questão complicada.
> Como (3^2005; 10) =1, o número de dígitos x deve ser a ordem de 10 mod
> 10^2005. Portanto x | 2*3^2004.
> Se 10 fosse uma raiz primitiva de 3^2005 aí daria x=2.3^2004. Mas parece
> que não...
> Achar essa ordem é muito difícil, pelo menos para mim.
> O que achei empiricamente foi a conjectura: ord 10 mod 3^n = 3^(n-2) para
> n>=2.
> Será que sai por indução, aí seriam 3^2003 algarismos. Caso a conjectura
> esteja correta.
>
> Saudações,
> PJMS
>
> Em qui., 20 de fev. de 2020 às 18:12, Prof. Douglas Oliveira <
> profdouglaso.del...@gmail.com> escreveu:
>
>> Qual o número de dígitos do período de 1/(3^2005) ?
>>
>>
>> Saudações
>> Douglas Oliveira
>>
>> --
>> Esta mensagem foi verificada pelo sistema de antivírus e
>> acredita-se estar livre de perigo.
>
>

-- 
Esta mensagem foi verificada pelo sistema de antiv�rus e
 acredita-se estar livre de perigo.



[obm-l] Re: [obm-l] Ajuda com dízima

2020-02-28 Por tôpico Pedro José
Boa tarde!
Questão complicada.
Como (3^2005; 10) =1, o número de dígitos x deve ser a ordem de 10 mod
10^2005. Portanto x | 2*3^2004.
Se 10 fosse uma raiz primitiva de 3^2005 aí daria x=2.3^2004. Mas parece
que não...
Achar essa ordem é muito difícil, pelo menos para mim.
O que achei empiricamente foi a conjectura: ord 10 mod 3^n = 3^(n-2) para
n>=2.
Será que sai por indução, aí seriam 3^2003 algarismos. Caso a conjectura
esteja correta.

Saudações,
PJMS

Em qui., 20 de fev. de 2020 às 18:12, Prof. Douglas Oliveira <
profdouglaso.del...@gmail.com> escreveu:

> Qual o número de dígitos do período de 1/(3^2005) ?
>
>
> Saudações
> Douglas Oliveira
>
> --
> Esta mensagem foi verificada pelo sistema de antivírus e
> acredita-se estar livre de perigo.

-- 
Esta mensagem foi verificada pelo sistema de antiv�rus e
 acredita-se estar livre de perigo.



[obm-l] Re: [obm-l] Re: [obm-l] Cálculo do Volume de um Sólido

2020-02-12 Por tôpico Pedro José
Bom dia!
Alguém poderia me ajudar e mostrar onde errei os limites? Resolvendo por
integral tripla, usando f(x,y,z)=1.

Grato,
PJMS

Em ter, 11 de fev de 2020 13:11, Pedro José  escreveu:

> Boa tarde!
>
> Resolvi por método numérico usando, pelo menos penso eu, os mesmos limites
> e encontrei 2,1329, muito próximo da resposta. Gostaria que alguém me
> ajudasse onde errei na integral tripla.
> Usei z^2-y e 2z-y como os limites para integral em dx. Em seguida, z^2 e
> 2z para dy e finalmente 0 e 2 para dz.
> Onde está o erro?
> Grato,
> PJMS
>
> Em ter, 11 de fev de 2020 12:49, Claudio Buffara <
> claudio.buff...@gmail.com> escreveu:
>
>> O sólido é a região do 1o octante (todas as coordenadas positivas)
>> compreendida entre os planos x-z e y-z, acima do plano z = (x+y)/2 e abaixo
>> da z = raiz(x+y).
>> A superfície e o plano se intersectam numa reta:
>> raiz(x+y) = (x+y)/2 ==> x+y = (x+y)^2/4 ==> x+y = 4, contida no plano z =
>> 2.
>>
>> Assim, o volume pode ser dado pela diferença entre duas integrais duplas,
>> calculadas sobre o domínio D, no plano x-y, dado por x > 0, y > 0 e x+y = 4.
>> Volume = Integral(D) raiz(x+y)*dA - Integral(D) (x+y)/2*dA.
>>
>> Usando coordenadas cartesianas, a primeira integral fica:
>> Integral(x=0...4)Integral(y=0...4-x)*raiz(x+y)*dy*dx
>> = Integral(0...4) (2/3)*(4^(3/2) - x^(3/2))*dx
>> = Integral(0...4) (16/3 - (2/3)*x^(3/2))
>> = 64/3 - (4/15)*4^(5/2)
>> = 64/3 - 128/15
>> = 64/5
>>
>> A segunda integral é:
>> Integral(x=0...4)Integral(y=0...4-x) (x+y)/2*dy*dx
>> = Integral(x=0...4) (1/2)*(x*(4-x) + (4-x)^2/2)*dx
>> = Integral(0...4) (4 - x^2/4)*dx
>> = 32/3
>>
>> Logo, o volume é 64/5 - 32/3 = 32/15  (se não errei nenhuma conta...)
>>
>> []s,
>> Claudio.
>>
>>
>> On Mon, Feb 3, 2020 at 8:55 PM Luiz Antonio Rodrigues <
>> rodrigue...@gmail.com> wrote:
>>
>>> Olá, pessoal!
>>> Tudo bem?
>>> Estou tentando resolver o seguinte problema:
>>>
>>> Ache o volume da região tridimensional definida por:
>>>
>>> z^2>>
>>> Sendo que:
>>> x>0 e y>0 e z>0
>>>
>>> Com o auxílio de um software eu consegui visualizar o sólido em questão.
>>> Eu calculei o volume do sólido girando em torno do eixo z e dividindo o
>>> resultado por 4.
>>> A resposta que eu obtive foi (16*pi)/15, que não está correta.
>>> Já refiz os cálculos muitas vezes e chego sempre na mesma resposta.
>>> Alguém pode me ajudar?
>>> Muito obrigado e um abraço!
>>>
>>> --
>>> Esta mensagem foi verificada pelo sistema de antivírus e
>>> acredita-se estar livre de perigo.
>>
>>
>> --
>> Esta mensagem foi verificada pelo sistema de antivírus e
>> acredita-se estar livre de perigo.
>
>

-- 
Esta mensagem foi verificada pelo sistema de antiv�rus e
 acredita-se estar livre de perigo.



[obm-l] Re: [obm-l] Re: [obm-l] Cálculo do Volume de um Sólido

2020-02-11 Por tôpico Pedro José
Boa tarde!

Resolvi por método numérico usando, pelo menos penso eu, os mesmos limites
e encontrei 2,1329, muito próximo da resposta. Gostaria que alguém me
ajudasse onde errei na integral tripla.
Usei z^2-y e 2z-y como os limites para integral em dx. Em seguida, z^2 e 2z
para dy e finalmente 0 e 2 para dz.
Onde está o erro?
Grato,
PJMS

Em ter, 11 de fev de 2020 12:49, Claudio Buffara 
escreveu:

> O sólido é a região do 1o octante (todas as coordenadas positivas)
> compreendida entre os planos x-z e y-z, acima do plano z = (x+y)/2 e abaixo
> da z = raiz(x+y).
> A superfície e o plano se intersectam numa reta:
> raiz(x+y) = (x+y)/2 ==> x+y = (x+y)^2/4 ==> x+y = 4, contida no plano z =
> 2.
>
> Assim, o volume pode ser dado pela diferença entre duas integrais duplas,
> calculadas sobre o domínio D, no plano x-y, dado por x > 0, y > 0 e x+y = 4.
> Volume = Integral(D) raiz(x+y)*dA - Integral(D) (x+y)/2*dA.
>
> Usando coordenadas cartesianas, a primeira integral fica:
> Integral(x=0...4)Integral(y=0...4-x)*raiz(x+y)*dy*dx
> = Integral(0...4) (2/3)*(4^(3/2) - x^(3/2))*dx
> = Integral(0...4) (16/3 - (2/3)*x^(3/2))
> = 64/3 - (4/15)*4^(5/2)
> = 64/3 - 128/15
> = 64/5
>
> A segunda integral é:
> Integral(x=0...4)Integral(y=0...4-x) (x+y)/2*dy*dx
> = Integral(x=0...4) (1/2)*(x*(4-x) + (4-x)^2/2)*dx
> = Integral(0...4) (4 - x^2/4)*dx
> = 32/3
>
> Logo, o volume é 64/5 - 32/3 = 32/15  (se não errei nenhuma conta...)
>
> []s,
> Claudio.
>
>
> On Mon, Feb 3, 2020 at 8:55 PM Luiz Antonio Rodrigues <
> rodrigue...@gmail.com> wrote:
>
>> Olá, pessoal!
>> Tudo bem?
>> Estou tentando resolver o seguinte problema:
>>
>> Ache o volume da região tridimensional definida por:
>>
>> z^2>
>> Sendo que:
>> x>0 e y>0 e z>0
>>
>> Com o auxílio de um software eu consegui visualizar o sólido em questão.
>> Eu calculei o volume do sólido girando em torno do eixo z e dividindo o
>> resultado por 4.
>> A resposta que eu obtive foi (16*pi)/15, que não está correta.
>> Já refiz os cálculos muitas vezes e chego sempre na mesma resposta.
>> Alguém pode me ajudar?
>> Muito obrigado e um abraço!
>>
>> --
>> Esta mensagem foi verificada pelo sistema de antivírus e
>> acredita-se estar livre de perigo.
>
>
> --
> Esta mensagem foi verificada pelo sistema de antivírus e
> acredita-se estar livre de perigo.

-- 
Esta mensagem foi verificada pelo sistema de antiv�rus e
 acredita-se estar livre de perigo.



[obm-l] Re: [obm-l] Re: [obm-l] Re: [obm-l] Re: [obm-l] Cálculo do Volume de um Sólido

2020-02-10 Por tôpico Pedro José
Boa noite!
Não sei onde errei está dando exatamente a metade 16/15.
Saudações,
PJMS

Em seg, 10 de fev de 2020 15:46, Luiz Antonio Rodrigues <
rodrigue...@gmail.com> escreveu:

> Olá, Pedro!
> Tudo bem?
> Obrigado pela resposta!
> A resposta realmente não tem pi: é 32/15.
> Eu percebi ontem que o meu erro foi fazer uma rotação em torno do eixo z.
> Se seccionarmos a figura no plano xy teremos um trapézio.
> Vou pensar na sua sugestão e tentar fazer tudo de novo.
> Muito obrigado!
> Abraços!
> Luiz
>
>
>
> Em seg, 10 de fev de 2020 1:38 PM, Pedro José 
> escreveu:
>
>> Boa tarde!
>> Como no caso você tem a resposta, facilitaria se a expusesse.
>> Para evitar que postemos soluções erradas.
>>
>> Saudações,
>> PJMS
>>
>> Em qui., 6 de fev. de 2020 às 07:41, Anderson Torres <
>> torres.anderson...@gmail.com> escreveu:
>>
>>> Em seg., 3 de fev. de 2020 às 20:55, Luiz Antonio Rodrigues
>>>  escreveu:
>>> >
>>> > Olá, pessoal!
>>> > Tudo bem?
>>> > Estou tentando resolver o seguinte problema:
>>> >
>>> > Ache o volume da região tridimensional definida por:
>>> >
>>> > z^2>> >
>>> > Sendo que:
>>> > x>0 e y>0 e z>0
>>> >
>>> > Com o auxílio de um software eu consegui visualizar o sólido em
>>> questão.
>>> > Eu calculei o volume do sólido girando em torno do eixo z e dividindo
>>> o resultado por 4.
>>> > A resposta que eu obtive foi (16*pi)/15, que não está correta.
>>> > Já refiz os cálculos muitas vezes e chego sempre na mesma resposta.
>>> > Alguém pode me ajudar?
>>>
>>> Tem como cê enviar as contas e o desenho que cê fez?
>>>
>>> > Muito obrigado e um abraço!
>>> >
>>> > --
>>> > Esta mensagem foi verificada pelo sistema de antivírus e
>>> > acredita-se estar livre de perigo.
>>>
>>> --
>>> Esta mensagem foi verificada pelo sistema de antivírus e
>>>  acredita-se estar livre de perigo.
>>>
>>>
>>> =
>>> Instru�ões para entrar na lista, sair da lista e usar a lista em
>>> http://www.mat.puc-rio.br/~obmlistas/obm-l.html
>>> =
>>>
>>
>> --
>> Esta mensagem foi verificada pelo sistema de antivírus e
>> acredita-se estar livre de perigo.
>
>
> --
> Esta mensagem foi verificada pelo sistema de antivírus e
> acredita-se estar livre de perigo.

-- 
Esta mensagem foi verificada pelo sistema de antiv�rus e
 acredita-se estar livre de perigo.



[obm-l] Re: [obm-l] Re: [obm-l] Cálculo do Volume de um Sólido

2020-02-10 Por tôpico Pedro José
Boa tarde!

Estou enferrujado.
Mas faria assim, e não vejo como aparecer PI() na resposta. Para mim é um
polinômio em z, aplicado em 0,2, o que dará um número racional.

Volume de z^2< x+y < 2z é igual ao volume de z^2 <= x+y <= 2z.

Int (0,2) Int (z2,2z) Int (z^2-y,^Z^2-x)  dxdydz. Os termos entre
parêntesis são os limites inferior e superior da integral. Int é o símbolo
da integral.

Como definir os intervalos de integração. O de x sai de graça z^2 < x + y <
2z. Basta jogar y para os dois lados da inequação.
Agora projetamos o sólido no Plano yZ, igualando x a 0 e obtemos que x
varia de z^2 a 2z.
Para achar o limite de z temos que z2<2z logo z varia de 0 a 2.
 Agora é resolver e verificar se dá a resposta,

Saudações,
PJMS



Em seg., 10 de fev. de 2020 às 13:25, Pedro José 
escreveu:

> Boa tarde!
> Como no caso você tem a resposta, facilitaria se a expusesse.
> Para evitar que postemos soluções erradas.
>
> Saudações,
> PJMS
>
> Em qui., 6 de fev. de 2020 às 07:41, Anderson Torres <
> torres.anderson...@gmail.com> escreveu:
>
>> Em seg., 3 de fev. de 2020 às 20:55, Luiz Antonio Rodrigues
>>  escreveu:
>> >
>> > Olá, pessoal!
>> > Tudo bem?
>> > Estou tentando resolver o seguinte problema:
>> >
>> > Ache o volume da região tridimensional definida por:
>> >
>> > z^2> >
>> > Sendo que:
>> > x>0 e y>0 e z>0
>> >
>> > Com o auxílio de um software eu consegui visualizar o sólido em questão.
>> > Eu calculei o volume do sólido girando em torno do eixo z e dividindo o
>> resultado por 4.
>> > A resposta que eu obtive foi (16*pi)/15, que não está correta.
>> > Já refiz os cálculos muitas vezes e chego sempre na mesma resposta.
>> > Alguém pode me ajudar?
>>
>> Tem como cê enviar as contas e o desenho que cê fez?
>>
>> > Muito obrigado e um abraço!
>> >
>> > --
>> > Esta mensagem foi verificada pelo sistema de antivírus e
>> > acredita-se estar livre de perigo.
>>
>> --
>> Esta mensagem foi verificada pelo sistema de antivírus e
>>  acredita-se estar livre de perigo.
>>
>>
>> =
>> Instru�ões para entrar na lista, sair da lista e usar a lista em
>> http://www.mat.puc-rio.br/~obmlistas/obm-l.html
>> =
>>
>

-- 
Esta mensagem foi verificada pelo sistema de antiv�rus e
 acredita-se estar livre de perigo.



[obm-l] Re: [obm-l] Re: [obm-l] Cálculo do Volume de um Sólido

2020-02-10 Por tôpico Pedro José
Boa tarde!
Como no caso você tem a resposta, facilitaria se a expusesse.
Para evitar que postemos soluções erradas.

Saudações,
PJMS

Em qui., 6 de fev. de 2020 às 07:41, Anderson Torres <
torres.anderson...@gmail.com> escreveu:

> Em seg., 3 de fev. de 2020 às 20:55, Luiz Antonio Rodrigues
>  escreveu:
> >
> > Olá, pessoal!
> > Tudo bem?
> > Estou tentando resolver o seguinte problema:
> >
> > Ache o volume da região tridimensional definida por:
> >
> > z^2 >
> > Sendo que:
> > x>0 e y>0 e z>0
> >
> > Com o auxílio de um software eu consegui visualizar o sólido em questão.
> > Eu calculei o volume do sólido girando em torno do eixo z e dividindo o
> resultado por 4.
> > A resposta que eu obtive foi (16*pi)/15, que não está correta.
> > Já refiz os cálculos muitas vezes e chego sempre na mesma resposta.
> > Alguém pode me ajudar?
>
> Tem como cê enviar as contas e o desenho que cê fez?
>
> > Muito obrigado e um abraço!
> >
> > --
> > Esta mensagem foi verificada pelo sistema de antivírus e
> > acredita-se estar livre de perigo.
>
> --
> Esta mensagem foi verificada pelo sistema de antivírus e
>  acredita-se estar livre de perigo.
>
>
> =
> Instru�ões para entrar na lista, sair da lista e usar a lista em
> http://www.mat.puc-rio.br/~obmlistas/obm-l.html
> =
>

-- 
Esta mensagem foi verificada pelo sistema de antiv�rus e
 acredita-se estar livre de perigo.



[obm-l] Re: [obm-l] O estranho ciclo de eclosão das cigarras

2019-12-16 Por tôpico Pedro José
Boa tarde!
Não seria 19 ao invés de 17.
1019=101*19

Saudações,
PJMS

Em seg, 16 de dez de 2019 12:38, jamil dasilva 
escreveu:

> Em Em 1919 um entomólogo descobriu um tipo de cigarra que depois veio a
> se descobrir só
> aparece em anos cujo menor divisor primo é 17. Se essa conjectura estiver
> correta responda:
> 1) Em que ano será a próxima eclosão ?
> 2) Quais os anos em que há eclosão no séc.XXI ?
> 3) Em que ano ocorrerá a 2020.º eclosão, partindo-se de 1919 como eclosão
> zero ?
> 4) Tomando-se 1919 como ano zero, em que ano ocorreu
> a eclosão de número -1, ou seja, em que ano deve
> ter ocorrido a 1.º eclosão antes de 1919 ?
> 1919 um entomólogo descobriu um tipo de cigarra que depois veio a se
> descobrir só aparece em anos cujo menor divisor primo é 17. Se essa
> conjectura estiver correta responda:
> Em 1919 u
> 2) Quais os anos em que há eclosão no séc.XXI ?
>
> 3) Em que ano ocorrerá a 2020.º eclosão, partindo-se
> de 1919 como eclosão zero ?
>
> 4) Tomando-se 1919 como ano zero, em que ano ocorreu
> a eclosão de número -1, ou seja, em que ano deve ter ocorrido a
> 1.º eclosão antes de 1919 ?
> 1) Em que ano será a próxima eclosão ?
>
> 2) Quais os anos em que há eclosão no séc.XXI ?
>
> 3) Em que ano ocorrerá a 2020.º eclosão, partindo-se
> de 1919 como eclosão zero ?
>
> 4) Tomando-se 1919 como ano zero, em que ano ocorreu
> a eclosão de número -1, ou seja, em que ano deve ter ocorrido a
> 1.º eclosão antes de 1919 ?
>
> --
> Esta mensagem foi verificada pelo sistema de antivírus e
> acredita-se estar livre de perigo.

-- 
Esta mensagem foi verificada pelo sistema de antiv�rus e
 acredita-se estar livre de perigo.



Re: [obm-l] Contagem , Coprimos e Divisores

2019-12-15 Por tôpico Pedro José
Boa noite!
Jamil,
correto mas não valeu. Foi muita barbeiragem.
Saudações,
PJMS

Em dom, 15 de dez de 2019 19:31, jamil dasilva 
escreveu:

> Correto: 2021
>
> Em dom., 15 de dez. de 2019 às 17:15, Pedro José 
> escreveu:
>
>> Na verdade 2.
>> 2021.
>> Por hoje chega..
>>
>> Em dom, 15 de dez de 2019 16:58, Pedro José 
>> escreveu:
>>
>>> Boa tarde!
>>> Hoje esta difícil.
>>> 8atenxe primeiro.
>>> 2027.
>>> Que vergonha
>>>
>>> Em dom, 15 de dez de 2019 16:55, Pedro José 
>>> escreveu:
>>>
>>>> Boa tarde!
>>>> Jamil me alertou do erro banal e fui por outro caminho.
>>>> 2019+x<> 2 mod2 ==> x<>1 mod2
>>>> 2019 +x<> 3 mod3 ==> x<>0 mod3
>>>> 2019 +x<> 5 mod5 ==> x<>1 mod5
>>>> 2019+x <> 7 mod 7 ==> x<>4 mod 17
>>>> O menor x queatende é 10
>>>> Portanto 2029 seria a resposta correta.
>>>> Acho que é primo.
>>>> Desculpe -me pela falha grosseira.
>>>>
>>>> Saudações,
>>>> PJMS
>>>>
>>>> Em dom, 15 de dez de 2019 14:04, Pedro José 
>>>> escreveu:
>>>>
>>>>> Boa tarde.
>>>>>
>>>>> 2019= 0 mod3 nã0 serve.
>>>>> É só fatorar sem usar esses primos.
>>>>> 11^3 <2019
>>>>> 11^2*13 <2019
>>>>> 11*13^2<2019
>>>>> 11^2*17=2057
>>>>> Acha o próximo
>>>>>
>>>>> Saudações.
>>>>>
>>>>> Em dom, 15 de dez de 2019 12:43, jamil dasilva 
>>>>> escreveu:
>>>>>
>>>>>> Se a eclosão de uma espécie de cigarra sempre acontece
>>>>>> em anos não divisíveis por 2, 3, 5 e 7, responda se ocorre
>>>>>> no presente ano de 2019 e qual o próximo ano ocorrerá após
>>>>>> 2019 ?
>>>>>>
>>>>>> --
>>>>>> Esta mensagem foi verificada pelo sistema de antivírus e
>>>>>> acredita-se estar livre de perigo.
>>>>>
>>>>>
>> --
>> Esta mensagem foi verificada pelo sistema de antivírus e
>> acredita-se estar livre de perigo.
>
>
> --
> Esta mensagem foi verificada pelo sistema de antivírus e
> acredita-se estar livre de perigo.

-- 
Esta mensagem foi verificada pelo sistema de antiv�rus e
 acredita-se estar livre de perigo.



Re: [obm-l] Contagem , Coprimos e Divisores

2019-12-15 Por tôpico Pedro José
Na verdade 2.
2021.
Por hoje chega..

Em dom, 15 de dez de 2019 16:58, Pedro José  escreveu:

> Boa tarde!
> Hoje esta difícil.
> 8atenxe primeiro.
> 2027.
> Que vergonha
>
> Em dom, 15 de dez de 2019 16:55, Pedro José 
> escreveu:
>
>> Boa tarde!
>> Jamil me alertou do erro banal e fui por outro caminho.
>> 2019+x<> 2 mod2 ==> x<>1 mod2
>> 2019 +x<> 3 mod3 ==> x<>0 mod3
>> 2019 +x<> 5 mod5 ==> x<>1 mod5
>> 2019+x <> 7 mod 7 ==> x<>4 mod 17
>> O menor x queatende é 10
>> Portanto 2029 seria a resposta correta.
>> Acho que é primo.
>> Desculpe -me pela falha grosseira.
>>
>> Saudações,
>> PJMS
>>
>> Em dom, 15 de dez de 2019 14:04, Pedro José 
>> escreveu:
>>
>>> Boa tarde.
>>>
>>> 2019= 0 mod3 nã0 serve.
>>> É só fatorar sem usar esses primos.
>>> 11^3 <2019
>>> 11^2*13 <2019
>>> 11*13^2<2019
>>> 11^2*17=2057
>>> Acha o próximo
>>>
>>> Saudações.
>>>
>>> Em dom, 15 de dez de 2019 12:43, jamil dasilva 
>>> escreveu:
>>>
>>>> Se a eclosão de uma espécie de cigarra sempre acontece
>>>> em anos não divisíveis por 2, 3, 5 e 7, responda se ocorre
>>>> no presente ano de 2019 e qual o próximo ano ocorrerá após
>>>> 2019 ?
>>>>
>>>> --
>>>> Esta mensagem foi verificada pelo sistema de antivírus e
>>>> acredita-se estar livre de perigo.
>>>
>>>

-- 
Esta mensagem foi verificada pelo sistema de antiv�rus e
 acredita-se estar livre de perigo.



Re: [obm-l] Contagem , Coprimos e Divisores

2019-12-15 Por tôpico Pedro José
Boa tarde!
Hoje esta difícil.
8atenxe primeiro.
2027.
Que vergonha

Em dom, 15 de dez de 2019 16:55, Pedro José  escreveu:

> Boa tarde!
> Jamil me alertou do erro banal e fui por outro caminho.
> 2019+x<> 2 mod2 ==> x<>1 mod2
> 2019 +x<> 3 mod3 ==> x<>0 mod3
> 2019 +x<> 5 mod5 ==> x<>1 mod5
> 2019+x <> 7 mod 7 ==> x<>4 mod 17
> O menor x queatende é 10
> Portanto 2029 seria a resposta correta.
> Acho que é primo.
> Desculpe -me pela falha grosseira.
>
> Saudações,
> PJMS
>
> Em dom, 15 de dez de 2019 14:04, Pedro José 
> escreveu:
>
>> Boa tarde.
>>
>> 2019= 0 mod3 nã0 serve.
>> É só fatorar sem usar esses primos.
>> 11^3 <2019
>> 11^2*13 <2019
>> 11*13^2<2019
>> 11^2*17=2057
>> Acha o próximo
>>
>> Saudações.
>>
>> Em dom, 15 de dez de 2019 12:43, jamil dasilva 
>> escreveu:
>>
>>> Se a eclosão de uma espécie de cigarra sempre acontece
>>> em anos não divisíveis por 2, 3, 5 e 7, responda se ocorre
>>> no presente ano de 2019 e qual o próximo ano ocorrerá após
>>> 2019 ?
>>>
>>> --
>>> Esta mensagem foi verificada pelo sistema de antivírus e
>>> acredita-se estar livre de perigo.
>>
>>

-- 
Esta mensagem foi verificada pelo sistema de antiv�rus e
 acredita-se estar livre de perigo.



Re: [obm-l] Contagem , Coprimos e Divisores

2019-12-15 Por tôpico Pedro José
Boa tarde!
Jamil me alertou do erro banal e fui por outro caminho.
2019+x<> 2 mod2 ==> x<>1 mod2
2019 +x<> 3 mod3 ==> x<>0 mod3
2019 +x<> 5 mod5 ==> x<>1 mod5
2019+x <> 7 mod 7 ==> x<>4 mod 17
O menor x queatende é 10
Portanto 2029 seria a resposta correta.
Acho que é primo.
Desculpe -me pela falha grosseira.

Saudações,
PJMS

Em dom, 15 de dez de 2019 14:04, Pedro José  escreveu:

> Boa tarde.
>
> 2019= 0 mod3 nã0 serve.
> É só fatorar sem usar esses primos.
> 11^3 <2019
> 11^2*13 <2019
> 11*13^2<2019
> 11^2*17=2057
> Acha o próximo
>
> Saudações.
>
> Em dom, 15 de dez de 2019 12:43, jamil dasilva 
> escreveu:
>
>> Se a eclosão de uma espécie de cigarra sempre acontece
>> em anos não divisíveis por 2, 3, 5 e 7, responda se ocorre
>> no presente ano de 2019 e qual o próximo ano ocorrerá após
>> 2019 ?
>>
>> --
>> Esta mensagem foi verificada pelo sistema de antivírus e
>> acredita-se estar livre de perigo.
>
>

-- 
Esta mensagem foi verificada pelo sistema de antiv�rus e
 acredita-se estar livre de perigo.



Re: [obm-l] Contagem , Coprimos e Divisores

2019-12-15 Por tôpico Pedro José
Boa tarde.

2019= 0 mod3 nã0 serve.
É só fatorar sem usar esses primos.
11^3 <2019
11^2*13 <2019
11*13^2<2019
11^2*17=2057
Acha o próximo

Saudações.

Em dom, 15 de dez de 2019 12:43, jamil dasilva 
escreveu:

> Se a eclosão de uma espécie de cigarra sempre acontece
> em anos não divisíveis por 2, 3, 5 e 7, responda se ocorre
> no presente ano de 2019 e qual o próximo ano ocorrerá após
> 2019 ?
>
> --
> Esta mensagem foi verificada pelo sistema de antivírus e
> acredita-se estar livre de perigo.

-- 
Esta mensagem foi verificada pelo sistema de antiv�rus e
 acredita-se estar livre de perigo.



[obm-l] Re: [obm-l] Re: [obm-l] Dúvida

2019-12-04 Por tôpico Pedro José
mpo dF/dx=0
> e dF/dy=0. O fato de pelo menos uma das derivadas parciais de F ser
> não-nula garante que não encontraremos problemas como os do parágrafo
> acima.
>
> abraços!
>
>
> Le mer. 4 déc. 2019 à 19:10, Pedro José  a écrit :
> >
> > Boa noite!
> > Estou dando uma repassada nas cônicas para auxiliar um filho de um amigo.
> > Dúvidas quanto à cônicas.
> > Alguns trabalhos até de mestrandos apontam a circunferência como sendo
> uma elipse, um caso particular.
> > Aprendera que o limite de uma elipse quando a distância entre os focos
> tendesse para zero era uma circunferência, não obstante a circunferência
> não é uma elipse.
> > A elipse tem dois focos. O que não ocorre na circunferência.
> > A elipse pode ser definida como o lugar geométrico do plano em que a
> razão entre a distância de um ponto ao foco direito e a distância entre
> esse ponto e uma reta (diretriz direita) é constante e menor que 1 e igual
> a excentricidade da cônica.
> > A circunferência não suporta tal definição.
> > Vejo muitos autores chamarem cônicas suaves.Significa que se escrevermos
> uma equação quadrática com F(x,y)=0 a função F(x,y) é suave?
> >
> > Grato!
> >
> > Saudações,
> > PJMS.
> >
> > --
> > Esta mensagem foi verificada pelo sistema de antivírus e
> > acredita-se estar livre de perigo.
>
> --
> Esta mensagem foi verificada pelo sistema de antivírus e
>  acredita-se estar livre de perigo.
>
>
> =
> Instru�ões para entrar na lista, sair da lista e usar a lista em
> http://www.mat.puc-rio.br/~obmlistas/obm-l.html
> =
>

-- 
Esta mensagem foi verificada pelo sistema de antiv�rus e
 acredita-se estar livre de perigo.



[obm-l] Dúvida

2019-12-04 Por tôpico Pedro José
Boa noite!
Estou dando uma repassada nas cônicas para auxiliar um filho de um amigo.
Dúvidas quanto à cônicas.
Alguns trabalhos até de mestrandos apontam a circunferência como sendo uma
elipse, um caso particular.
Aprendera que o limite de uma elipse quando a distância entre os focos
tendesse para zero era uma circunferência, não obstante a circunferência
não é uma elipse.
A elipse tem dois focos. O que não ocorre na circunferência.
A elipse pode ser definida como o lugar geométrico do plano em que a razão
entre a distância de um ponto ao foco direito e a distância entre esse
ponto e uma reta (diretriz direita) é constante e menor que 1 e igual a
excentricidade da cônica.
A circunferência não suporta tal definição.
Vejo muitos autores chamarem cônicas suaves.Significa que se escrevermos
uma equação quadrática com F(x,y)=0 a função F(x,y) é suave?

Grato!

Saudações,
PJMS.

-- 
Esta mensagem foi verificada pelo sistema de antiv�rus e
 acredita-se estar livre de perigo.



Re: [obm-l] Quadrados perfeitos

2019-11-28 Por tôpico Pedro José
Boa dia!
Para 11n +10^10 ser um quadrado perfeito
se faz necessário que seja da forma
(10^5+a)^2 com a > 0; pois,  n>=1 e a <= [raiz(12)-1*10^5] onde[x]= parts
inteira de x; pois,
(10^5+a)^2 <=11*10^10+10^10
10^5+a <=raiz(12)*10^5
a <= (raiz(12)-1)*10^5
Temos também que 11| 2*10^5a +a^2; pois, (10^5+a)^2=
10^10+2*10^5*a+a^2=10^10 +11*n
10^5=-1mod11 então:
-2a +a^2=0 mod11; a(a-2)=0 mod11.
Como 11 é primo a=2 ou a=0 mod11.
Agora é só contar quantos temos.
n11=[[(raiz(12)-1)*10^5]/11]=22.400
n2=[([(raiz(12)-1)*10^5]-2)/11)]=22400
Nt=44.800
Saudações,
PJMS

Em qua, 27 de nov de 2019 20:36, Esdras Muniz 
escreveu:

> Percebi agora que tô errado. Desculpa.
>
> Em qua, 27 de nov de 2019 19:22, Esdras Muniz 
> escreveu:
>
>> Pensei assim, o 10^10= (10^5)^2 é qp, daí, (10^5+1)^2, (10^5+2)^2, ...,
>> [Sqrt{12×10^5}] são só quadrados que queremos contar.
>>
>> Estou usando [x] para demorar a parte interna de x.
>>
>> Em qua, 27 de nov de 2019 15:30, Caio Costa 
>> escreveu:
>>
>>> 10^5([sqrt{2}]-1) ??
>>>
>>>
>>> Em qua., 27 de nov. de 2019 às 13:41, Esdras Muniz <
>>> esdrasmunizm...@gmail.com> escreveu:
>>>
 10^5([sqrt{12}]-1)

 Em qua, 27 de nov de 2019 08:57, marcone augusto araújo borges <
 marconeborge...@hotmail.com> escreveu:

> Seja n E N tal que 1 < = n < = 10^10. Quantos  números M = 11n +
> 10^10  são quadrados perfeitos?
> --
> Esta mensagem foi verificada pelo sistema de antivírus e
> acredita-se estar livre de perigo.
>

 --
 Esta mensagem foi verificada pelo sistema de antivírus e
 acredita-se estar livre de perigo.
>>>
>>>
>>> --
>>> Esta mensagem foi verificada pelo sistema de antivírus e
>>> acredita-se estar livre de perigo.
>>
>>
> --
> Esta mensagem foi verificada pelo sistema de antivírus e
> acredita-se estar livre de perigo

-- 
Esta mensagem foi verificada pelo sistema de antiv�rus e
 acredita-se estar livre de perigo.



[obm-l] Re: [obm-l] Re: [obm-l] Triângulos.

2019-11-24 Por tôpico Pedro José
Boa tarde!
Esdras,
Não seria z>=3.
3, 2, 2 dá um obtusângulo.

Saudações,
PJMS

Em sáb, 23 de nov de 2019 01:52, Esdras Muniz 
escreveu:

> Acho que a questão pressupõe que os lados devem ser inteiros. Daí se os
> lados são x, y e z, com x<=y x^2+y^2x^2+y^2 e
> z Daí, z é ao menos 4, vc sai contando caso a caso...
>
> Em sex, 22 de nov de 2019 20:39, Claudio Buffara <
> claudio.buff...@gmail.com> escreveu:
>
>> Do jeito que está escrito, uma infinidade.
>>
>> Enviado do meu iPhone
>>
>> > Em 22 de nov de 2019, à(s) 19:18, Guilherme Abbehusen <
>> gui.abbehuse...@gmail.com> escreveu:
>> >
>> > 
>> > Olá,Â
>> >   Preciso de ajuda com a seguinte questão:Â
>> >
>> > Tendo em vista a leis dos Cossenos, marque a quantidade de triângulos
>> obtusângulos que podemos formar com lados menores do que 7.
>> > a) 6
>> > b) 7
>> > c) 8Â
>> > d) 9
>> > e) 10
>> >
>> > --
>> > Esta mensagem foi verificada pelo sistema de antivírus e
>> > acredita-se estar livre de perigo.
>>
>> --
>> Esta mensagem foi verificada pelo sistema de antivírus e
>>  acredita-se estar livre de perigo.
>>
>>
>> =
>> Instru�ões para entrar na lista, sair da lista e usar a lista em
>> http://www.mat.puc-rio.br/~obmlistas/obm-l.html
>> =
>>
>
> --
> Esta mensagem foi verificada pelo sistema de antivírus e
> acredita-se estar livre de perigo.

-- 
Esta mensagem foi verificada pelo sistema de antiv�rus e
 acredita-se estar livre de perigo.



Re: [obm-l]

2019-11-16 Por tôpico Pedro José
Boa tarde!
Curioso, a solução (2,2) sai para q =0 no segundo caso 3q+2.
Todavia, falta mostrar que para os côngruos de 3 mod81, embora 6q^2+8q+3
dívida 81, não é uma potência de 3, já vi que ficou capenga.
Saudações,
PJMS

Em sáb, 16 de nov de 2019 14:54, Pedro José  escreveu:

> Boa tarde!
> O Esdras conseguiu para a e b par.
> Creio ter conseguido para a e b ímpares.
> Já havia encontrado (1,1) é (5,11)além  de (2,2) para se b pares.
> Vamos atrás dos peixes maiores.
> 3^a=2*(3q+c)^2+1, 0= c=1 ou c=2.
> Para c=1.
> 3^a=18q^2+12q+3
> 3^(a-1)=6q^2+4q+1
> Note que a solução  (1,1) acontece para q =0.
> E novamente uma restrição q=2 mod3pois já encontramos a soluçao para q =0.
> Mas como não havia soluções menores que as citadas com a e b ímpares,
> 3^a>81.
> Então 81 |6q^2+4q+1 Para algum resíduo de{5, 8 , 11...77,80}, o que não
> acontece.
> Para c =2
> 3^a =2(3q^2+2)^2+1
> 3^(a-1)=6q^2+8q+3 E temos nova restrição q=0 mod3.
> Observe que a solução (5,11) vem de q=3.
> Usando o mesmo raciocínio anterior,
> 81 | 6q^2+8q+3 para algum resíduo de {6,9,12..75 78}
> O que não acontece.
> Então juntando essa restrição braçal com a refinada do Esdras só existem
> as soluções que mencionara lá atrás.: (1,1); (2,2) e (5,11).
> Alguém poderia verificar se está correto?
> Saudações,
> PJMS
>
>

-- 
Esta mensagem foi verificada pelo sistema de antiv�rus e
 acredita-se estar livre de perigo.



Re: [obm-l]

2019-11-16 Por tôpico Pedro José
Boa tarde!
O Esdras conseguiu para a e b par.
Creio ter conseguido para a e b ímpares.
Já havia encontrado (1,1) é (5,11)além  de (2,2) para se b pares.
Vamos atrás dos peixes maiores.
3^a=2*(3q+c)^2+1, 0=81.
Então 81 |6q^2+4q+1 Para algum resíduo de{5, 8 , 11...77,80}, o que não
acontece.
Para c =2
3^a =2(3q^2+2)^2+1
3^(a-1)=6q^2+8q+3 E temos nova restrição q=0 mod3.
Observe que a solução (5,11) vem de q=3.
Usando o mesmo raciocínio anterior,
81 | 6q^2+8q+3 para algum resíduo de {6,9,12..75 78}
O que não acontece.
Então juntando essa restrição braçal com a refinada do Esdras só existem as
soluções que mencionara lá atrás.: (1,1); (2,2) e (5,11).
Alguém poderia verificar se está correto?
Saudações,
PJMS

-- 
Esta mensagem foi verificada pelo sistema de antiv�rus e
 acredita-se estar livre de perigo.



Re: [obm-l]

2019-11-15 Por tôpico Pedro José
Boa tarde!
Esdras,
Boa sacada!
(b^2+1)^2=b^4+2b^2+1=b^4+(3^k)^2.
Depois ternos pitagóricos sem restrição de primitivo.
Aí subtraindo a primeira da segunda ou somando dão quadrados perfeitos em p
e q. Basta igualar a1 ou então tira a raiz e iguala u^2 - v^2. Sai que
p-q=1.
Aí fica fácil.
Parabéns!
Falta achar uma lei de geração para outras soluções ou uma restrição
(acredito mais nessa) para a e b ímpares.
Saudações,
PJMS

Em sex, 15 de nov de 2019 13:05, Pedro José  escreveu:

> Bom dia!
> Esdras,
> grato, vou tentar seguir a linha.
>
> Douglas,
> Tentei combinar mod 8 com mod9 e não saiu uma restrição.
>
> Carlos Gustavo,
> teria como propor material sobre o tema que você levantou. Compreendi a
> fatoração, mas não como seriam os primos nesse universo.
> Ainda sem tempo para tentar uma restrição.
>
> Saudações,
> PJMS
>
>
> Em ter., 12 de nov. de 2019 às 23:21, Esdras Muniz <
> esdrasmunizm...@gmail.com> escreveu:
>
>> O caso "a" par eu fiz assim: a=2k, daí, (3^k)^2+ b^4=(d^2+1)^2, então vc
>> usa que para algum par p, q, com 0> p^2-q^2. Daí vc mostra que p=q+1 e em seguida que q=1.
>>
>> Em ter, 12 de nov de 2019 22:29, Prof. Douglas Oliveira <
>> profdouglaso.del...@gmail.com> escreveu:
>>
>>> Será que não sai usando somente congruência módulo 8?
>>>
>>> Em ter., 12 de nov. de 2019 às 20:07, Pedro José 
>>> escreveu:
>>>
>>>> Boa noite!
>>>> Esdras,
>>>> tem como você postar, mesmo para o caso apenas de n par?
>>>>
>>>> Grato!
>>>>
>>>> Saudações,
>>>> PJMS.
>>>>
>>>> Em ter., 12 de nov. de 2019 às 19:52, Pedro José 
>>>> escreveu:
>>>>
>>>>> Boa noite!
>>>>> Carlos Gustavo,
>>>>> grato pela luz, estava tão obsecado e só rodando em círculos, tal qual
>>>>> patrulha perdida.
>>>>>
>>>>> Saudações,
>>>>> PJMS
>>>>>
>>>>> Em ter., 12 de nov. de 2019 às 19:19, Esdras Muniz <
>>>>> esdrasmunizm...@gmail.com> escreveu:
>>>>>
>>>>>> Dá para mostrar que a única solução com a e b pares é (2, 2). Agora
>>>>>> com a e b ímpares, não consegui.
>>>>>>
>>>>>> Em ter, 12 de nov de 2019 18:19, Pedro José 
>>>>>> escreveu:
>>>>>>
>>>>>>> Boa noite!
>>>>>>> Agora captei vosso pensamento.
>>>>>>> Só que ao transformar a equação em uma equação de Pell, nós
>>>>>>> maculamos a função 3^n.
>>>>>>> Em verdade a solução para a par a= 2n, seria (2,2); pois, como
>>>>>>> mencionara anteriormente se a é par, b também o é.
>>>>>>> Só que quando procuramos as outras soluções, baseando-se na
>>>>>>> propriedade de que a norma em Q [RAiz(A)] conserva a multiplicação. Só 
>>>>>>> que
>>>>>>> quando eu pego a solução
>>>>>>> 3 + 2 Raiz(2) e elevo ao quadrado 17 + 12 Raiz(2). Se eu pegar
>>>>>>> 17^2-2*12^2=1 eu atendo x^2 - 2Y^2=1. E assim sucessivamente. Mas não
>>>>>>> existe n inteiro tal que 3^n=17, então não é uma solução da equação
>>>>>>> original.
>>>>>>> Creio que seja um pouco mais complicada a solução. Pois o difícil é
>>>>>>> saber quando atende também a 3^n.
>>>>>>> Acredito que deva haver uma forma de restringir a essas soluções,
>>>>>>> pois, definir em que condições a solução terá x como uma potência de 3 
>>>>>>> seja
>>>>>>> bem difícil.
>>>>>>> Estou apanhando mais do que mala velha em véspera de viagem.
>>>>>>> Se alguém postar uma solução, me ajudaria bastante.
>>>>>>>
>>>>>>> Saudações,
>>>>>>> PJMS
>>>>>>>
>>>>>>>
>>>>>>> Saudações,
>>>>>>> PJMS.
>>>>>>>
>>>>>>>
>>>>>>>
>>>>>>> Em ter., 12 de nov. de 2019 às 17:25, Pedro José <
>>>>>>> petroc...@gmail.com> escreveu:
>>>>>>>
>>>>>>>> Boa tarde!
>>>>>>>> Douglas,
>>>>>>>> perdoe-me pela minha miopia, mas você poderia detalhar melhor onde
>>>>>>>> entra a equ

Re: [obm-l]

2019-11-15 Por tôpico Pedro José
Bom dia!
Esdras,
grato, vou tentar seguir a linha.

Douglas,
Tentei combinar mod 8 com mod9 e não saiu uma restrição.

Carlos Gustavo,
teria como propor material sobre o tema que você levantou. Compreendi a
fatoração, mas não como seriam os primos nesse universo.
Ainda sem tempo para tentar uma restrição.

Saudações,
PJMS


Em ter., 12 de nov. de 2019 às 23:21, Esdras Muniz <
esdrasmunizm...@gmail.com> escreveu:

> O caso "a" par eu fiz assim: a=2k, daí, (3^k)^2+ b^4=(d^2+1)^2, então vc
> usa que para algum par p, q, com 0 p^2-q^2. Daí vc mostra que p=q+1 e em seguida que q=1.
>
> Em ter, 12 de nov de 2019 22:29, Prof. Douglas Oliveira <
> profdouglaso.del...@gmail.com> escreveu:
>
>> Será que não sai usando somente congruência módulo 8?
>>
>> Em ter., 12 de nov. de 2019 às 20:07, Pedro José 
>> escreveu:
>>
>>> Boa noite!
>>> Esdras,
>>> tem como você postar, mesmo para o caso apenas de n par?
>>>
>>> Grato!
>>>
>>> Saudações,
>>> PJMS.
>>>
>>> Em ter., 12 de nov. de 2019 às 19:52, Pedro José 
>>> escreveu:
>>>
>>>> Boa noite!
>>>> Carlos Gustavo,
>>>> grato pela luz, estava tão obsecado e só rodando em círculos, tal qual
>>>> patrulha perdida.
>>>>
>>>> Saudações,
>>>> PJMS
>>>>
>>>> Em ter., 12 de nov. de 2019 às 19:19, Esdras Muniz <
>>>> esdrasmunizm...@gmail.com> escreveu:
>>>>
>>>>> Dá para mostrar que a única solução com a e b pares é (2, 2). Agora
>>>>> com a e b ímpares, não consegui.
>>>>>
>>>>> Em ter, 12 de nov de 2019 18:19, Pedro José 
>>>>> escreveu:
>>>>>
>>>>>> Boa noite!
>>>>>> Agora captei vosso pensamento.
>>>>>> Só que ao transformar a equação em uma equação de Pell, nós maculamos
>>>>>> a função 3^n.
>>>>>> Em verdade a solução para a par a= 2n, seria (2,2); pois, como
>>>>>> mencionara anteriormente se a é par, b também o é.
>>>>>> Só que quando procuramos as outras soluções, baseando-se na
>>>>>> propriedade de que a norma em Q [RAiz(A)] conserva a multiplicação. Só 
>>>>>> que
>>>>>> quando eu pego a solução
>>>>>> 3 + 2 Raiz(2) e elevo ao quadrado 17 + 12 Raiz(2). Se eu pegar
>>>>>> 17^2-2*12^2=1 eu atendo x^2 - 2Y^2=1. E assim sucessivamente. Mas não
>>>>>> existe n inteiro tal que 3^n=17, então não é uma solução da equação
>>>>>> original.
>>>>>> Creio que seja um pouco mais complicada a solução. Pois o difícil é
>>>>>> saber quando atende também a 3^n.
>>>>>> Acredito que deva haver uma forma de restringir a essas soluções,
>>>>>> pois, definir em que condições a solução terá x como uma potência de 3 
>>>>>> seja
>>>>>> bem difícil.
>>>>>> Estou apanhando mais do que mala velha em véspera de viagem.
>>>>>> Se alguém postar uma solução, me ajudaria bastante.
>>>>>>
>>>>>> Saudações,
>>>>>> PJMS
>>>>>>
>>>>>>
>>>>>> Saudações,
>>>>>> PJMS.
>>>>>>
>>>>>>
>>>>>>
>>>>>> Em ter., 12 de nov. de 2019 às 17:25, Pedro José 
>>>>>> escreveu:
>>>>>>
>>>>>>> Boa tarde!
>>>>>>> Douglas,
>>>>>>> perdoe-me pela minha miopia, mas você poderia detalhar melhor onde
>>>>>>> entra a equação de Pell?
>>>>>>> A equação de Pell não é x^2-Dy^2 = N?
>>>>>>> Se a é par b é par e se a ímpar b é ímpar para atender mod8,
>>>>>>> Não consegui captar a sugestão.
>>>>>>>
>>>>>>> Saudações,
>>>>>>> PJMS
>>>>>>>
>>>>>>> Em ter., 12 de nov. de 2019 às 16:50, Prof. Douglas Oliveira <
>>>>>>> profdouglaso.del...@gmail.com> escreveu:
>>>>>>>
>>>>>>>> Hum, então, vamos analisar o caso de a ser par do tipo 2n.
>>>>>>>>
>>>>>>>> Assim podemos escrever que (3^n+b(sqrt2))(3^n-b(sqrt2))=1
>>>>>>>> Dai através da solução mínima que o Pedro fez, como (1,1) por
>>>>>>>> exemplo, da pra ver que são infinitas soluções usando a equação de 
>>>>>>>> Pell.
>>>>>>>>
>>>>>>>> Abraco
>>>>>>>> Douglas Oliveira.
>>>>>>>>
>>>>>>>>
>>>>>>>>
>>>>>>>> Em dom, 10 de nov de 2019 19:33, gilberto azevedo <
>>>>>>>> gil159...@gmail.com> escreveu:
>>>>>>>>
>>>>>>>>> [HELP]
>>>>>>>>>
>>>>>>>>> Achas todos os pares (a,b) inteiros positivos tais que :
>>>>>>>>> 3^a = 2b² + 1.
>>>>>>>>>
>>>>>>>>>
>>>>>>>>> --
>>>>>>>>> Esta mensagem foi verificada pelo sistema de antivírus e
>>>>>>>>> acredita-se estar livre de perigo.
>>>>>>>>
>>>>>>>>
>>>>>>>> --
>>>>>>>> Esta mensagem foi verificada pelo sistema de antivírus e
>>>>>>>> acredita-se estar livre de perigo.
>>>>>>>
>>>>>>>
>>>>>> --
>>>>>> Esta mensagem foi verificada pelo sistema de antivírus e
>>>>>> acredita-se estar livre de perigo.
>>>>>
>>>>>
>>>>> --
>>>>> Esta mensagem foi verificada pelo sistema de antivírus e
>>>>> acredita-se estar livre de perigo.
>>>>
>>>>
>>> --
>>> Esta mensagem foi verificada pelo sistema de antivírus e
>>> acredita-se estar livre de perigo.
>>
>>
>> --
>> Esta mensagem foi verificada pelo sistema de antivírus e
>> acredita-se estar livre de perigo.
>
>
> --
> Esta mensagem foi verificada pelo sistema de antivírus e
> acredita-se estar livre de perigo.

-- 
Esta mensagem foi verificada pelo sistema de antiv�rus e
 acredita-se estar livre de perigo.



[obm-l] Re: [obm-l] Re: [obm-l] Re: [obm-l] Re: [obm-l] Re: [obm-l] ordem lexicográfica dos numerais

2019-11-12 Por tôpico Pedro José
É curioso, pois, no problema que você postou com letras às vinha depois de
t.

Saudações,
PJMS

Em ter, 12 de nov de 2019 21:22, lumpa lumpa <1vp4l...@gmail.com> escreveu:

>
>
>
>
>
>
>
>
> Boa noite !
>
> Não. 01 vem depois de 00 que é o sucessor de 0, assim:
>
> 0, 00, 01, 02, 03, 04, 05, 06, 07, 08, 09, 1, 10, 11, 12, 13, 14, 15, 16,
> 17, 18, 19, 2, 20, 21, ... etc.
>
> É óbvio que a sequencia acima mostra apenas as combinações de no máximo
> dois algarismos, mas sabemos que há outros infinitos termos entre eles.
> Mostrando as combinações de três algarismos, seria assim:
>
> 0, 00, 000, 001, 002, 003, 004, 00, 006, 007, 008, 009, 01, 010, 011, 012,
> 013, 014, 01, 016, 017, 018, 019, 02, ., 03, 04, ,,..., 09,
> 090, 091, 092, 093, 094, 095, 096, 097, 098, 099, 1
>
>
>
>
>
>
>
>
>
>
>
>
>
>
>
>
>
>
>
> Em ter., 12 de nov. de 2019 às 19:49, Pedro José 
> escreveu:
>
>> Boa noite!
>> Mas 1 ocorre antes de 01, não. Tenho que esgotar primeiro as de uma
>> posição, para depois usar as de duas se não eu não andaria nunca. 0, 00,
>> 000,  
>> Só confirme que penso uma solução, caso consiga.
>>
>> Saudações,
>> PJMS
>>
>> Em ter., 12 de nov. de 2019 às 18:15, lumpa lumpa <1vp4l...@gmail.com>
>> escreveu:
>>
>>> Boa tarde, Pedro.
>>>
>>> Por menor posição, estou considerando aquela que conta a partir de zero
>>> todas as sequencias de no máximo quatro algarismos até  2019.
>>>
>>> 0, 00, 000,  são todos sequencias diferentes. Pense nos algarismos
>>> como símbolos quaisquer, como se fossem letras e as combinações palavras na
>>> ordem alfabética
>>>
>>> [0], 00, 01, 02, 03, 04, 05, 06, 07, 08, 09, [1], 10, 11, 12, 13, 14,
>>> 15, 16, 17, 18, 19, [2], 20, 21, 22, 23 etc
>>>
>>> A linha acima é só um pequeno exemplo dos primeiros termos da sequencia
>>> com termos de no máximo dois algarismos..
>>>
>>> Em ter., 12 de nov. de 2019 às 17:45, Pedro José 
>>> escreveu:
>>>
>>>> Boa tarde!
>>>> Vai depender do conceito!
>>>> 0 e 00 são contados como um só ou duas vezes?
>>>> Não entendi a menor posição. No meu entender há uma bijeção entre a
>>>> posição e o número.
>>>> A menos, que se contem 02019 e 2019 como o mesmo número, porém como
>>>> "palavras diferentes.
>>>>
>>>> Saudações,
>>>> PJMS
>>>>
>>>>
>>>> Em ter., 12 de nov. de 2019 às 15:31, lumpa lumpa <1vp4l...@gmail.com>
>>>> escreveu:
>>>>
>>>>> Qual a menor posição do número 2019 na ordem lexicográfica de todas as
>>>>> sequências possíveis dos algarismos indo-arábicos: {0, 1, 2, 3, 4, 5, 6, 
>>>>> 7,
>>>>> 8, 9} ?
>>>>>
>>>>> --
>>>>> Esta mensagem foi verificada pelo sistema de antivírus e
>>>>> acredita-se estar livre de perigo.
>>>>
>>>>
>>>> --
>>>> Esta mensagem foi verificada pelo sistema de antivírus e
>>>> acredita-se estar livre de perigo.
>>>
>>>
>>> --
>>> Esta mensagem foi verificada pelo sistema de antivírus e
>>> acredita-se estar livre de perigo.
>>
>>
>> --
>> Esta mensagem foi verificada pelo sistema de antivírus e
>> acredita-se estar livre de perigo.
>
>
> --
> Esta mensagem foi verificada pelo sistema de antivírus e
> acredita-se estar livre de perigo.

-- 
Esta mensagem foi verificada pelo sistema de antiv�rus e
 acredita-se estar livre de perigo.



Re: [obm-l]

2019-11-12 Por tôpico Pedro José
Boa noite!
Esdras,
tem como você postar, mesmo para o caso apenas de n par?

Grato!

Saudações,
PJMS.

Em ter., 12 de nov. de 2019 às 19:52, Pedro José 
escreveu:

> Boa noite!
> Carlos Gustavo,
> grato pela luz, estava tão obsecado e só rodando em círculos, tal qual
> patrulha perdida.
>
> Saudações,
> PJMS
>
> Em ter., 12 de nov. de 2019 às 19:19, Esdras Muniz <
> esdrasmunizm...@gmail.com> escreveu:
>
>> Dá para mostrar que a única solução com a e b pares é (2, 2). Agora com a
>> e b ímpares, não consegui.
>>
>> Em ter, 12 de nov de 2019 18:19, Pedro José 
>> escreveu:
>>
>>> Boa noite!
>>> Agora captei vosso pensamento.
>>> Só que ao transformar a equação em uma equação de Pell, nós maculamos a
>>> função 3^n.
>>> Em verdade a solução para a par a= 2n, seria (2,2); pois, como
>>> mencionara anteriormente se a é par, b também o é.
>>> Só que quando procuramos as outras soluções, baseando-se na propriedade
>>> de que a norma em Q [RAiz(A)] conserva a multiplicação. Só que quando eu
>>> pego a solução
>>> 3 + 2 Raiz(2) e elevo ao quadrado 17 + 12 Raiz(2). Se eu pegar
>>> 17^2-2*12^2=1 eu atendo x^2 - 2Y^2=1. E assim sucessivamente. Mas não
>>> existe n inteiro tal que 3^n=17, então não é uma solução da equação
>>> original.
>>> Creio que seja um pouco mais complicada a solução. Pois o difícil é
>>> saber quando atende também a 3^n.
>>> Acredito que deva haver uma forma de restringir a essas soluções, pois,
>>> definir em que condições a solução terá x como uma potência de 3 seja bem
>>> difícil.
>>> Estou apanhando mais do que mala velha em véspera de viagem.
>>> Se alguém postar uma solução, me ajudaria bastante.
>>>
>>> Saudações,
>>> PJMS
>>>
>>>
>>> Saudações,
>>> PJMS.
>>>
>>>
>>>
>>> Em ter., 12 de nov. de 2019 às 17:25, Pedro José 
>>> escreveu:
>>>
>>>> Boa tarde!
>>>> Douglas,
>>>> perdoe-me pela minha miopia, mas você poderia detalhar melhor onde
>>>> entra a equação de Pell?
>>>> A equação de Pell não é x^2-Dy^2 = N?
>>>> Se a é par b é par e se a ímpar b é ímpar para atender mod8,
>>>> Não consegui captar a sugestão.
>>>>
>>>> Saudações,
>>>> PJMS
>>>>
>>>> Em ter., 12 de nov. de 2019 às 16:50, Prof. Douglas Oliveira <
>>>> profdouglaso.del...@gmail.com> escreveu:
>>>>
>>>>> Hum, então, vamos analisar o caso de a ser par do tipo 2n.
>>>>>
>>>>> Assim podemos escrever que (3^n+b(sqrt2))(3^n-b(sqrt2))=1
>>>>> Dai através da solução mínima que o Pedro fez, como (1,1) por exemplo,
>>>>> da pra ver que são infinitas soluções usando a equação de Pell.
>>>>>
>>>>> Abraco
>>>>> Douglas Oliveira.
>>>>>
>>>>>
>>>>>
>>>>> Em dom, 10 de nov de 2019 19:33, gilberto azevedo 
>>>>> escreveu:
>>>>>
>>>>>> [HELP]
>>>>>>
>>>>>> Achas todos os pares (a,b) inteiros positivos tais que :
>>>>>> 3^a = 2b² + 1.
>>>>>>
>>>>>>
>>>>>> --
>>>>>> Esta mensagem foi verificada pelo sistema de antivírus e
>>>>>> acredita-se estar livre de perigo.
>>>>>
>>>>>
>>>>> --
>>>>> Esta mensagem foi verificada pelo sistema de antivírus e
>>>>> acredita-se estar livre de perigo.
>>>>
>>>>
>>> --
>>> Esta mensagem foi verificada pelo sistema de antivírus e
>>> acredita-se estar livre de perigo.
>>
>>
>> --
>> Esta mensagem foi verificada pelo sistema de antivírus e
>> acredita-se estar livre de perigo.
>
>

-- 
Esta mensagem foi verificada pelo sistema de antiv�rus e
 acredita-se estar livre de perigo.



Re: [obm-l]

2019-11-12 Por tôpico Pedro José
Boa noite!
Carlos Gustavo,
grato pela luz, estava tão obsecado e só rodando em círculos, tal qual
patrulha perdida.

Saudações,
PJMS

Em ter., 12 de nov. de 2019 às 19:19, Esdras Muniz <
esdrasmunizm...@gmail.com> escreveu:

> Dá para mostrar que a única solução com a e b pares é (2, 2). Agora com a
> e b ímpares, não consegui.
>
> Em ter, 12 de nov de 2019 18:19, Pedro José 
> escreveu:
>
>> Boa noite!
>> Agora captei vosso pensamento.
>> Só que ao transformar a equação em uma equação de Pell, nós maculamos a
>> função 3^n.
>> Em verdade a solução para a par a= 2n, seria (2,2); pois, como mencionara
>> anteriormente se a é par, b também o é.
>> Só que quando procuramos as outras soluções, baseando-se na propriedade
>> de que a norma em Q [RAiz(A)] conserva a multiplicação. Só que quando eu
>> pego a solução
>> 3 + 2 Raiz(2) e elevo ao quadrado 17 + 12 Raiz(2). Se eu pegar
>> 17^2-2*12^2=1 eu atendo x^2 - 2Y^2=1. E assim sucessivamente. Mas não
>> existe n inteiro tal que 3^n=17, então não é uma solução da equação
>> original.
>> Creio que seja um pouco mais complicada a solução. Pois o difícil é saber
>> quando atende também a 3^n.
>> Acredito que deva haver uma forma de restringir a essas soluções, pois,
>> definir em que condições a solução terá x como uma potência de 3 seja bem
>> difícil.
>> Estou apanhando mais do que mala velha em véspera de viagem.
>> Se alguém postar uma solução, me ajudaria bastante.
>>
>> Saudações,
>> PJMS
>>
>>
>> Saudações,
>> PJMS.
>>
>>
>>
>> Em ter., 12 de nov. de 2019 às 17:25, Pedro José 
>> escreveu:
>>
>>> Boa tarde!
>>> Douglas,
>>> perdoe-me pela minha miopia, mas você poderia detalhar melhor onde entra
>>> a equação de Pell?
>>> A equação de Pell não é x^2-Dy^2 = N?
>>> Se a é par b é par e se a ímpar b é ímpar para atender mod8,
>>> Não consegui captar a sugestão.
>>>
>>> Saudações,
>>> PJMS
>>>
>>> Em ter., 12 de nov. de 2019 às 16:50, Prof. Douglas Oliveira <
>>> profdouglaso.del...@gmail.com> escreveu:
>>>
>>>> Hum, então, vamos analisar o caso de a ser par do tipo 2n.
>>>>
>>>> Assim podemos escrever que (3^n+b(sqrt2))(3^n-b(sqrt2))=1
>>>> Dai através da solução mínima que o Pedro fez, como (1,1) por exemplo,
>>>> da pra ver que são infinitas soluções usando a equação de Pell.
>>>>
>>>> Abraco
>>>> Douglas Oliveira.
>>>>
>>>>
>>>>
>>>> Em dom, 10 de nov de 2019 19:33, gilberto azevedo 
>>>> escreveu:
>>>>
>>>>> [HELP]
>>>>>
>>>>> Achas todos os pares (a,b) inteiros positivos tais que :
>>>>> 3^a = 2b² + 1.
>>>>>
>>>>>
>>>>> --
>>>>> Esta mensagem foi verificada pelo sistema de antivírus e
>>>>> acredita-se estar livre de perigo.
>>>>
>>>>
>>>> --
>>>> Esta mensagem foi verificada pelo sistema de antivírus e
>>>> acredita-se estar livre de perigo.
>>>
>>>
>> --
>> Esta mensagem foi verificada pelo sistema de antivírus e
>> acredita-se estar livre de perigo.
>
>
> --
> Esta mensagem foi verificada pelo sistema de antivírus e
> acredita-se estar livre de perigo.

-- 
Esta mensagem foi verificada pelo sistema de antiv�rus e
 acredita-se estar livre de perigo.



[obm-l] Re: [obm-l] Re: [obm-l] Re: [obm-l] ordem lexicográfica dos numerais

2019-11-12 Por tôpico Pedro José
Boa noite!

Usa os algarismos {0,1,2,3,4,5,6,7,8,9,A} Onde o algarismo A representa o
número 10
Pode usar o mesmo algoritmo que já mencionara. Só que agora na base 10.

1o Passo transformar o número para que só tenha algarismos significativos,
evitar zero a esquerda.
2019 --> 312A
2o Passo substitui os algarismos pelo seus antecessores.
2019
Note que nesse caso ficamos com uma identidade, então podemos esquecer
esses passos e o conjunto anterior.

1o passo somar (1...1) com tantos algarismos quanto forem os do número
incluindo os algarismos zero a esquerda.
2019+= 3130 e já está na base 10 não precisa fazer mais nada.

Ou poderia sair por contagem.
0 1 2 3 4 5 6 7 8 9 00 01 01 

Se não houvessem os algarismos destacados em amarelo e nem os com zero mais
a esquerda a ordem seria 2019.
Então basta somar quantos números temos com o zero a esquerda.
Com 1 algarismo 0 apenas 1 10^(1-1)
com dois algarismos 00; 01; 02..08; 09. 10 algarismos 10^(2-1)
com três algarismos 10^2
com 4 algarismos e menor que 2019
todos os números que tenham pelo menos um zero no algarismo mais a esquerda
e 4 algarismos < 2019
logo 0 X X X , com 10 opções para cada X, basta multiplicar 10^3
A posição é 2019 +1 +10 +100+1000= 3130

Porém, o algoritmo se compreendido é bem melhor, pois, se você pegar, e.g.,
0387
Você não precisa se preocupar com a posição de onde aparece ,
para depois contar quantos há até 0387.
Basta 0387+=1498

Fica bem mais fácil.

Saudações,
PJMS








Em ter., 12 de nov. de 2019 às 18:39, Pedro José 
escreveu:

> Boa noite!
> Mas 1 ocorre antes de 01, não. Tenho que esgotar primeiro as de uma
> posição, para depois usar as de duas se não eu não andaria nunca. 0, 00,
> 000,  
> Só confirme que penso uma solução, caso consiga.
>
> Saudações,
> PJMS
>
> Em ter., 12 de nov. de 2019 às 18:15, lumpa lumpa <1vp4l...@gmail.com>
> escreveu:
>
>> Boa tarde, Pedro.
>>
>> Por menor posição, estou considerando aquela que conta a partir de zero
>> todas as sequencias de no máximo quatro algarismos até  2019.
>>
>> 0, 00, 000,  são todos sequencias diferentes. Pense nos algarismos
>> como símbolos quaisquer, como se fossem letras e as combinações palavras na
>> ordem alfabética
>>
>> [0], 00, 01, 02, 03, 04, 05, 06, 07, 08, 09, [1], 10, 11, 12, 13, 14, 15,
>> 16, 17, 18, 19, [2], 20, 21, 22, 23 etc
>>
>> A linha acima é só um pequeno exemplo dos primeiros termos da sequencia
>> com termos de no máximo dois algarismos..
>>
>> Em ter., 12 de nov. de 2019 às 17:45, Pedro José 
>> escreveu:
>>
>>> Boa tarde!
>>> Vai depender do conceito!
>>> 0 e 00 são contados como um só ou duas vezes?
>>> Não entendi a menor posição. No meu entender há uma bijeção entre a
>>> posição e o número.
>>> A menos, que se contem 02019 e 2019 como o mesmo número, porém como
>>> "palavras diferentes.
>>>
>>> Saudações,
>>> PJMS
>>>
>>>
>>> Em ter., 12 de nov. de 2019 às 15:31, lumpa lumpa <1vp4l...@gmail.com>
>>> escreveu:
>>>
>>>> Qual a menor posição do número 2019 na ordem lexicográfica de todas as
>>>> sequências possíveis dos algarismos indo-arábicos: {0, 1, 2, 3, 4, 5, 6, 7,
>>>> 8, 9} ?
>>>>
>>>> --
>>>> Esta mensagem foi verificada pelo sistema de antivírus e
>>>> acredita-se estar livre de perigo.
>>>
>>>
>>> --
>>> Esta mensagem foi verificada pelo sistema de antivírus e
>>> acredita-se estar livre de perigo.
>>
>>
>> --
>> Esta mensagem foi verificada pelo sistema de antivírus e
>> acredita-se estar livre de perigo.
>
>

-- 
Esta mensagem foi verificada pelo sistema de antiv�rus e
 acredita-se estar livre de perigo.



[obm-l] Re: [obm-l] Re: [obm-l] Re: [obm-l] ordem lexicográfica dos numerais

2019-11-12 Por tôpico Pedro José
Boa noite!
Mas 1 ocorre antes de 01, não. Tenho que esgotar primeiro as de uma
posição, para depois usar as de duas se não eu não andaria nunca. 0, 00,
000,  
Só confirme que penso uma solução, caso consiga.

Saudações,
PJMS

Em ter., 12 de nov. de 2019 às 18:15, lumpa lumpa <1vp4l...@gmail.com>
escreveu:

> Boa tarde, Pedro.
>
> Por menor posição, estou considerando aquela que conta a partir de zero
> todas as sequencias de no máximo quatro algarismos até  2019.
>
> 0, 00, 000,  são todos sequencias diferentes. Pense nos algarismos
> como símbolos quaisquer, como se fossem letras e as combinações palavras na
> ordem alfabética
>
> [0], 00, 01, 02, 03, 04, 05, 06, 07, 08, 09, [1], 10, 11, 12, 13, 14, 15,
> 16, 17, 18, 19, [2], 20, 21, 22, 23 etc
>
> A linha acima é só um pequeno exemplo dos primeiros termos da sequencia
> com termos de no máximo dois algarismos..
>
> Em ter., 12 de nov. de 2019 às 17:45, Pedro José 
> escreveu:
>
>> Boa tarde!
>> Vai depender do conceito!
>> 0 e 00 são contados como um só ou duas vezes?
>> Não entendi a menor posição. No meu entender há uma bijeção entre a
>> posição e o número.
>> A menos, que se contem 02019 e 2019 como o mesmo número, porém como
>> "palavras diferentes.
>>
>> Saudações,
>> PJMS
>>
>>
>> Em ter., 12 de nov. de 2019 às 15:31, lumpa lumpa <1vp4l...@gmail.com>
>> escreveu:
>>
>>> Qual a menor posição do número 2019 na ordem lexicográfica de todas as
>>> sequências possíveis dos algarismos indo-arábicos: {0, 1, 2, 3, 4, 5, 6, 7,
>>> 8, 9} ?
>>>
>>> --
>>> Esta mensagem foi verificada pelo sistema de antivírus e
>>> acredita-se estar livre de perigo.
>>
>>
>> --
>> Esta mensagem foi verificada pelo sistema de antivírus e
>> acredita-se estar livre de perigo.
>
>
> --
> Esta mensagem foi verificada pelo sistema de antivírus e
> acredita-se estar livre de perigo.

-- 
Esta mensagem foi verificada pelo sistema de antiv�rus e
 acredita-se estar livre de perigo.



Re: [obm-l]

2019-11-12 Por tôpico Pedro José
Boa noite!
Agora captei vosso pensamento.
Só que ao transformar a equação em uma equação de Pell, nós maculamos a
função 3^n.
Em verdade a solução para a par a= 2n, seria (2,2); pois, como mencionara
anteriormente se a é par, b também o é.
Só que quando procuramos as outras soluções, baseando-se na propriedade de
que a norma em Q [RAiz(A)] conserva a multiplicação. Só que quando eu pego
a solução
3 + 2 Raiz(2) e elevo ao quadrado 17 + 12 Raiz(2). Se eu pegar
17^2-2*12^2=1 eu atendo x^2 - 2Y^2=1. E assim sucessivamente. Mas não
existe n inteiro tal que 3^n=17, então não é uma solução da equação
original.
Creio que seja um pouco mais complicada a solução. Pois o difícil é saber
quando atende também a 3^n.
Acredito que deva haver uma forma de restringir a essas soluções, pois,
definir em que condições a solução terá x como uma potência de 3 seja bem
difícil.
Estou apanhando mais do que mala velha em véspera de viagem.
Se alguém postar uma solução, me ajudaria bastante.

Saudações,
PJMS


Saudações,
PJMS.



Em ter., 12 de nov. de 2019 às 17:25, Pedro José 
escreveu:

> Boa tarde!
> Douglas,
> perdoe-me pela minha miopia, mas você poderia detalhar melhor onde entra a
> equação de Pell?
> A equação de Pell não é x^2-Dy^2 = N?
> Se a é par b é par e se a ímpar b é ímpar para atender mod8,
> Não consegui captar a sugestão.
>
> Saudações,
> PJMS
>
> Em ter., 12 de nov. de 2019 às 16:50, Prof. Douglas Oliveira <
> profdouglaso.del...@gmail.com> escreveu:
>
>> Hum, então, vamos analisar o caso de a ser par do tipo 2n.
>>
>> Assim podemos escrever que (3^n+b(sqrt2))(3^n-b(sqrt2))=1
>> Dai através da solução mínima que o Pedro fez, como (1,1) por exemplo, da
>> pra ver que são infinitas soluções usando a equação de Pell.
>>
>> Abraco
>> Douglas Oliveira.
>>
>>
>>
>> Em dom, 10 de nov de 2019 19:33, gilberto azevedo 
>> escreveu:
>>
>>> [HELP]
>>>
>>> Achas todos os pares (a,b) inteiros positivos tais que :
>>> 3^a = 2b² + 1.
>>>
>>>
>>> --
>>> Esta mensagem foi verificada pelo sistema de antivírus e
>>> acredita-se estar livre de perigo.
>>
>>
>> --
>> Esta mensagem foi verificada pelo sistema de antivírus e
>> acredita-se estar livre de perigo.
>
>

-- 
Esta mensagem foi verificada pelo sistema de antiv�rus e
 acredita-se estar livre de perigo.



Re: [obm-l]

2019-11-12 Por tôpico Pedro José
Boa tarde!
Douglas,
perdoe-me pela minha miopia, mas você poderia detalhar melhor onde entra a
equação de Pell?
A equação de Pell não é x^2-Dy^2 = N?
Se a é par b é par e se a ímpar b é ímpar para atender mod8,
Não consegui captar a sugestão.

Saudações,
PJMS

Em ter., 12 de nov. de 2019 às 16:50, Prof. Douglas Oliveira <
profdouglaso.del...@gmail.com> escreveu:

> Hum, então, vamos analisar o caso de a ser par do tipo 2n.
>
> Assim podemos escrever que (3^n+b(sqrt2))(3^n-b(sqrt2))=1
> Dai através da solução mínima que o Pedro fez, como (1,1) por exemplo, da
> pra ver que são infinitas soluções usando a equação de Pell.
>
> Abraco
> Douglas Oliveira.
>
>
>
> Em dom, 10 de nov de 2019 19:33, gilberto azevedo 
> escreveu:
>
>> [HELP]
>>
>> Achas todos os pares (a,b) inteiros positivos tais que :
>> 3^a = 2b² + 1.
>>
>>
>> --
>> Esta mensagem foi verificada pelo sistema de antivírus e
>> acredita-se estar livre de perigo.
>
>
> --
> Esta mensagem foi verificada pelo sistema de antivírus e
> acredita-se estar livre de perigo.

-- 
Esta mensagem foi verificada pelo sistema de antiv�rus e
 acredita-se estar livre de perigo.



[obm-l] Re: [obm-l] ordem lexicográfica dos numerais

2019-11-12 Por tôpico Pedro José
Boa tarde!
Vai depender do conceito!
0 e 00 são contados como um só ou duas vezes?
Não entendi a menor posição. No meu entender há uma bijeção entre a posição
e o número.
A menos, que se contem 02019 e 2019 como o mesmo número, porém como
"palavras diferentes.

Saudações,
PJMS


Em ter., 12 de nov. de 2019 às 15:31, lumpa lumpa <1vp4l...@gmail.com>
escreveu:

> Qual a menor posição do número 2019 na ordem lexicográfica de todas as
> sequências possíveis dos algarismos indo-arábicos: {0, 1, 2, 3, 4, 5, 6, 7,
> 8, 9} ?
>
> --
> Esta mensagem foi verificada pelo sistema de antivírus e
> acredita-se estar livre de perigo.

-- 
Esta mensagem foi verificada pelo sistema de antiv�rus e
 acredita-se estar livre de perigo.



Re: [obm-l]

2019-11-12 Por tôpico Pedro José
Bom dia!
Não consegui restringir a essas.
1)a=b=1
2)a=b=2
3)a=5 e b=11.

Em dom, 10 de nov de 2019 20:33, gilberto azevedo 
escreveu:

> [HELP]
>
> Achas todos os pares (a,b) inteiros positivos tais que :
> 3^a = 2b² + 1.
>
>
> --
> Esta mensagem foi verificada pelo sistema de antivírus e
> acredita-se estar livre de perigo.

-- 
Esta mensagem foi verificada pelo sistema de antiv�rus e
 acredita-se estar livre de perigo.



[obm-l] Re: [obm-l] Re: [obm-l] Re: [obm-l] Re: [obm-l] Ordem Lexicográfica

2019-11-11 Por tôpico Pedro José
Boa tarde!

Errata
3o passo  (4.052.405.310)base6+ (1.111.111.111)base6= (5.203.520.421)base6
e não 3o passo  (4.052.405.31)base6+ (1.111.111.111)base6=
(5.203.520.421)base6

Então você agora, pode tanto compor uma palavra de ordem n, quanto
descobrir qual ordem de uma dada palavra e não. Então você agora, pode
tanto compor uma palavra de ordem n, quanto descobrir qual a palavra de
ordem n.

Saudações,
PJMS

Em seg., 11 de nov. de 2019 às 11:51, Pedro José 
escreveu:

> Bom dia!
>
> É muita coincidência. Teve um problema agora a respeito de numeração na
> terra do IMPA que é muito, mas muito semelhante a esse.
> Só que nesse caso caso é o contrário, ou seja a função inversa. O da terra
> dos Impa, dá uma posição e quer saber qual o número IMPA.
> Aqui se dá uma palavra e se quer a posição.
> Podemos fazer uma bijeção: {a,e,c,i,m,t} em {1,2,3,4,5,6}  se x
> f(x) Usando-se o conceito que já mencionara no problema referido.
> Para saber, no caso a palavra de3 ordem n.
> Passo 1. Escrever o número n na base 6.
> Passo 2. Subtrair na base 6 (n)base 6 - (...111) base6 o maior número
> somente com algarismos 1 e menor que n. Nota considerar da esquerda para
> direita, tantos algarismos quanto o do número .,
> Passo 3 Substituir cada algarismo do resultado da conta anterior por esse
> algarismo acrescido de 1.
> Passo 4. Usar a função inversa da bijeção e tornar os algarismos em letras.
>
> Por exemplo vamos pegar a palavra de ordem imediatamente inferior a
> MATEMATICA, como a resposta foi dada 53.929.309.
> A palavra seria de ordem: 53.929.308
> 1o passo. 53.929.30= (5203520420)base6
> 2o passo (5.203.520.420)base6 - (1.111.111.111)base6= (4.052.405.305)
> 3o passo (5.163.516416)
> 4o passo MATEMATIAT, que realmente é a palavra imediatamente anterior a
> MATEMATICA.
> Então já temos como ir da ordem para a palavra. Agora é só determinar a
> inversa.
> 1o Passo Usar a bijeção e transformar a palavra em Número
> 2o Passo Substituir cada algarismo por o algarismo que o antecede.
> 3o passo Somar (.111)Base6 com tantos algarismos obtidos no passo
> anterior ao obtido no passo anterior, considerando-o como base 6.
> 4o passar transformar o valor obtido da base 6 para pase 10.
>
> Aplicando para MATEMATICA
>
> 1o passo: 5.163.516.421
> 2opasso 4.052.405.310
> 3o passo  (4.052.405.31)base6+ (1.111.111.111)base6= (5.203.520.421)base6
> 4o passo 5*6^9+2*6^8+3*6^6+5*6^5+2*6^4+4*6^2+2*6+1 =53.929.309
>
> Se você quiser entender o porquê, veja nota Minha solução para o item c)
> do problema 3 da prova da OBM-2017 lá eu faço uma abordagem, pegando
> carona na ideia do Cauã, e aqui uma adaptação para a base 6.
>
> Então você agora, pode tanto compor uma palavra de ordem n, quanto
> descobrir qual a palavra de ordem n.
>
> Deixo claro, que me apoiei na ideia do Cauã. A ideia de levar para a base
> 5 no caso dos Impa foi dele. Infelizmente ele não deu um prosseguimento
> satisfatório.
>
> Mas a ideia foi muito boa e serviu até para esse novo problema. Uma grata
> coincidência. Pois a abordagem anterior foi, digamos, um pouco vilipendiada.
> Talvez agora, com mais uma aplicação receba mais atenção e méritos ao Cauã.
>
> Saudações,
> PJMS
>
>
>
>
>
> Em dom., 10 de nov. de 2019 às 21:22, lumpa lumpa <1vp4l...@gmail.com>
> escreveu:
>
>> Boa solução, Rodrigo, mas pq sua contagem resultou um a menos ?
>> A resposta é 53.929.309
>>
>> On Sun, Nov 10, 2019 at 9:31 PM Rodrigo Ângelo 
>> wrote:
>>
>>> A primeira palavra com 10 letras tem posição δ = 6 + 6^2 + ... + 6^9 =
>>> 12.093.234.
>>>
>>> Das palavras que tem 10 letras, fixando a primeira letra em "a", temos
>>> 6^9 palavras. Depois, fixando a primeira letra em "c", "e", e "i", temos a
>>> mesma quantidade.
>>>
>>> Então, das palavras de 10 letras, a primeira que começa com "m" tem
>>> posição
>>> δ + 4*6^9
>>>
>>> Repetindo esse processo para todas as letras que formam a palavra
>>> desejada, temos:
>>>
>>> δ + 4*6^9 + 5*6^7 + 2*6^6 + 4*6^5 + 5*6^3 + 3*6^2 + 6 = 53.929.308.
>>>
>>> Atenciosamente,
>>> Rodrigo Angelo
>>>
>>> On Sun, Nov 10, 2019, 18:42 jamil silva  wrote:
>>>
>>>> Será que sai por base seis ?
>>>>
>>>> Em dom., 10 de nov. de 2019 às 18:31, lumpa lumpa <1vp4l...@gmail.com>
>>>> escreveu:
>>>>
>>>>> Qual a posição da palavra "matematica" na série de todas as sequências
>>>>> possíveis em ordem alfabética das letras do conjunto da palavra matemática
>>>>> {a, 

[obm-l] Re: [obm-l] Re: [obm-l] Re: [obm-l] Re: [obm-l] Ordem Lexicográfica

2019-11-11 Por tôpico Pedro José
Bom dia!

É muita coincidência. Teve um problema agora a respeito de numeração na
terra do IMPA que é muito, mas muito semelhante a esse.
Só que nesse caso caso é o contrário, ou seja a função inversa. O da terra
dos Impa, dá uma posição e quer saber qual o número IMPA.
Aqui se dá uma palavra e se quer a posição.
Podemos fazer uma bijeção: {a,e,c,i,m,t} em {1,2,3,4,5,6}  se x
f(x)
escreveu:

> Boa solução, Rodrigo, mas pq sua contagem resultou um a menos ?
> A resposta é 53.929.309
>
> On Sun, Nov 10, 2019 at 9:31 PM Rodrigo Ângelo 
> wrote:
>
>> A primeira palavra com 10 letras tem posição δ = 6 + 6^2 + ... + 6^9 =
>> 12.093.234.
>>
>> Das palavras que tem 10 letras, fixando a primeira letra em "a", temos
>> 6^9 palavras. Depois, fixando a primeira letra em "c", "e", e "i", temos a
>> mesma quantidade.
>>
>> Então, das palavras de 10 letras, a primeira que começa com "m" tem
>> posição
>> δ + 4*6^9
>>
>> Repetindo esse processo para todas as letras que formam a palavra
>> desejada, temos:
>>
>> δ + 4*6^9 + 5*6^7 + 2*6^6 + 4*6^5 + 5*6^3 + 3*6^2 + 6 = 53.929.308.
>>
>> Atenciosamente,
>> Rodrigo Angelo
>>
>> On Sun, Nov 10, 2019, 18:42 jamil silva  wrote:
>>
>>> Será que sai por base seis ?
>>>
>>> Em dom., 10 de nov. de 2019 às 18:31, lumpa lumpa <1vp4l...@gmail.com>
>>> escreveu:
>>>
 Qual a posição da palavra "matematica" na série de todas as sequências
 possíveis em ordem alfabética das letras do conjunto da palavra matemática
 {a, c, e, i, m, t} ?
 essa sequencia inicia assim:
 a, c, e, i, m, t, aa, ac, ae, ai, am, at, ca, cc, ce, ci, cm, ct, ea,
 ec, ee, ei, em, et, ia, ic, ie, ii, im ,it, ma, mc, me, mi, mm, mt, ta, tc,
 te, ti, tm, tt, aaa, aac, aae, ...

 --
 Esta mensagem foi verificada pelo sistema de antivírus e
 acredita-se estar livre de perigo.
>>>
>>>
>>> --
>>> Esta mensagem foi verificada pelo sistema de antivírus e
>>> acredita-se estar livre de perigo.
>>
>>
>> --
>> Esta mensagem foi verificada pelo sistema de antivírus e
>> acredita-se estar livre de perigo.
>
>
> --
> Esta mensagem foi verificada pelo sistema de antivírus e
> acredita-se estar livre de perigo.

-- 
Esta mensagem foi verificada pelo sistema de antiv�rus e
 acredita-se estar livre de perigo.



[obm-l] Re: [obm-l] Re: [obm-l] Minha solução para o item c) do problema 3 da prova da OBM-2017

2019-11-08 Por tôpico Pedro José
Bom dia!
Cláudio,
peço máxima vênia e venho discordar de você.
Se você pegar a soma da PG: 1, 5, 25 , que é a quebra do número de
algarismos.
Ou seja a partir de S1=1, temos pelo menos um algarismo no impa
A partir de S2=6 temos pelo menos dois algarismos no impa
A partir de S3= 31 temos pelo menos três algarismos no impa
A partir de Sn=(5^n-1)/4 temos pelo menos n algarismo no impa

Há um padrão de repetição dos algarismos no sistema impa.
AnAn-1...A3A2A1Ao
Quando na posição mais a direita de menor ordem, posição Ao
13579135791357119
Portanto para definir o ´algarismo Ao. Nota y=[x], significa parte inteira.
Basta pegar ko= n-S1 e fazer xo=ko mod 5 e pegar o resíduo em {0,1,2,3,4}
se Ao= 2xo+1.
Para a posição de A1 a partir de S2=6 temos:
13579 13579...
Basta pegar k1= [(n-s2)/5] e fazer x1=k1 mod5 e aplicar A1=2x1+1

Para um algarismo de ordem i
Basta pegar ki=[(n-Si+1)/5^i] e fazer xi= k1 mod5 e Ai=2xi+1.

Mas Si+1 = (1,1,1,...1,1) base 5 com i+1 elementos 1.
então Zi = (n)base 5 - (1,1,1...111)= (BnBn-1...Bi+1BiBi-1...A3A2A1Ao
ki=[Zi/5î] = Bn*5^(n-i)+ Bn-a*5^(n-i-1)+ +Bi+1*5+ Bi
Fora o Bi as demais parcelas obrigatoriamente são côngruas de zero mod5.
Então xi=ki mod5=Bi
Aí fica muito fáciL;
Passo 1, define n na base 5
Passo 2 Subtrai o maior número na base 5 formado só por ...1 e menor
que n.
Passo 3 Substitui os algarismos encontrados por Ai=2xi+1, até termos tantos
algarismos (da direita para esquerda) quanto os do número que subtraímos
...111, inclusive os zeros a esquerda.
E temos o número.
E.g., Como representar 9839962
1o Passo 9839962= (1004334322) base 5
2o passo 1004334322-1 base 5 =3343223211
3o passo 7797557533 impa.

Gastei 5 min com conferência.


Para fazer esse número com contagem, fica complicado,

Por exemplo para o número 2017
31032
31032-1=14421
39953.

Em 1min e 20 s.

Acho que a sugestão de Cauã foi muito boa.


Saudações,
PJMS


Em sex., 8 de nov. de 2019 às 09:08, Claudio Buffara <
claudio.buff...@gmail.com> escreveu:

> Uma forma de ver se sua solução está certa é tentar dar outra solução,
> essencialmente diferente da primeira.
>
> A meu ver, a solução mais elementar é por enumeração pura e simples e usa
> apenas o princípio multiplicativo, sem nenhuma "sacada brilhante".
>
> Há 5 números de um algarismo no sistema Impa (1, 3, 5, 7, 9);
> 5^2 = 25 números de dois algarismos;
> 5^3 = 125 de três;
> 5^4 = 625 de quatro.
> Isso significa que , o maior número de quatro algarismos no sistema
> Impa, ocupa o lugar de número 5+25+125+625 = 780 na sequência e, portanto,
> corresponde ao número 780 no sistema decimal.
>
> 5^5 = 3125 > 2017, o que implica que 2017 corresponde a um número Impa de
> cinco algarismos.
> Há 625 números Impa de 5 algarismos começando com 1 (de 1 a 1) ==>
> 1 corresponde a 780+625 = 1405
> Há 625 deles começando com 3 (de 3 a 3) ==> 3 corresponde a
> 1405+780 = 2185.
> Ou seja, o número desejado (correspondente a 2017) começa com "3".
>
> Voltando ao 1 (correspondente ao nosso 1405) ...
> O número seguinte é 3 (1406).
> Há:
> - 125 números começando com 31 (de 3 a 31999): 1405+125 = 1530;
> - 125 começando com 33 (33111 a 33999): 1530+125 = 1655
> - 125 começando com 35: 1655+125 = 1780
> - 125 começando com 37: 1780+125 = 1905
> - 125 começando com 39: 1905+125 = 2030 ==> o número desejado começa com
> "39".
>
> Tomemos, então, o Impa 37999 (correspondente a 1905). O seguinte é o 39111.
> Há:
> - 25 números começando com 391 (39111 a 39199): 1905+25 = 1930
> - 25 começando com 393: 1930+25 = 1955
> - 25 começando com 395: 1955+25 = 1980
> - 25 começando com 397: 1980+25 = 2005 ==> 2005 corresponde ao Impa 39799
> ==> 2006 é 39911
>
> E, por fim, há:
> - 5 números começando com 3991: 2005+5 = 2010
> - 5 começando com 3993: 2010+5 = 2015 ==> 2015 corresponde ao Impa 39939
> ==> 2016 é 39951 ==> 2017 é 39953.
>
> []s,
> Claudio.
>
>
> On Thu, Nov 7, 2019 at 12:36 PM Cauã DSR  wrote:
>
>>
>> Tenho um pequeno problema, eu fiz o item C) do problema 3 da prova da OBM
>> de 2017, mas não tenho certeza sobre seu resultado, então achei uma boa
>> fazer minha primeira aparição no grupo perguntando se o que fiz está certo.
>>
>> 3. Na Terra dos Impas, somente os algarismos ímpares são utilizados para
>> contar e escrever números. Assim, em vez dos numeros 1, 2, 3, 4, 5, 6, 7,
>> 8, 9, 10, 11, 12,. . . os Impas tem os números correspondentes 1, 3, 5, 7,
>> 9, 11, 13, 15, 17, 19, 31, 33, . . . (note que os números dos Impas tem
>> somente algarismos ímpares). Por exemplo, se
>> uma criança tem 11 anos, os Impas diriam que ela tem 31 anos.
>>
>> c) Escreva, na linguagem dos Impas, o numero que na nossa representação
>> decimal é escrito como 2017.
>>
>> Minha solução:
>> Como no problema só temos Ímpares para usar como algarismo {1,3,5,7,9},
>> temos um sistema de numeração de base 5, porém com os algarismos ímpares ao
>> invés da base 5 comumente usada 

[obm-l] Re: [obm-l] Re: [obm-l] Re: [obm-l] Re: [obm-l] Minha solução para o item c) do problema 3 da prova da OBM-2017

2019-11-08 Por tôpico Pedro José
Bom dia!
Creio que não.
Por exemplo, 11 na base 6 é 15.
Daria 39, do jeito que você propôs. Mas dá 31.
Fiz a transformação de 2017 de várias formas e deu sempre 39953.
Alguém tem a resposta?

Saudações, PJMS


Em sex, 8 de nov de 2019 06:58, Esdras Muniz 
escreveu:

> Acho que é só passar 2017 para a base 6 e depois substituir os algarismos
> 0, 1, 2, 3, 4, 5 por 1, 3, 5, 7, 9 respectivamente.
>
> Assim, 2017 na base 6 é 13201, trocando os algarismos, fica: 37513.
>
> Em qui, 7 de nov de 2019 22:16, Cauã DSR 
> escreveu:
>
>> Muito obrigado! É realmente uma honra ler isso.
>> Sobre a questão eu ficarei de analisá-la (principalmente algumas funções
>> que não entendi ainda) no sábado, se possível
>>
>> Em qui, 7 de nov de 2019 9:27 PM, Pedro José 
>> escreveu:
>>
>>> Boa noite!
>>>
>>> Pode-se usar a soma da PG de razão 5 e o primeir termo 1
>>>
>>> então, no sistema impa, teremos 5 números com 1 algarismo, 30 números
>>> com 1ou 2 algarismos, 155 números com até 3 algarismos, 780 números com até
>>> 4 algarismos e Sn=(5^n-1)/4 números com até n algarismos.
>>>
>>> Os algarismos de ordem mais baixa tem um padrão 1 3 5 7 9 1 3 5 7 9...
>>> depois 1 3 5 7 9, depois ...1(5^2vezes)  3..33
>>> .55
>>> 77.777  e assim sucessivamente.
>>>
>>> Do algarismo menos significativo para o mais.
>>> Até 5 só há um algarismo.
>>> De 6= S2 em diante teremos pelo menos dois algarismos
>>> De  31= S3 em diante teremos pelo menos dois algarismos.
>>> De 156 = S4 em diante teremos pelo menos três algarismos
>>>
>>> De Sn+1 teremos pelo menos n algarismos.
>>> Podemos achar os algarismos xn xn-1 ...x2 x1  para um número na base
>>> decimal assim
>>> se ai= 0 xi=1, se ai =1 xi=3; se ai=3 x1= 7 e se ai=4 xi=9 para i <=n
>>> k1=Int (y-S1) e a1= mod (k1;5)
>>> k2=int((y-S2)/5) e a2= mod(k2;5)
>>>
>>> kn=int((y-Sn)/ 5^(n-1)) e an = mod(kn;5)
>>>
>>> Para o número em questão: 2017
>>> k1 = 2016 e a1=1  então x1=3
>>> k2=int((2017-6)/5)=402; a2=2 então x2=5
>>> k3=int((2017-31)/25)=79. a3=4 então x3=9
>>> k4=int((2017-156)/125)=14; a4=4 e x4=9
>>> k5=int((2017-781)/625)=1; a5=1 e x5=3
>>>
>>> Não há mais algarismos pois 2017 <3906=S6. Portanto a representação é:
>>> 39953.
>>>
>>> Porém você, Cauã DSR ,deu uma ideia muito legal.
>>>
>>> Estou querendo provar duas coisas, que não consegui, mas estou certo que
>>> acontece.
>>>
>>>
>>> Se o número em decimal passado para base 5 não tiver algarismos zero,
>>> você pode simplesmente.
>>> 1 permanece 1 na impa
>>> 2 vira 3 na impa
>>> 3 vira 5 na impa
>>> 4 vira 7 na impa.
>>>
>>> Caso você tenha um número com algarismo zero quando transformado para a
>>> base 5,e.g., y= (x6x5x40x2x1)base5
>>> Você pode,sendo o o indicado o menos significativo, Impa (y)=
>>> Concat(impa(x6x5x40);impa(x2x1)) oNde concat é a concatenação.
>>>
>>> Assim para o nosso número original 2017= (31032) base5
>>> impa (310)base 5
>>> (310)base5=80
>>> k1=79; a1=4 e x1=9
>>> k2=int((80-6)/5=14 ;a2=4 e x2=9
>>> k3=int(80-31)/25=1 a3= 1 e x3 = 3
>>>
>>> então impa (310)base5= 399
>>> impa(32)= 53, faz direto pois não tem nenhum algarismo zero.
>>> Então impa (31032)=Concat (impa(310);(impa(32))= 39953; como achado
>>> acima.
>>>
>>> É uma sacada legal. Pois; se não tem algarismo zero na base 5 sai direto.
>>>
>>> Caso haja você quebra o número o da direita sai direto. E na esquerda
>>> você trabalha com um número menor.
>>> Depois é só concatenar.
>>> Só não consegui provar ainda. Sua ideia foi muito boa.
>>>
>>> Parabéns,
>>> PJMS.
>>>
>>>
>>>
>>>
>>>
>>>
>>>
>>>
>>>
>>>
>>>
>>>
>>> Em qui., 7 de nov. de 2019 às 17:27, Pedro José 
>>> escreveu:
>>>
>>>> Boa tarde!
>>>> Você seguiu uma linha de argumentação interessante.
>>>> Mas não está correto.
>>>> Pois existem 5 números com 1 algarismo 5^2 números com 2 algarismos,
>>>> 5^3 com 3 e assim sucessivamente.
>>>> Usando a soma da PG
>>>> 6-11
>>>> 31 -111
>>>> 156 -
>>>> 781- 1
>>>> Assim o maior nú

[obm-l] Re: [obm-l] Minha solução para o item c) do problema 3 da prova da OBM-2017

2019-11-07 Por tôpico Pedro José
Boa noite!

Pode-se usar a soma da PG de razão 5 e o primeir termo 1

então, no sistema impa, teremos 5 números com 1 algarismo, 30 números com
1ou 2 algarismos, 155 números com até 3 algarismos, 780 números com até 4
algarismos e Sn=(5^n-1)/4 números com até n algarismos.

Os algarismos de ordem mais baixa tem um padrão 1 3 5 7 9 1 3 5 7 9...
depois 1 3 5 7 9, depois ...1(5^2vezes)  3..33
.55
77.777  e assim sucessivamente.

Do algarismo menos significativo para o mais.
Até 5 só há um algarismo.
De 6= S2 em diante teremos pelo menos dois algarismos
De  31= S3 em diante teremos pelo menos dois algarismos.
De 156 = S4 em diante teremos pelo menos três algarismos

De Sn+1 teremos pelo menos n algarismos.
Podemos achar os algarismos xn xn-1 ...x2 x1  para um número na base
decimal assim
se ai= 0 xi=1, se ai =1 xi=3; se ai=3 x1= 7 e se ai=4 xi=9 para i <=n
k1=Int (y-S1) e a1= mod (k1;5)
k2=int((y-S2)/5) e a2= mod(k2;5)

kn=int((y-Sn)/ 5^(n-1)) e an = mod(kn;5)

Para o número em questão: 2017
k1 = 2016 e a1=1  então x1=3
k2=int((2017-6)/5)=402; a2=2 então x2=5
k3=int((2017-31)/25)=79. a3=4 então x3=9
k4=int((2017-156)/125)=14; a4=4 e x4=9
k5=int((2017-781)/625)=1; a5=1 e x5=3

Não há mais algarismos pois 2017 <3906=S6. Portanto a representação é:
39953.

Porém você, Cauã DSR ,deu uma ideia muito legal.

Estou querendo provar duas coisas, que não consegui, mas estou certo que
acontece.


Se o número em decimal passado para base 5 não tiver algarismos zero, você
pode simplesmente.
1 permanece 1 na impa
2 vira 3 na impa
3 vira 5 na impa
4 vira 7 na impa.

Caso você tenha um número com algarismo zero quando transformado para a
base 5,e.g., y= (x6x5x40x2x1)base5
Você pode,sendo o o indicado o menos significativo, Impa (y)=
Concat(impa(x6x5x40);impa(x2x1)) oNde concat é a concatenação.

Assim para o nosso número original 2017= (31032) base5
impa (310)base 5
(310)base5=80
k1=79; a1=4 e x1=9
k2=int((80-6)/5=14 ;a2=4 e x2=9
k3=int(80-31)/25=1 a3= 1 e x3 = 3

então impa (310)base5= 399
impa(32)= 53, faz direto pois não tem nenhum algarismo zero.
Então impa (31032)=Concat (impa(310);(impa(32))= 39953; como achado acima.

É uma sacada legal. Pois; se não tem algarismo zero na base 5 sai direto.

Caso haja você quebra o número o da direita sai direto. E na esquerda você
trabalha com um número menor.
Depois é só concatenar.
Só não consegui provar ainda. Sua ideia foi muito boa.

Parabéns,
PJMS.












Em qui., 7 de nov. de 2019 às 17:27, Pedro José 
escreveu:

> Boa tarde!
> Você seguiu uma linha de argumentação interessante.
> Mas não está correto.
> Pois existem 5 números com 1 algarismo 5^2 números com 2 algarismos, 5^3
> com 3 e assim sucessivamente.
> Usando a soma da PG
> 6-11
> 31 -111
> 156 -
> 781- 1
> Assim o maior número de 4 algarismos  representaria 780.
> O número teria que ter 5 algarismos.
>
> Saudações,
> PJMS
>
>
>
> Em qui., 7 de nov. de 2019 às 12:36, Cauã DSR 
> escreveu:
>
>>
>> Tenho um pequeno problema, eu fiz o item C) do problema 3 da prova da OBM
>> de 2017, mas não tenho certeza sobre seu resultado, então achei uma boa
>> fazer minha primeira aparição no grupo perguntando se o que fiz está certo.
>>
>> 3. Na Terra dos Impas, somente os algarismos ímpares são utilizados para
>> contar e escrever números. Assim, em vez dos numeros 1, 2, 3, 4, 5, 6, 7,
>> 8, 9, 10, 11, 12,. . . os Impas tem os números correspondentes 1, 3, 5, 7,
>> 9, 11, 13, 15, 17, 19, 31, 33, . . . (note que os números dos Impas tem
>> somente algarismos ímpares). Por exemplo, se
>> uma criança tem 11 anos, os Impas diriam que ela tem 31 anos.
>>
>> c) Escreva, na linguagem dos Impas, o numero que na nossa representação
>> decimal é escrito como 2017.
>>
>> Minha solução:
>> Como no problema só temos Ímpares para usar como algarismo {1,3,5,7,9},
>> temos um sistema de numeração de base 5, porém com os algarismos ímpares ao
>> invés da base 5 comumente usada {0,1,2,3,4}. Ao analisar isso decidi
>> transformar 2017 em um número de Base 5 {1,2,3,4,5}, ao usar esta base,
>> percebi que para transformar um número de Base Decimal em um de Base 5
>> {1,2,3,4,5} é quase o mesmo processo para transformá-lo em um número de
>> Base 5 {0,1,2,3,4}, onde a única diferença é que podemos usar 5x5^n e que
>> quando tivermos 0x5^n apenas basta ignorá-lo e partir para a próxima
>> potência de 5 (5^n-1).
>> Ao fazer isto obtive o seguinte:
>>
>> 2017= 3x5^4+1x5^3+3x5^1+2x5^0
>> 2017= 3132
>>
>> Agora, saibam que tem como transformar um número n de base 5 {1,2,3,4,5}
>> em um número x de base 5 {1,3,5,7,9} apenas mudando os algarismos
>> correspondentes, uma vez que os dois tem base 5.
>> então temos os seguinte correspond

[obm-l] Re: [obm-l] Minha solução para o item c) do problema 3 da prova da OBM-2017

2019-11-07 Por tôpico Pedro José
Boa tarde!
Você seguiu uma linha de argumentação interessante.
Mas não está correto.
Pois existem 5 números com 1 algarismo 5^2 números com 2 algarismos, 5^3
com 3 e assim sucessivamente.
Usando a soma da PG
6-11
31 -111
156 -
781- 1
Assim o maior número de 4 algarismos  representaria 780.
O número teria que ter 5 algarismos.

Saudações,
PJMS



Em qui., 7 de nov. de 2019 às 12:36, Cauã DSR 
escreveu:

>
> Tenho um pequeno problema, eu fiz o item C) do problema 3 da prova da OBM
> de 2017, mas não tenho certeza sobre seu resultado, então achei uma boa
> fazer minha primeira aparição no grupo perguntando se o que fiz está certo.
>
> 3. Na Terra dos Impas, somente os algarismos ímpares são utilizados para
> contar e escrever números. Assim, em vez dos numeros 1, 2, 3, 4, 5, 6, 7,
> 8, 9, 10, 11, 12,. . . os Impas tem os números correspondentes 1, 3, 5, 7,
> 9, 11, 13, 15, 17, 19, 31, 33, . . . (note que os números dos Impas tem
> somente algarismos ímpares). Por exemplo, se
> uma criança tem 11 anos, os Impas diriam que ela tem 31 anos.
>
> c) Escreva, na linguagem dos Impas, o numero que na nossa representação
> decimal é escrito como 2017.
>
> Minha solução:
> Como no problema só temos Ímpares para usar como algarismo {1,3,5,7,9},
> temos um sistema de numeração de base 5, porém com os algarismos ímpares ao
> invés da base 5 comumente usada {0,1,2,3,4}. Ao analisar isso decidi
> transformar 2017 em um número de Base 5 {1,2,3,4,5}, ao usar esta base,
> percebi que para transformar um número de Base Decimal em um de Base 5
> {1,2,3,4,5} é quase o mesmo processo para transformá-lo em um número de
> Base 5 {0,1,2,3,4}, onde a única diferença é que podemos usar 5x5^n e que
> quando tivermos 0x5^n apenas basta ignorá-lo e partir para a próxima
> potência de 5 (5^n-1).
> Ao fazer isto obtive o seguinte:
>
> 2017= 3x5^4+1x5^3+3x5^1+2x5^0
> 2017= 3132
>
> Agora, saibam que tem como transformar um número n de base 5 {1,2,3,4,5}
> em um número x de base 5 {1,3,5,7,9} apenas mudando os algarismos
> correspondentes, uma vez que os dois tem base 5.
> então temos os seguinte correspondentes das Bases 5 {1,2,3,4,5} e
> {1,3,5,7,9} respectivamente
> 1=1
> 2=3
> 3=5
> 4=7
> 5=9
>
> Portanto o número 3132 da Base 5 {1,2,3,4,5} vira 5153 da Base 5
> {1,3,5,7,9}
>
> --
> Esta mensagem foi verificada pelo sistema de antivírus e
> acredita-se estar livre de perigo.

-- 
Esta mensagem foi verificada pelo sistema de antiv�rus e
 acredita-se estar livre de perigo.



[obm-l] Re: [obm-l] Congruência

2019-11-06 Por tôpico Pedro José
Boa tarde!

Daria para ter melhorado a procura para (10x+6)^3
10x(5x+8) = 20 mod125 ==> 10x(5x+8)=20 + 250*q ==> x(5x+8)=2 +25 q ==>
x(5x+8) = 2 mod 25
x(5x+8) tem que acabar em 2 ou em 7.
1 não
2 não
3 não
4 temos 28*4=112 não atende.
5 não
6 não
7 não
8 não
9 temos 477 = 2 mod25 OK!!!
10 não
11 não
12  não

Assim para 124 resíduos, só precisamos verificar 6 resíduos. 6, 21, 71,
121, 46 e 96. Diminuiu bastante a procura. Embora, deva haver uma forma
mais elegante.

Saudações,
PJMS




Em qua., 6 de nov. de 2019 às 11:32, Pedro José 
escreveu:

> Boa tarde!
> Esqueci o "vai um" que a tia Loló me ensinou: 346 e não 246.
>
> Desculpem-me,
> PJMS
>
> Em qua., 6 de nov. de 2019 às 10:59, Pedro José 
> escreveu:
>
>> Boa tarde!
>>
>> Só consegui na grosseria.
>>
>> Para que o resto por 125 acabe em 1, x deverá acabar em 1 ou em 6.
>>
>> 6^3=216 não atende
>>
>> (10x+1)^3 = 300x^2 + 30x +1 = 111 mod 125 ==> 300x^2 + 30x= 110 + 250 q,
>> com q pertencente a |N.
>> 30x^2+3x =11 +25q.
>>
>> Tem que ser um x entre [1,12] e o produto 3x (10x+1) tem que acabar com 1
>> ou 6.
>> Quem vai comandar é 3x, únicos candidatos 2, 7 e 12.
>> x=2 ==>126=1 mod25 não atende.
>> x=7 ==> 1491= 16 mod 25
>> x=12 ==> 4356 = 6 mod25
>>
>> Logo não há número terminado em 1 que atenda o proposto.
>>
>> (10x+6)^3= 1800x^2 + 1080x + 216 = 111 mod 125 ==> 10x(5x+8) = 20 mod125
>> aqui não encontrei restrição, tem de ir na marra.
>> 1 ==>130 = 20 mod 125, não atende
>> 2==> 360 = 20 mod 125 não
>> 3==> 690 = 20 mod 125 não
>> 4 ==> 1120 = 20 mod 125 não
>> 5 ==> 1650 = 20 mod 125 não
>> 6===> 2280 = 20 mod125 não
>> 7 ==> 3010 = 20 mod125 não
>> 8==> 3840 =20 mod125 não
>> 9==> 4770 = 20 mod125 OK!
>> 10 ==> 5800 = 20 mod125 não
>> 11 ==> 6930 = 20 mod 125 não
>> 12 ==> 8160 = 20 mod125 não.
>>
>> Dentre os resíduos apenas os côngruos de 96 atendem. Como são os
>> primeiros positivos.
>> (96, 96 + 125, 96+250)= (96, 221, 246).
>>
>> Deve haver alguma forma mais elegante.
>>
>> Saudações,
>> PJMS
>>
>>
>>
>> Em ter., 5 de nov. de 2019 às 23:30, marcone augusto araújo borges <
>> marconeborge...@hotmail.com> escreveu:
>>
>>> Determine os três menores inteiros positivos x, tais que x^3 = = 111
>>> (mod 5^3).
>>> Desde já agradeço
>>> --
>>> Esta mensagem foi verificada pelo sistema de antivírus e
>>> acredita-se estar livre de perigo.
>>>
>>

-- 
Esta mensagem foi verificada pelo sistema de antiv�rus e
 acredita-se estar livre de perigo.



[obm-l] Re: [obm-l] Congruência

2019-11-06 Por tôpico Pedro José
Boa tarde!
Esqueci o "vai um" que a tia Loló me ensinou: 346 e não 246.

Desculpem-me,
PJMS

Em qua., 6 de nov. de 2019 às 10:59, Pedro José 
escreveu:

> Boa tarde!
>
> Só consegui na grosseria.
>
> Para que o resto por 125 acabe em 1, x deverá acabar em 1 ou em 6.
>
> 6^3=216 não atende
>
> (10x+1)^3 = 300x^2 + 30x +1 = 111 mod 125 ==> 300x^2 + 30x= 110 + 250 q,
> com q pertencente a |N.
> 30x^2+3x =11 +25q.
>
> Tem que ser um x entre [1,12] e o produto 3x (10x+1) tem que acabar com 1
> ou 6.
> Quem vai comandar é 3x, únicos candidatos 2, 7 e 12.
> x=2 ==>126=1 mod25 não atende.
> x=7 ==> 1491= 16 mod 25
> x=12 ==> 4356 = 6 mod25
>
> Logo não há número terminado em 1 que atenda o proposto.
>
> (10x+6)^3= 1800x^2 + 1080x + 216 = 111 mod 125 ==> 10x(5x+8) = 20 mod125
> aqui não encontrei restrição, tem de ir na marra.
> 1 ==>130 = 20 mod 125, não atende
> 2==> 360 = 20 mod 125 não
> 3==> 690 = 20 mod 125 não
> 4 ==> 1120 = 20 mod 125 não
> 5 ==> 1650 = 20 mod 125 não
> 6===> 2280 = 20 mod125 não
> 7 ==> 3010 = 20 mod125 não
> 8==> 3840 =20 mod125 não
> 9==> 4770 = 20 mod125 OK!
> 10 ==> 5800 = 20 mod125 não
> 11 ==> 6930 = 20 mod 125 não
> 12 ==> 8160 = 20 mod125 não.
>
> Dentre os resíduos apenas os côngruos de 96 atendem. Como são os primeiros
> positivos.
> (96, 96 + 125, 96+250)= (96, 221, 246).
>
> Deve haver alguma forma mais elegante.
>
> Saudações,
> PJMS
>
>
>
> Em ter., 5 de nov. de 2019 às 23:30, marcone augusto araújo borges <
> marconeborge...@hotmail.com> escreveu:
>
>> Determine os três menores inteiros positivos x, tais que x^3 = = 111 (mod
>> 5^3).
>> Desde já agradeço
>> --
>> Esta mensagem foi verificada pelo sistema de antivírus e
>> acredita-se estar livre de perigo.
>>
>

-- 
Esta mensagem foi verificada pelo sistema de antiv�rus e
 acredita-se estar livre de perigo.



[obm-l] Re: [obm-l] Congruência

2019-11-06 Por tôpico Pedro José
Boa tarde!

Só consegui na grosseria.

Para que o resto por 125 acabe em 1, x deverá acabar em 1 ou em 6.

6^3=216 não atende

(10x+1)^3 = 300x^2 + 30x +1 = 111 mod 125 ==> 300x^2 + 30x= 110 + 250 q,
com q pertencente a |N.
30x^2+3x =11 +25q.

Tem que ser um x entre [1,12] e o produto 3x (10x+1) tem que acabar com 1
ou 6.
Quem vai comandar é 3x, únicos candidatos 2, 7 e 12.
x=2 ==>126=1 mod25 não atende.
x=7 ==> 1491= 16 mod 25
x=12 ==> 4356 = 6 mod25

Logo não há número terminado em 1 que atenda o proposto.

(10x+6)^3= 1800x^2 + 1080x + 216 = 111 mod 125 ==> 10x(5x+8) = 20 mod125
aqui não encontrei restrição, tem de ir na marra.
1 ==>130 = 20 mod 125, não atende
2==> 360 = 20 mod 125 não
3==> 690 = 20 mod 125 não
4 ==> 1120 = 20 mod 125 não
5 ==> 1650 = 20 mod 125 não
6===> 2280 = 20 mod125 não
7 ==> 3010 = 20 mod125 não
8==> 3840 =20 mod125 não
9==> 4770 = 20 mod125 OK!
10 ==> 5800 = 20 mod125 não
11 ==> 6930 = 20 mod 125 não
12 ==> 8160 = 20 mod125 não.

Dentre os resíduos apenas os côngruos de 96 atendem. Como são os primeiros
positivos.
(96, 96 + 125, 96+250)= (96, 221, 246).

Deve haver alguma forma mais elegante.

Saudações,
PJMS



Em ter., 5 de nov. de 2019 às 23:30, marcone augusto araújo borges <
marconeborge...@hotmail.com> escreveu:

> Determine os três menores inteiros positivos x, tais que x^3 = = 111 (mod
> 5^3).
> Desde já agradeço
> --
> Esta mensagem foi verificada pelo sistema de antivírus e
> acredita-se estar livre de perigo.
>

-- 
Esta mensagem foi verificada pelo sistema de antiv�rus e
 acredita-se estar livre de perigo.



[obm-l] Re: [obm-l] Re: [obm-l] Re: [obm-l] Re: [obm-l] Re: [obm-l] Máximo e Mínimo de uma Função

2019-11-03 Por tôpico Pedro José
Bom dia!
Eu coloquei só o resultado do cálculo.
Note que, para cada jogo de pontos, há três pontos. Os dois da extremidade
possuem sinais diversos na primeira derivada. Significa que entre eles a
derivada se anula porque é contínua.

Como o cos(x) apresenta picos de Pi/2 em Pi/2. Você pode fazer uma tabela
com pontos defasados em 0,5 e verificar quando ocorre uma variação de sinal
na derivada. Aí entre esses pontos tem um que anula a derivada.
Pode-se fazer um segmento de reta e ver onde se anula. Nesse ponto calcula
de novo o valor da derivada e dececide entre que pontos estão. Até que o
intervalo fique nem pequeno ou o  valor da derivada bem próximo de zero.
Mas pode usar pesquisa binária que é mais simples. Calcula a média dos "x"
e o valor da derivada na média.  Depois define entre que pontos se anula.

Saudações,
PJMS

Em sáb, 2 de nov de 2019 21:20, Luiz Antonio Rodrigues <
rodrigue...@gmail.com> escreveu:

> Olá, Pedro!
> Boa noite!
> Tudo bem?
> Muito obrigado pelas informações!
> Vou aguardar seus cálculos!
> Um abraço!
> Luiz
>
> On Sat, Nov 2, 2019, 6:02 PM Pedro José  wrote:
>
>> Boa tarde!
>>
>> Quando se fala em o máximo e o mínimo. Entendo como sendo globais, ou vão
>> acontecer nas extremidades ou em algum máximo e mínimo local, que também
>> será global.
>>
>> f(-12) = 0,453
>> f(-3) = -0,475
>>
>> Não se está pedindo qual o máximo ou mínimo. Se fosse isso dever-se-ia
>> usar algum método numérico.
>> Mas a função é contínua nesse intervalo, se ela for monótona esses seriam
>> o máximo e mínimo.
>>
>> Mas se não for ou o máximo e mínimo ocorrerá nas extremidades ou em algum
>> ponto de derivada zero. Mas está garantido que exista tanto máximo quanto
>> mínimo.
>>
>> Os intervalos, aos quais o 0 pertence também não tem pois a Lim f(x)
>> quando x --> 0+ não existe, tende a 00 e quando tende a )- também não
>> existe, tende a -oo.
>>
>> E o último intervalo certamente terá um máximo. Mas não um mínimo pois a
>> primeira parcela de f(x), é monótona decrescente e tende a zero quando
>> x-->oo e a segundo oscila periodicamente.
>>
>> Vamos supor que o mínimo ocorra em xo ==> 1/xo + sen(xo). Agora pegando o
>> menor valor de x1 com x1> xo e sen (x1) =1 temos que f(x1) = 1/x1 + sen(x1)
>>
>> onde 1/x1 <1/xo e sen(x1)<=sen(xo) ==> f(x1) < f(xo), absurdo. Não existe
>> mínimo.
>>
>> A resposta certa é a a)
>>
>> Vou tentar achá-los por método numérico e dou uma "garibada" na resposta.
>>
>> Saudações,
>> PJMS.
>>
>>
>>
>>
>>
>>
>>
>>
>>
>>
>> Em sáb., 2 de nov. de 2019 às 16:28, Luiz Antonio Rodrigues <
>> rodrigue...@gmail.com> escreveu:
>>
>>> Olá, Esdras!
>>> Olá, Rodrigo!
>>> Tudo bem?
>>> Muito obrigado pela ajuda!
>>> Sim, eu também pensei que a questão não tem solução...
>>> Vou começar a pensar que o problema pede intervalo, ou intervalos, nos
>>> quais existam mínimos ou máximos locais.
>>> Se for assim, acho que a saída é pensar nos intervalos onde o zero não
>>> está presente...
>>> Acho que meu erro foi pensar que em intervalos fechados existem, com
>>> certeza, máximos e mínimos locais...
>>> Vou continuar pensando e escrevo se tiver novidades.
>>> Abraços!
>>> Luiz
>>>
>>> On Sat, Nov 2, 2019, 2:55 PM Rodrigo Ângelo 
>>> wrote:
>>>
>>>> Luiz,
>>>>
>>>> Quando x tende a zero pela direita e pela esquerda, f tende a mais e
>>>> menos infinito, respetivamente.
>>>>
>>>> À rigor, a função não tem máximo nem mínimo, porque para todo x no
>>>> domínio da f, é possível encontrar um xmax e um xmin tais que f(xmax) >
>>>> f(x) e f(xmin) < f(x).
>>>>
>>>> Dito isso, eu responderia a alternativa c) por ser a única que contém o
>>>> zero.
>>>>
>>>> On Sat, Nov 2, 2019, 13:53 Luiz Antonio Rodrigues <
>>>> rodrigue...@gmail.com> wrote:
>>>>
>>>>> Olá, pessoal!
>>>>> Bom dia!
>>>>> Estou tentando resolver o seguinte problema:
>>>>>
>>>>> É dada a função:
>>>>>
>>>>> f(x)=(1/x)+sen(x)
>>>>>
>>>>> Pergunta-se:
>>>>>
>>>>> Em quais intervalos abaixo é garantido que encontremos o máximo e o
>>>>> mínimo desta função?
>>>>>
>>>>> a) [-12;-3]
>>>>> 

[obm-l] Re: [obm-l] Re: [obm-l] Re: [obm-l] Máximo e Mínimo de uma Função

2019-11-02 Por tôpico Pedro José
Boa tarde!

Quando se fala em o máximo e o mínimo. Entendo como sendo globais, ou vão
acontecer nas extremidades ou em algum máximo e mínimo local, que também
será global.

f(-12) = 0,453
f(-3) = -0,475

Não se está pedindo qual o máximo ou mínimo. Se fosse isso dever-se-ia usar
algum método numérico.
Mas a função é contínua nesse intervalo, se ela for monótona esses seriam o
máximo e mínimo.

Mas se não for ou o máximo e mínimo ocorrerá nas extremidades ou em algum
ponto de derivada zero. Mas está garantido que exista tanto máximo quanto
mínimo.

Os intervalos, aos quais o 0 pertence também não tem pois a Lim f(x) quando
x --> 0+ não existe, tende a 00 e quando tende a )- também não existe,
tende a -oo.

E o último intervalo certamente terá um máximo. Mas não um mínimo pois a
primeira parcela de f(x), é monótona decrescente e tende a zero quando
x-->oo e a segundo oscila periodicamente.

Vamos supor que o mínimo ocorra em xo ==> 1/xo + sen(xo). Agora pegando o
menor valor de x1 com x1> xo e sen (x1) =1 temos que f(x1) = 1/x1 + sen(x1)

onde 1/x1 <1/xo e sen(x1)<=sen(xo) ==> f(x1) < f(xo), absurdo. Não existe
mínimo.

A resposta certa é a a)

Vou tentar achá-los por método numérico e dou uma "garibada" na resposta.

Saudações,
PJMS.










Em sáb., 2 de nov. de 2019 às 16:28, Luiz Antonio Rodrigues <
rodrigue...@gmail.com> escreveu:

> Olá, Esdras!
> Olá, Rodrigo!
> Tudo bem?
> Muito obrigado pela ajuda!
> Sim, eu também pensei que a questão não tem solução...
> Vou começar a pensar que o problema pede intervalo, ou intervalos, nos
> quais existam mínimos ou máximos locais.
> Se for assim, acho que a saída é pensar nos intervalos onde o zero não
> está presente...
> Acho que meu erro foi pensar que em intervalos fechados existem, com
> certeza, máximos e mínimos locais...
> Vou continuar pensando e escrevo se tiver novidades.
> Abraços!
> Luiz
>
> On Sat, Nov 2, 2019, 2:55 PM Rodrigo Ângelo 
> wrote:
>
>> Luiz,
>>
>> Quando x tende a zero pela direita e pela esquerda, f tende a mais e
>> menos infinito, respetivamente.
>>
>> À rigor, a função não tem máximo nem mínimo, porque para todo x no
>> domínio da f, é possível encontrar um xmax e um xmin tais que f(xmax) >
>> f(x) e f(xmin) < f(x).
>>
>> Dito isso, eu responderia a alternativa c) por ser a única que contém o
>> zero.
>>
>> On Sat, Nov 2, 2019, 13:53 Luiz Antonio Rodrigues 
>> wrote:
>>
>>> Olá, pessoal!
>>> Bom dia!
>>> Estou tentando resolver o seguinte problema:
>>>
>>> É dada a função:
>>>
>>> f(x)=(1/x)+sen(x)
>>>
>>> Pergunta-se:
>>>
>>> Em quais intervalos abaixo é garantido que encontremos o máximo e o
>>> mínimo desta função?
>>>
>>> a) [-12;-3]
>>> b) (-2;-1)
>>> c) [-pi;pi]
>>> d) [pi;2pi]
>>> e) [5;+ infinito)
>>>
>>> Eu só consegui encontrar um ponto crítico em x=0.
>>> Ele não é o único, pois vi isso num gráfico da função.
>>> Não sei como resolver a equação f'(x)=0.
>>> Acho que estamos lidando com números complexos.
>>> Intervalos fechados fazem parte da solução?
>>> Pergunto isso porque foi minha resposta, que está errada.
>>> Estou confuso.
>>> Alguém pode me ajudar?
>>> Muito obrigado e um abraço!
>>> Luiz
>>>
>>>
>>>
>>> --
>>> Esta mensagem foi verificada pelo sistema de antivírus e
>>> acredita-se estar livre de perigo.
>>
>>
>> --
>> Esta mensagem foi verificada pelo sistema de antivírus e
>> acredita-se estar livre de perigo.
>
>
> --
> Esta mensagem foi verificada pelo sistema de antivírus e
> acredita-se estar livre de perigo.

-- 
Esta mensagem foi verificada pelo sistema de antiv�rus e
 acredita-se estar livre de perigo.



[obm-l] Re: [obm-l] Re: [obm-l] Re: [obm-l] Re: [obm-l] Re: [obm-l] Domínio de uma Função

2019-10-30 Por tôpico Pedro José
Boa tarde!
Faltara mencionar que o máximo também era local.
Quando eu falo vizinhança de 0, é 0+ e 0-
Se você observar para 0+ temos a primeira derivada positiva, logo a função
é crescente. x^(-1/3)+1
Para 0- o primeiro termo se sobressai ao segundo e a derivada é negativa,
logo o valor também cresce se andarmos para esquerda, logo é mínimo local.
Mas se a função tiver um comportamento monótono, tanto a esquerda quanto a
direita do ponto, num intervalo mais amplo, você pode usar esse intervalo
para analisar.
Só lembre que se na prova, caso você seja estudante, não pode usar excel.

Saudações,
PJMS

Em ter, 29 de out de 2019 às 21:54, Luiz Antonio Rodrigues <
rodrigue...@gmail.com> escreveu:

> Olá, Pedro!
> Olá, Claudio!
> Tudo bem?
> Sim, cheguei agora há pouco nestes valores para máximo e mínimo locais.
> Muito obrigado!
> E você citou a minha próxima dúvida: existe um tamanho "ideal" para o
> intervalo na vizinhança do ponto crítico?
> Eu sei que ele não pode ser muito grande...
> Mas ele pode ser bem pequeno?
> Por exemplo, se o ponto crítico tiver x=5, o intervalo pode ser (4,6)?
> E (4.5,5.5)?
> Fiz várias pesquisas na internet e continuo confuso...
> Claudio, vou seguir sua sugestão para analisar outras funções utiluzando o
> Excel.
> Muito obrigado pela ajuda!
> Luiz
>
>
>
> On Tue, Oct 29, 2019, 7:38 PM Pedro José  wrote:
>
>> Boa noite.
>> Não tinha conhecimento do fato citado por Ralph;
>> Mas essa função tem um mínimo local em x=0 e um máximo em x=-1
>> No ponto x=1 a segunda derivada é negativa, Em x=0 não existe a primeira
>> derivad, tem que fazer análise da vizinhança do ponto.
>>
>> Saudações,
>> PJMS
>>
>> Em ter, 29 de out de 2019 às 12:29, Claudio Buffara <
>> claudio.buff...@gmail.com> escreveu:
>>
>>> Use uma planilha. Eu acho melhor pra analisar funções.
>>>
>>> Enviado do meu iPhone
>>>
>>> Em 29 de out de 2019, à(s) 11:23, Luiz Antonio Rodrigues <
>>> rodrigue...@gmail.com> escreveu:
>>>
>>> 
>>> Olá, Claudio!
>>> Bom dia!
>>> Foi assim que eu pensei também...
>>> Não entendi por que a calculadora gráfica indicou domínio [0, +
>>> infinito).
>>> Vou verificar tudo novamente...
>>> Muito obrigado pela ajuda!Â
>>> Abraço!
>>> Luiz
>>>
>>> On Tue, Oct 29, 2019, 10:49 AM Claudio Buffara <
>>> claudio.buff...@gmail.com> wrote:
>>>
>>>> Estritamente falando, o domínio da função não foi definido.
>>>> Nestes casos, o usual é tomar por domínio o maior subconjunto de R no
>>>> qual a fórmula faz sentido.
>>>> E, neste caso específico, a fórmula faz sentido para todo x real.
>>>>
>>>> O gráfico de h(x) = x^(2/3) tem uma "ponta" em x = 0, de modo que  a
>>>> derivada h'(x) não é definida na origem.
>>>>
>>>> Mas não deveria haver problema algum em x = -1.
>>>>
>>>>
>>>> On Tue, Oct 29, 2019 at 4:57 AM Luiz Antonio Rodrigues <
>>>> rodrigue...@gmail.com> wrote:
>>>>
>>>>> Olá, pessoal!
>>>>> Tudo bem?
>>>>> Estou tentando descobrir os pontos  de máximo e mínimo da função:
>>>>>
>>>>> f(x)=1.5*(x)^(2/3)+x
>>>>>
>>>>> A primeira derivada se anula em x=-1.
>>>>> Mas porque -1 não pertence ao domínio da função?
>>>>> Vi isso numa calculadora gráfica.
>>>>> Eu não consigo entender isso...
>>>>> Não estou tirando a raiz cúbica de um número ao quadrado?
>>>>> Alguém pode me ajudar?
>>>>> Muito obrigado!
>>>>> Luiz
>>>>>
>>>>> --
>>>>> Esta mensagem foi verificada pelo sistema de antivírus e
>>>>> acredita-se estar livre de perigo.
>>>>
>>>>
>>>> --
>>>> Esta mensagem foi verificada pelo sistema de antivírus e
>>>> acredita-se estar livre de perigo.
>>>
>>>
>>> --
>>> Esta mensagem foi verificada pelo sistema de antivírus e
>>> acredita-se estar livre de perigo.
>>>
>>>
>>> --
>>> Esta mensagem foi verificada pelo sistema de antivírus e
>>> acredita-se estar livre de perigo.
>>>
>>
>> --
>> Esta mensagem foi verificada pelo sistema de antivírus e
>> acredita-se estar livre de perigo.
>
>
> --
> Esta mensagem foi verificada pelo sistema de antivírus e
> acredita-se estar livre de perigo.

-- 
Esta mensagem foi verificada pelo sistema de antiv�rus e
 acredita-se estar livre de perigo.



[obm-l] Re: [obm-l] Re: [obm-l] Re: [obm-l] Domínio de uma Função

2019-10-29 Por tôpico Pedro José
Boa noite.
Não tinha conhecimento do fato citado por Ralph;
Mas essa função tem um mínimo local em x=0 e um máximo em x=-1
No ponto x=1 a segunda derivada é negativa, Em x=0 não existe a primeira
derivad, tem que fazer análise da vizinhança do ponto.

Saudações,
PJMS

Em ter, 29 de out de 2019 às 12:29, Claudio Buffara <
claudio.buff...@gmail.com> escreveu:

> Use uma planilha. Eu acho melhor pra analisar funções.
>
> Enviado do meu iPhone
>
> Em 29 de out de 2019, à(s) 11:23, Luiz Antonio Rodrigues <
> rodrigue...@gmail.com> escreveu:
>
> 
> Olá, Claudio!
> Bom dia!
> Foi assim que eu pensei também...
> Não entendi por que a calculadora gráfica indicou domínio [0, +
> infinito).
> Vou verificar tudo novamente...
> Muito obrigado pela ajuda!Â
> Abraço!
> Luiz
>
> On Tue, Oct 29, 2019, 10:49 AM Claudio Buffara 
> wrote:
>
>> Estritamente falando, o domínio da função não foi definido.
>> Nestes casos, o usual é tomar por domínio o maior subconjunto de R no
>> qual a fórmula faz sentido.
>> E, neste caso específico, a fórmula faz sentido para todo x real.
>>
>> O gráfico de h(x) = x^(2/3) tem uma "ponta" em x = 0, de modo que  a
>> derivada h'(x) não é definida na origem.
>>
>> Mas não deveria haver problema algum em x = -1.
>>
>>
>> On Tue, Oct 29, 2019 at 4:57 AM Luiz Antonio Rodrigues <
>> rodrigue...@gmail.com> wrote:
>>
>>> Olá, pessoal!
>>> Tudo bem?
>>> Estou tentando descobrir os pontos  de máximo e mínimo da função:
>>>
>>> f(x)=1.5*(x)^(2/3)+x
>>>
>>> A primeira derivada se anula em x=-1.
>>> Mas porque -1 não pertence ao domínio da função?
>>> Vi isso numa calculadora gráfica.
>>> Eu não consigo entender isso...
>>> Não estou tirando a raiz cúbica de um número ao quadrado?
>>> Alguém pode me ajudar?
>>> Muito obrigado!
>>> Luiz
>>>
>>> --
>>> Esta mensagem foi verificada pelo sistema de antivírus e
>>> acredita-se estar livre de perigo.
>>
>>
>> --
>> Esta mensagem foi verificada pelo sistema de antivírus e
>> acredita-se estar livre de perigo.
>
>
> --
> Esta mensagem foi verificada pelo sistema de antivírus e
> acredita-se estar livre de perigo.
>
>
> --
> Esta mensagem foi verificada pelo sistema de antivírus e
> acredita-se estar livre de perigo.
>

-- 
Esta mensagem foi verificada pelo sistema de antiv�rus e
 acredita-se estar livre de perigo.



[obm-l] Re: [obm-l] Congruência

2019-10-25 Por tôpico Pedro José
 Boa tarde!
Primeiramente, temos que considerar k positivo.
Depois temos que calcular  ord19 10
Mas ord19 10 | Fi(19)=18 então os possíveis valores são: 1, 2, 3, 6, 9 ,18.
Pois, ord19 10| Fi(19)
10^1=10; 1 não atente
10^2= 100= 5 mod19; 2 não atende
10^3= 5*10= 12 mod 19; 3 não atende
10^6= (10^3)^2= 144= 11 mod19, 6 não atende
10^9=10^3*10^6=132= 18 mod19; 9 não atende. Portanto, ord19 10=Fi(19)=18,
ou seja, 10 é uma raiz primitiva  mod19.
se 10^ko =2 ==>10^(ko+n* ord19 10)= 2
Mas 2.10= 1 mod19 ==> Portanto, 10^18=2*10 mod 19; e (19,10)=1, temos que
10^17=2 mod 19; portanto k=17 é o primeira solução positiva.
Pois, se existisse um k< 17, com 10^k=2, teríamos que 10^(k+1)=1 com k+1
<18 = ord19 10, absurdo.
Então a primeira é para k=17
E as seguintes, 35 e 53.
Note que foi necessário restringir k como positivo, pois, 10^-1, 10^-19,
10^-37, 10^-55... são soluções
Não sei se ficou claro, mas se houvesse um período p menor que 18 = ord19
10. 10^xo =10^(xo+p) mod19, teríamos 10^p=1 mod19, com p< 18 = ord19 1;
absurdo.
Saudações,
PJMS


Em sex, 25 de out de 2019 às 00:15, marcone augusto araújo borges <
marconeborge...@hotmail.com> escreveu:

> Quais são as 3 primeiras soluções da congruência 10^k = = 2 (mod 19)?
> --
> Esta mensagem foi verificada pelo sistema de antivírus e
> acredita-se estar livre de perigo.
>

-- 
Esta mensagem foi verificada pelo sistema de antiv�rus e
 acredita-se estar livre de perigo.



[obm-l] Re: [obm-l] Congruência

2019-10-25 Por tôpico Pedro José
Boa tarde!
Primeiramente, temos que calcular  ord19 10 .
Mas ord19 10 | Fi(19)=18 então os possíveis valores são: 1, 2, 3, 6, 9 ,18
1 não atente
10^2= 100= 5 mod19; 2 não atende
10^3= 5*10= 12 mod 19; 3 não atende
10^6= 5*12 =

Em sex, 25 de out de 2019 às 00:15, marcone augusto araújo borges <
marconeborge...@hotmail.com> escreveu:

> Quais são as 3 primeiras soluções da congruência 10^k = = 2 (mod 19)?
> --
> Esta mensagem foi verificada pelo sistema de antivírus e
> acredita-se estar livre de perigo.
>

-- 
Esta mensagem foi verificada pelo sistema de antiv�rus e
 acredita-se estar livre de perigo.



[obm-l] Re: [obm-l] Re: [obm-l] Retas envolvendo uma parábola

2019-10-15 Por tôpico Pedro José
Boa tarde!
Acho estranho. Pois, se um ponto iniciar de uma distância doa do ponto de
interseção sobre uma reta e o outro de dob do ponto interseção e ambos com
o mesmo sentido. E se doa/va = dob/vb, vai gerar um feixe de retas
paralelas, sendo va e vb a velocidade dos pontos, que não é esperado para
um envelope de uma parábola.

Aguardo a solução do problema.

Saudações,
PJMS.

Em sáb, 12 de out de 2019 às 02:04, Ralph Teixeira 
escreveu:

> Eu penso "EDO de Clairaut", que fornece uma maneira de encontrar o
> envelope de uma familia de retas dadas.
>
> Abraco, Ralph.
>
> On Fri, Oct 11, 2019 at 10:40 PM Luís Lopes  wrote:
>
>> Sauda,c~oes,
>>
>> Numa troca de mensagens sobre um procedimento de resolução do problema
>> "construir um triângulo ABC dados " apareceu a seguinte
>> afirmação:
>>
>> When two points move along two intersecting
>> straight lines at constant speeds (not necessarily equal)
>> line connecting them envelopes a parabola.
>>
>> Alguém poderia me dar a prova ou referências sobre isso ?
>>
>> Obrigado.
>>
>> Luís
>>
>>
>>
>> --
>> Esta mensagem foi verificada pelo sistema de antivírus e
>> acredita-se estar livre de perigo.
>>
>
> --
> Esta mensagem foi verificada pelo sistema de antivírus e
> acredita-se estar livre de perigo.

-- 
Esta mensagem foi verificada pelo sistema de antiv�rus e
 acredita-se estar livre de perigo.



[obm-l] Re: [obm-l] Re: [obm-l] Re: [obm-l] Congruência (?)

2019-10-09 Por tôpico Pedro José
Boa noite!
Faltara também a explicação.
Seja a = r  mod 10 então a^n=(r)^n  mod 100  se n é múltiplo de 10.
Mas é só usar o binômio de Newton, para (10q+r)^n  só sobra o último termo.

Saudações.

Em qua, 9 de out de 2019 às 11:09, Pedro José 
escreveu:

> Bom dia!
>
> Achei um outro modo de resolver, só que ao retornar me apercebi de que
> "engolira a classe 6', ao invés de ir na PA(2,4,6,8) segui pela PG (2,4,8)
>
> Faltou então para o algarismo 6.
>
> 6^20=2^20.3^20 e ord1003=20então 2^20= 1 mod 100 então 6=^20=2^20 mod
> 100
> Se 3^n= 1 mod100 então 3^n= 1 mod10
> ord103=4
> (3)^n=1 mod100 então né múltiplo de 4. Então n=4k par k>1 inteiro.
> (3)^n=(81)^k=(10*8+1)^k
> Pelo binômio de Newton, só sobram os dois últimos termos. Os demais terão
> 10^m com m>2 que côngruo de 0  mod100
> k.10*8 +1, e portanto o menor k que satisfaz é k=5. Então ord1003=20
>
> Com isso completa o que faltara da resolução anterior.
>
> 2^10=1024=24 mod100
> 2^20=24^2=76 mod100
> 4^20=(2^20)^2=76^2=(-24)^2=576=76 mod100
> 8^20=2^20.4^20=76^2=24 mod100
> 6^20=3^20.2^20=2^20 pois ord1003=20
>
> Essa última ficou melhor.
>
> Saudações,
> PJMS
>
>
> Em sáb, 5 de out de 2019 às 08:58, Pedro José 
> escreveu:
>
>> Bom dia!
>> Esdras, tem como postar a resposta.
>> Não consigo ver a^p=a modp, para p primo se encaixando no problema, pois
>> 10 não é primo.
>>
>> Grato!
>>
>> Saudações,
>> PJMS
>>
>>
>> Em sex, 4 de out de 2019 às 20:20, Esdras Muniz <
>> esdrasmunizm...@gmail.com> escreveu:
>>
>>> Dá pra fazer tb usando o pequeno teorema de Fermat.
>>>
>>>
>>> <https://www.avast.com/sig-email?utm_medium=email_source=link_campaign=sig-email_content=webmail>
>>>  Livre
>>> de vírus. www.avast.com
>>> <https://www.avast.com/sig-email?utm_medium=email_source=link_campaign=sig-email_content=webmail>.
>>>
>>> <#m_3285326544539962876_m_-140568092169550719_m_1063528150960112747_m_-1601668305501320773_m_-5542290881960747167_m_-611650024147786599_DAB4FAD8-2DD7-40BB-A1B8-4E2AA1F9FDF2>
>>>
>>> Em sex, 4 de out de 2019 às 17:36, Pedro José 
>>> escreveu:
>>>
>>>> Boa tarde!
>>>> Com minhas escusas retificação da solução.
>>>> n<>o mod10 e não: "n<> 0 mod100"
>>>> (100,4) <>1 e não: "(100,4) =1"
>>>> b^x não se repete e não: "b^x não se repetem"
>>>> Sds,
>>>> PJMS.
>>>>
>>>>
>>>> Em sex, 4 de out de 2019 às 17:16, Pedro José 
>>>> escreveu:
>>>>
>>>>> Boa tarde!
>>>>> Se 10 não divide n então n<>0 mod100; pois nesse caso daria "00".
>>>>> Então os números são 2,4 ou 8 côngruos mod10.
>>>>> 2^20=4^10
>>>>> 8^20 = 4^40
>>>>> 4^1= 4 mod10
>>>>> 4^2=6 mod10
>>>>> 4^3= 4 mod10
>>>>> Logo temos que 4^(2m+1) = 4 mod 10 (i)
>>>>> Se
>>>>> a=4 mod 100 ==> a=4 mod 10 (ii)
>>>>>
>>>>> Então vamos procurar o período de a^n mod100,  Não existe a que
>>>>> satisfaça a^m= 1 mod100, com m<>0, pois (100,4)=1
>>>>> Vamos tentar verificar se há repetição do 4.
>>>>> De (i) e (ii) , temos que: 4^(2m+1) = 4 mod 100
>>>>> m=1 ==> 4^3 = 64 mod 104, não serve
>>>>> m=2 ==> 4^5= (4^(3*2))*8 = 28*8= 224=24 mod 100, não serve
>>>>> m=3 ==> 4^7= 24*16=384=84 mod 100
>>>>> m=4 ==> 4^9= (2*84)*8=68*8= 544=44 mod100
>>>>> m=5 ==> 4^11=44*16= 704= 4 mod 100
>>>>> Portanto o período de 4^a mod100 é 1gual a 10, ou seja, 4^a=4(10x+a)
>>>>> mod100. com x,a não nulos (Cuidado, que para alguns casos em que (b,m)<>1,
>>>>> b^x não se repetem para x < xo,e.g., 2^a= 2 mod 100, só é atendido para
>>>>> a=1, aí tem de sair no braço para ver qual que se repete e pode-se gastar
>>>>> mais tempo. Por sorte o quatro repetiu. Mas o enunciado dava a dica de que
>>>>> repetiria, pois, 4^20=4^10 mod 100 para que o problema tenha uma solução
>>>>> única.
>>>>> 4^20 = 4^10= 4^9*4=44*4=176=76 mod100
>>>>> 8^20=4^40=4^10=76 mod100
>>>>> 2^20=4^10=76 mod 100.
>>>>>
>>>>> Portanto o algarismo da dezena é 7 e das unidades 6.
>>>>>
>>>>> Saudações,
>>>>> PJMS
>>>>>
>>>>>
>>>>>
>>>>>
>>>>> Em qui, 3 de out de 2019 às 17:51, marcone augusto araújo borges <
>>>>> marconeborge...@hotmail.com> escreveu:
>>>>>
>>>>>> Se n é um número natural par não divisível por 10, quais são os dois
>>>>>> últimos algarismos de n^20?
>>>>>> --
>>>>>> Esta mensagem foi verificada pelo sistema de antivírus e
>>>>>> acredita-se estar livre de perigo.
>>>>>>
>>>>>
>>>> --
>>>> Esta mensagem foi verificada pelo sistema de antivírus e
>>>> acredita-se estar livre de perigo.
>>>
>>>
>>>
>>> --
>>> Esdras Muniz Mota
>>> Mestrando em Matemática
>>> Universidade Federal do Ceará
>>>
>>>
>>>
>>> --
>>> Esta mensagem foi verificada pelo sistema de antivírus e
>>> acredita-se estar livre de perigo.
>>
>>

-- 
Esta mensagem foi verificada pelo sistema de antiv�rus e
 acredita-se estar livre de perigo.



[obm-l] Re: [obm-l] Re: [obm-l] Re: [obm-l] Congruência (?)

2019-10-09 Por tôpico Pedro José
Bom dia!

Achei um outro modo de resolver, só que ao retornar me apercebi de que
"engolira a classe 6', ao invés de ir na PA(2,4,6,8) segui pela PG (2,4,8)

Faltou então para o algarismo 6.

6^20=2^20.3^20 e ord1003=20então 2^20= 1 mod 100 então 6=^20=2^20 mod100
Se 3^n= 1 mod100 então 3^n= 1 mod10
ord103=4
(3)^n=1 mod100 então né múltiplo de 4. Então n=4k par k>1 inteiro.
(3)^n=(81)^k=(10*8+1)^k
Pelo binômio de Newton, só sobram os dois últimos termos. Os demais terão
10^m com m>2 que côngruo de 0  mod100
k.10*8 +1, e portanto o menor k que satisfaz é k=5. Então ord1003=20

Com isso completa o que faltara da resolução anterior.

2^10=1024=24 mod100
2^20=24^2=76 mod100
4^20=(2^20)^2=76^2=(-24)^2=576=76 mod100
8^20=2^20.4^20=76^2=24 mod100
6^20=3^20.2^20=2^20 pois ord1003=20

Essa última ficou melhor.

Saudações,
PJMS


Em sáb, 5 de out de 2019 às 08:58, Pedro José 
escreveu:

> Bom dia!
> Esdras, tem como postar a resposta.
> Não consigo ver a^p=a modp, para p primo se encaixando no problema, pois
> 10 não é primo.
>
> Grato!
>
> Saudações,
> PJMS
>
>
> Em sex, 4 de out de 2019 às 20:20, Esdras Muniz 
> escreveu:
>
>> Dá pra fazer tb usando o pequeno teorema de Fermat.
>>
>>
>> <https://www.avast.com/sig-email?utm_medium=email_source=link_campaign=sig-email_content=webmail>
>>  Livre
>> de vírus. www.avast.com
>> <https://www.avast.com/sig-email?utm_medium=email_source=link_campaign=sig-email_content=webmail>.
>>
>> <#m_-140568092169550719_m_1063528150960112747_m_-1601668305501320773_m_-5542290881960747167_m_-611650024147786599_DAB4FAD8-2DD7-40BB-A1B8-4E2AA1F9FDF2>
>>
>> Em sex, 4 de out de 2019 às 17:36, Pedro José 
>> escreveu:
>>
>>> Boa tarde!
>>> Com minhas escusas retificação da solução.
>>> n<>o mod10 e não: "n<> 0 mod100"
>>> (100,4) <>1 e não: "(100,4) =1"
>>> b^x não se repete e não: "b^x não se repetem"
>>> Sds,
>>> PJMS.
>>>
>>>
>>> Em sex, 4 de out de 2019 às 17:16, Pedro José 
>>> escreveu:
>>>
>>>> Boa tarde!
>>>> Se 10 não divide n então n<>0 mod100; pois nesse caso daria "00".
>>>> Então os números são 2,4 ou 8 côngruos mod10.
>>>> 2^20=4^10
>>>> 8^20 = 4^40
>>>> 4^1= 4 mod10
>>>> 4^2=6 mod10
>>>> 4^3= 4 mod10
>>>> Logo temos que 4^(2m+1) = 4 mod 10 (i)
>>>> Se
>>>> a=4 mod 100 ==> a=4 mod 10 (ii)
>>>>
>>>> Então vamos procurar o período de a^n mod100,  Não existe a que
>>>> satisfaça a^m= 1 mod100, com m<>0, pois (100,4)=1
>>>> Vamos tentar verificar se há repetição do 4.
>>>> De (i) e (ii) , temos que: 4^(2m+1) = 4 mod 100
>>>> m=1 ==> 4^3 = 64 mod 104, não serve
>>>> m=2 ==> 4^5= (4^(3*2))*8 = 28*8= 224=24 mod 100, não serve
>>>> m=3 ==> 4^7= 24*16=384=84 mod 100
>>>> m=4 ==> 4^9= (2*84)*8=68*8= 544=44 mod100
>>>> m=5 ==> 4^11=44*16= 704= 4 mod 100
>>>> Portanto o período de 4^a mod100 é 1gual a 10, ou seja, 4^a=4(10x+a)
>>>> mod100. com x,a não nulos (Cuidado, que para alguns casos em que (b,m)<>1,
>>>> b^x não se repetem para x < xo,e.g., 2^a= 2 mod 100, só é atendido para
>>>> a=1, aí tem de sair no braço para ver qual que se repete e pode-se gastar
>>>> mais tempo. Por sorte o quatro repetiu. Mas o enunciado dava a dica de que
>>>> repetiria, pois, 4^20=4^10 mod 100 para que o problema tenha uma solução
>>>> única.
>>>> 4^20 = 4^10= 4^9*4=44*4=176=76 mod100
>>>> 8^20=4^40=4^10=76 mod100
>>>> 2^20=4^10=76 mod 100.
>>>>
>>>> Portanto o algarismo da dezena é 7 e das unidades 6.
>>>>
>>>> Saudações,
>>>> PJMS
>>>>
>>>>
>>>>
>>>>
>>>> Em qui, 3 de out de 2019 às 17:51, marcone augusto araújo borges <
>>>> marconeborge...@hotmail.com> escreveu:
>>>>
>>>>> Se n é um número natural par não divisível por 10, quais são os dois
>>>>> últimos algarismos de n^20?
>>>>> --
>>>>> Esta mensagem foi verificada pelo sistema de antivírus e
>>>>> acredita-se estar livre de perigo.
>>>>>
>>>>
>>> --
>>> Esta mensagem foi verificada pelo sistema de antivírus e
>>> acredita-se estar livre de perigo.
>>
>>
>>
>> --
>> Esdras Muniz Mota
>> Mestrando em Matemática
>> Universidade Federal do Ceará
>>
>>
>>
>> --
>> Esta mensagem foi verificada pelo sistema de antivírus e
>> acredita-se estar livre de perigo.
>
>

-- 
Esta mensagem foi verificada pelo sistema de antiv�rus e
 acredita-se estar livre de perigo.



[obm-l] Re: [obm-l] Re: [obm-l] Re: [obm-l] Congruência (?)

2019-10-05 Por tôpico Pedro José
Bom dia!
Esdras, tem como postar a resposta.
Não consigo ver a^p=a modp, para p primo se encaixando no problema, pois 10
não é primo.

Grato!

Saudações,
PJMS


Em sex, 4 de out de 2019 às 20:20, Esdras Muniz 
escreveu:

> Dá pra fazer tb usando o pequeno teorema de Fermat.
>
>
> <https://www.avast.com/sig-email?utm_medium=email_source=link_campaign=sig-email_content=webmail>
>  Livre
> de vírus. www.avast.com
> <https://www.avast.com/sig-email?utm_medium=email_source=link_campaign=sig-email_content=webmail>.
>
> <#m_-5542290881960747167_m_-611650024147786599_DAB4FAD8-2DD7-40BB-A1B8-4E2AA1F9FDF2>
>
> Em sex, 4 de out de 2019 às 17:36, Pedro José 
> escreveu:
>
>> Boa tarde!
>> Com minhas escusas retificação da solução.
>> n<>o mod10 e não: "n<> 0 mod100"
>> (100,4) <>1 e não: "(100,4) =1"
>> b^x não se repete e não: "b^x não se repetem"
>> Sds,
>> PJMS.
>>
>>
>> Em sex, 4 de out de 2019 às 17:16, Pedro José 
>> escreveu:
>>
>>> Boa tarde!
>>> Se 10 não divide n então n<>0 mod100; pois nesse caso daria "00".
>>> Então os números são 2,4 ou 8 côngruos mod10.
>>> 2^20=4^10
>>> 8^20 = 4^40
>>> 4^1= 4 mod10
>>> 4^2=6 mod10
>>> 4^3= 4 mod10
>>> Logo temos que 4^(2m+1) = 4 mod 10 (i)
>>> Se
>>> a=4 mod 100 ==> a=4 mod 10 (ii)
>>>
>>> Então vamos procurar o período de a^n mod100,  Não existe a que
>>> satisfaça a^m= 1 mod100, com m<>0, pois (100,4)=1
>>> Vamos tentar verificar se há repetição do 4.
>>> De (i) e (ii) , temos que: 4^(2m+1) = 4 mod 100
>>> m=1 ==> 4^3 = 64 mod 104, não serve
>>> m=2 ==> 4^5= (4^(3*2))*8 = 28*8= 224=24 mod 100, não serve
>>> m=3 ==> 4^7= 24*16=384=84 mod 100
>>> m=4 ==> 4^9= (2*84)*8=68*8= 544=44 mod100
>>> m=5 ==> 4^11=44*16= 704= 4 mod 100
>>> Portanto o período de 4^a mod100 é 1gual a 10, ou seja, 4^a=4(10x+a)
>>> mod100. com x,a não nulos (Cuidado, que para alguns casos em que (b,m)<>1,
>>> b^x não se repetem para x < xo,e.g., 2^a= 2 mod 100, só é atendido para
>>> a=1, aí tem de sair no braço para ver qual que se repete e pode-se gastar
>>> mais tempo. Por sorte o quatro repetiu. Mas o enunciado dava a dica de que
>>> repetiria, pois, 4^20=4^10 mod 100 para que o problema tenha uma solução
>>> única.
>>> 4^20 = 4^10= 4^9*4=44*4=176=76 mod100
>>> 8^20=4^40=4^10=76 mod100
>>> 2^20=4^10=76 mod 100.
>>>
>>> Portanto o algarismo da dezena é 7 e das unidades 6.
>>>
>>> Saudações,
>>> PJMS
>>>
>>>
>>>
>>>
>>> Em qui, 3 de out de 2019 às 17:51, marcone augusto araújo borges <
>>> marconeborge...@hotmail.com> escreveu:
>>>
>>>> Se n é um número natural par não divisível por 10, quais são os dois
>>>> últimos algarismos de n^20?
>>>> --
>>>> Esta mensagem foi verificada pelo sistema de antivírus e
>>>> acredita-se estar livre de perigo.
>>>>
>>>
>> --
>> Esta mensagem foi verificada pelo sistema de antivírus e
>> acredita-se estar livre de perigo.
>
>
>
> --
> Esdras Muniz Mota
> Mestrando em Matemática
> Universidade Federal do Ceará
>
>
>
> --
> Esta mensagem foi verificada pelo sistema de antivírus e
> acredita-se estar livre de perigo.

-- 
Esta mensagem foi verificada pelo sistema de antiv�rus e
 acredita-se estar livre de perigo.



[obm-l] Re: [obm-l] Congruência (?)

2019-10-04 Por tôpico Pedro José
Boa tarde!
Com minhas escusas retificação da solução.
n<>o mod10 e não: "n<> 0 mod100"
(100,4) <>1 e não: "(100,4) =1"
b^x não se repete e não: "b^x não se repetem"
Sds,
PJMS.


Em sex, 4 de out de 2019 às 17:16, Pedro José 
escreveu:

> Boa tarde!
> Se 10 não divide n então n<>0 mod100; pois nesse caso daria "00".
> Então os números são 2,4 ou 8 côngruos mod10.
> 2^20=4^10
> 8^20 = 4^40
> 4^1= 4 mod10
> 4^2=6 mod10
> 4^3= 4 mod10
> Logo temos que 4^(2m+1) = 4 mod 10 (i)
> Se
> a=4 mod 100 ==> a=4 mod 10 (ii)
>
> Então vamos procurar o período de a^n mod100,  Não existe a que satisfaça
> a^m= 1 mod100, com m<>0, pois (100,4)=1
> Vamos tentar verificar se há repetição do 4.
> De (i) e (ii) , temos que: 4^(2m+1) = 4 mod 100
> m=1 ==> 4^3 = 64 mod 104, não serve
> m=2 ==> 4^5= (4^(3*2))*8 = 28*8= 224=24 mod 100, não serve
> m=3 ==> 4^7= 24*16=384=84 mod 100
> m=4 ==> 4^9= (2*84)*8=68*8= 544=44 mod100
> m=5 ==> 4^11=44*16= 704= 4 mod 100
> Portanto o período de 4^a mod100 é 1gual a 10, ou seja, 4^a=4(10x+a)
> mod100. com x,a não nulos (Cuidado, que para alguns casos em que (b,m)<>1,
> b^x não se repetem para x < xo,e.g., 2^a= 2 mod 100, só é atendido para
> a=1, aí tem de sair no braço para ver qual que se repete e pode-se gastar
> mais tempo. Por sorte o quatro repetiu. Mas o enunciado dava a dica de que
> repetiria, pois, 4^20=4^10 mod 100 para que o problema tenha uma solução
> única.
> 4^20 = 4^10= 4^9*4=44*4=176=76 mod100
> 8^20=4^40=4^10=76 mod100
> 2^20=4^10=76 mod 100.
>
> Portanto o algarismo da dezena é 7 e das unidades 6.
>
> Saudações,
> PJMS
>
>
>
>
> Em qui, 3 de out de 2019 às 17:51, marcone augusto araújo borges <
> marconeborge...@hotmail.com> escreveu:
>
>> Se n é um número natural par não divisível por 10, quais são os dois
>> últimos algarismos de n^20?
>> --
>> Esta mensagem foi verificada pelo sistema de antivírus e
>> acredita-se estar livre de perigo.
>>
>

-- 
Esta mensagem foi verificada pelo sistema de antiv�rus e
 acredita-se estar livre de perigo.



[obm-l] Re: [obm-l] Congruência (?)

2019-10-04 Por tôpico Pedro José
Boa tarde!
Se 10 não divide n então n<>0 mod100; pois nesse caso daria "00".
Então os números são 2,4 ou 8 côngruos mod10.
2^20=4^10
8^20 = 4^40
4^1= 4 mod10
4^2=6 mod10
4^3= 4 mod10
Logo temos que 4^(2m+1) = 4 mod 10 (i)
Se
a=4 mod 100 ==> a=4 mod 10 (ii)

Então vamos procurar o período de a^n mod100,  Não existe a que satisfaça
a^m= 1 mod100, com m<>0, pois (100,4)=1
Vamos tentar verificar se há repetição do 4.
De (i) e (ii) , temos que: 4^(2m+1) = 4 mod 100
m=1 ==> 4^3 = 64 mod 104, não serve
m=2 ==> 4^5= (4^(3*2))*8 = 28*8= 224=24 mod 100, não serve
m=3 ==> 4^7= 24*16=384=84 mod 100
m=4 ==> 4^9= (2*84)*8=68*8= 544=44 mod100
m=5 ==> 4^11=44*16= 704= 4 mod 100
Portanto o período de 4^a mod100 é 1gual a 10, ou seja, 4^a=4(10x+a)
mod100. com x,a não nulos (Cuidado, que para alguns casos em que (b,m)<>1,
b^x não se repetem para x < xo,e.g., 2^a= 2 mod 100, só é atendido para
a=1, aí tem de sair no braço para ver qual que se repete e pode-se gastar
mais tempo. Por sorte o quatro repetiu. Mas o enunciado dava a dica de que
repetiria, pois, 4^20=4^10 mod 100 para que o problema tenha uma solução
única.
4^20 = 4^10= 4^9*4=44*4=176=76 mod100
8^20=4^40=4^10=76 mod100
2^20=4^10=76 mod 100.

Portanto o algarismo da dezena é 7 e das unidades 6.

Saudações,
PJMS




Em qui, 3 de out de 2019 às 17:51, marcone augusto araújo borges <
marconeborge...@hotmail.com> escreveu:

> Se n é um número natural par não divisível por 10, quais são os dois
> últimos algarismos de n^20?
> --
> Esta mensagem foi verificada pelo sistema de antivírus e
> acredita-se estar livre de perigo.
>

-- 
Esta mensagem foi verificada pelo sistema de antiv�rus e
 acredita-se estar livre de perigo.



Re: [obm-l] Triplas pitagoricas

2019-08-27 Por tôpico Pedro José
Bom dia!
Faltou que st=ab, também.

desculpem-me

Saudações,
PJMS

Em ter, 27 de ago de 2019 às 09:29, Pedro José 
escreveu:

> Bom dia!
>
> Para haver solução, tem de haver s,t,a,b estritamente naturais. Com
> (s,t)=1 s ímpar e t par. (a,b)=1 , paridade de a <> paridade de b e
> a^2+b^2=s^2-t^2.
>
> Tentei achar uma restrição que impossibilitasse, mas não consegui.
>
> Talvez ajude.
>
> Saudações,
> PJMS
>
>
>
>
> Em dom, 25 de ago de 2019 às 21:41, Joao Breno 
> escreveu:
>
>> Eu tô achando que o enunciado dessa questão está mal formulado.
>> Nessa questão é pra considerar o zero ou não?
>> Obs.: Alguns autores consideram o zero como sendo um natural e outros não.
>>
>> Att, Breno.
>>
>> Em ter, 13 de ago de 2019 19:29, Jeferson Almir 
>> escreveu:
>>
>>> Como eu provo que não existem 2 naturais cuja soma e diferença de seus
>>> quadrados sejam quadrados ?
>>>
>>> Ps: eu tentei pegar a solução clássica da equação da soma x^2 + y^2 =
>>> z^2 e tentei jogar na diferença pra aparecer algum absurdo em algum módulo
>>> mas obtive sucesso.
>>>
>>> --
>>> Esta mensagem foi verificada pelo sistema de antivírus e
>>> acredita-se estar livre de perigo.
>>
>>
>> --
>> Esta mensagem foi verificada pelo sistema de antivírus e
>> acredita-se estar livre de perigo.
>
>

-- 
Esta mensagem foi verificada pelo sistema de antiv�rus e
 acredita-se estar livre de perigo.



  1   2   3   4   5   6   >